nle step 2_2009 si115-116 and nle_step_2_2009 nctms editors cut key

59
SIRIRAJ SIRIRAJ SIRIRAJ ชุดที1 Medicine 1.) ตอบ 4. Defibillation Ventricular fibrillation - EKG : no p wave ,no QRS cpx can be identified, irregular rhythm ,rate >300 Management : VF and pulseless : Defibillation เพิ่มเติม VF และคลํา pulseได 1.vital sign stable : amiodarone 2.vital sign unstable(shock) : cardioversion 2.) ตอบ 1. 3 rd heart block 3 rd heart block พบวา P กับ QRS จะไมสัมพันธกัน และจะมีอาการไดบอย , 2 nd mobitz I พบวา PR ยาวขึ้นเรื่อยๆ จน QRS หายไปเปนบางครั้งแลวจึงกลับมาเปนปกติ ไม คอยมีอาการ, 2 nd mobitz II พบวา PR ยาวเทาๆเดิม แตอยูQRS ก็หายไป พบอาการได บาง, 1 st degree พบ PR ยาวขึ้นเฉยๆ แลวก็ไมมีอาการ 3.) ตอบ 4. adrenaline im Anaphylaxis criteria dx มี 3วิธี 1.มีอาการทางskin + Rs dysfn or BP drop 2.มี Hx สัมผัสสารที่นาจะเปน allergen + อาการ 2/4 (1.skin or mucosa 2.RS dysfn 3. BP drop 4.GI) 3.มี Hx สัมผัส known allergen + SBP < 90 หรือ ลดลงจาก30% baseline Management : adrenaline im 1:1000 dose 0.01 ml/kg ( child max 0.3ml, adult 0.5ml) ใหซ้ําได 1-2 ครั้ง หางกัน 10-15 นาที 4.) ตอบ 5. Paroxysmal nocturnal hemoglobinuria(PNH) PBS : NCNC / เปน intravascular hemolysis (ปสสาวะมีสีชาหรือโคลา) , อาการ แสดง : pancytopenia

Upload: loveis1able-khumpuangdee

Post on 20-Aug-2015

4.093 views

Category:

Documents


18 download

TRANSCRIPT

Page 1: Nle step 2_2009 si115-116 and nle_step_2_2009 nctms editors cut key

SIRIRAJ

SIRIRA

J SIRIR

AJ

ชุดที่ 1 Medicine

1.) ตอบ 4. Defibillation

Ventricular fibrillation - EKG : no p wave ,no QRS cpx can be identified, irregular

rhythm ,rate >300

Management : VF and pulseless : Defibillation

เพิ่มเติม VF และคลํา pulseได

1.vital sign stable : amiodarone

2.vital sign unstable(shock) : cardioversion

2.) ตอบ 1. 3rd heart block

3rd heart block พบวา P กับ QRS จะไมสัมพันธกัน และจะมีอาการไดบอย, 2nd

mobitz I พบวา PR ยาวขึ้นเรื่อยๆ จน QRS หายไปเปนบางครั้งแลวจึงกลับมาเปนปกติ ไม

คอยมีอาการ, 2nd mobitz II พบวา PR ยาวเทาๆเดิม แตอยูๆ QRS ก็หายไป พบอาการได

บาง, 1st degree พบ PR ยาวขึ้นเฉยๆ แลวก็ไมมีอาการ

3.) ตอบ 4. adrenaline im

Anaphylaxis criteria dx มี 3วิธี

1.มีอาการทางskin + Rs dysfn or BP drop

2.มี Hx สัมผัสสารที่นาจะเปน allergen + อาการ 2/4 (1.skin or mucosa 2.RS dysfn 3.

BP drop 4.GI)

3.มี Hx สัมผัส known allergen + SBP < 90 หรือ ลดลงจาก30% baseline

Management : adrenaline im 1:1000 dose 0.01 ml/kg

( child max 0.3ml, adult 0.5ml) ใหซ้ําได 1-2 ครั้ง หางกัน 10-15 นาที

4.) ตอบ 5. Paroxysmal nocturnal hemoglobinuria(PNH)

PBS : NCNC / เปน intravascular hemolysis (ปสสาวะมีสีชาหรือโคลา) , อาการ

แสดง : pancytopenia

Page 2: Nle step 2_2009 si115-116 and nle_step_2_2009 nctms editors cut key

SIRIRAJ

SIRIRA

J SIRIR

AJ

การวินิจฉัยจะตองทํา Ham’s test (acid serum test) หรือ sucrose hemolysis test

choice ขออื่น AIHA , HS ,Thalassemia เปน extravascular hemolysis

G-6-PD เปน intravascular hemolysis / PBS : bite cell ,Heinz bodies

มักเปนผูชาย(XR) และมักมีประวัติ drug-induced

5.) ตอบ 3. AIHA

จาก ญ วัยรุน ซีด เหลืองเล็กนอย ตับไมโต มามโตเล็กนอย reti ขึ้น (แตทําไมMVC

เล็ก ซึ่งจะนึกถึง thalassemia, iron def. หากจะตอบ thalassemia ตองเปน H disease ที่

สามารถมี reti ขึ้นไดประมาณ 5 %)

Thalassemia ไมมีอาการ จนถึง ซีดเหลืองรุนแรง ตับมามโต chipmuck face

hyperpigmentation MCV เล็ก

Iron def ซีด เหนื่อย มีประวัติเสียเลือดเรื้อรัง glossitis, koilonychias, angular

stomatitis,Wbc,plt ปกติ พบ serum ferritin, Serum Iron, Transferrin Saturation ต่ํา TIBC

สูง MVC เล็ก

G6PD มักพบใน ช> ญ ไมพบมามโต ซีดฉับพลันและเปนชั่วคราว อาจเหลือง

เล็กนอย อาจมีปสสาวะดํา พบ Heinz body, ghost cell

HS มีอาการรุนแรงเมื่อมีภาวะติดเชื้อหรือมีไข มาพบแพทยดวย เหลือง มักพบมามโต

มักพบภาวะแทรกซอนเปนนิ่วในถุงน้ําดี พบ spherocyte, reticulocyte มากขึ้น

AIHA warm type – ญ>ช อายุ 20-40 ตรวจพบโลหิตจาง เหลืองเล็กนอย อาจจบมาม

หรือตับโตเล็กนอย reticulocyte มากขึ้น wbc ต่ํา หรือ สูงขึ้น plt ปกติ

6.) ตอบ 5. FFP

I/C of FFP - 1.replace of multiple coagulation factors ex. liver disease , DIC ,

massive transfusion(Pt,APTT ratio>1.5x normal)

2.Treatment of TTP, HUS

7.) ตอบ 2. Prostigmine test

เนื่องจากนึกถึง Myasthenia gravis / Hx&PE : fluctuating weakness increased by

exertion, Weakness increases during the day and improves with rest, proximal weakness

greater than distal muscle weakness, ptosis, limited to the EOM

Ix : NEOSTIGMINE TEST ( Prostigmin test) 1-2 mg im evaluate 20-40 mins

Rx : Anticholinesterase medication (oral pyridostigmine) เปน symptomatic

treatment; onset 15-30 mins, last for 3-4 hrs; begin with moderate dose 60mg 3-5 times

per day, maximal dose 120mg every 3 hrs during daytime; adverse effect : increased

muscle weakness and muscarinic effect, antidote is propantheline bromide

8.) ตอบ 1. ขอความยินยอมพอแมเพื่อ LP

ผูปวยเด็กอายุ 8 เดือน มีอาการไขสูง มี Brudzinski’s sign positive จากโจทยทําใหนึก

ถึงภาวะ meningitis มากที่สุดผูปวยรายนี้ อาการที่พบไดบอยของภาวะนี้คือ ไข ปวดศีรษะ

ซึม สับสน คลื่นไสอาเจียน ปวดคอ คอแข็ง ตรวจรางกายอาจพบ ระดับความรูสึกตัวที่

ลดลง และอาการแสดงของ Meningeal irritation ทั้ง Kernig’s และ Brudzinski’s sign อาจ

มี nuchal rigidity นอกจากนี้ยังอาจพบความผิดปกติของ CN รวมดวย เชน III ,IV, VIII /

การวินิจฉัย gold standard ในผูปวยที่สงสัยภาวะนี้ตองทํา Lumbar puncture เพื่อดู CSF

profile วาเปนmeningitis หรือไม และเกิดจากเชื้อกลุมใด อยางไรก็ตาม มีขอหามในการทํา

LP ถาผูปวยมี Focal neurologic sign,มี Evidence of space-occupying lesion with

increased ICP , Immunocompromised host, , มี new onset of seizure ภายใน 1 สัปดาห

และมี papilledema ผูที่มีขอหามขอใดขอหนึ่ง ควรไดรับการทํา CT brain กอน เนื่องจาก

กลัว brain herniation ถาพบรอยโรคใน brain จรงิ จะไมทํา LP แลว treat ไปเลย

การรักษาคือ การให Empirical antibiotics ทันทีหลังจากการทํา LP และ Hemoculture

แลว นอกจากนี้การให dexamethasone ยังมีประโยชนในผูปวยที่เปน pneumococcal

meningitis

Page 3: Nle step 2_2009 si115-116 and nle_step_2_2009 nctms editors cut key

SIRIRAJ

SIRIRA

J SIRIR

AJ

กลับมาพูดถึงผูปวยรายนี้ มีปญหาคือมี anterior fontanelle โปง เขาใจวานาจะหมายถึง

มี ICP สูง เราไมแนใจวาเทาที่โจทยใหมาเปน C/I ในการทํา LP หรือไม แตคิดวาไมนาจะ

หามทํา LP เพราะเด็กยังไมมีอาการ focal neurologic sign ที่ชัดเจน (มั้ง?) ดังนั้นขอตอบขอ

1 คือขอความยินยอมจากพอแมเพื่อ LP (แตไมแนใจวาตอง CT กอนรึปาวนะ แตหลักการ

ตามนี้ละ)

9.) ตอบ 5. ให amitriptyline

อาการปวดศีรษะแบบตุบๆขางเดียว เปนๆหายๆ คิดถึง Migraine without aura ซึ่ง

หลักการรักษาคือ 1. หลีกเลี่ยงสิ่งกระตุน & 2. Medication โดย

Acute : paracetamol, NSAIDs; Ergotamine tartrate; Triptans ( 5HT agonists)

Chronic : ใหยา : β blocker, CCB, TCA, SSRIs เพื่อปองกันเมื่อ

1. ปวดบอยมากกวา 2 ครั้งตอเดือน ซึ่งทําใหเกิด disability มากกวา 3 วัน

2. ปวดแตละครั้งรุนแรงมาก

3. ปวดมากกวา 2 ครั้งตอสัปดาห

4. ไมสามารถใชยา acute medication ได

5. ใชยา acute medication มากเกินไป

10.) ตอบ 2. levodopa

Parkinson disease is a progressive neurodegenerative disorder associated with a

loss of dopaminergic neurons in the substantia nigra / Cardinal signs : resting tremor,

cogwheel rigidity, bradykinesia, Postural instability

- levodopa: standard of symptomatic treatment for Parkinson disease, fewest

adverse effects in the short term, its chronic use is associated with the development of

fluctuations and dyskinesias.

- Bromocriptine(Dopamine agonists): provide moderate symptomatic benefit and

rarely cause fluctuations and dyskinesias , but more side effects than levodopa,

including sleepiness and impulse control disorders, these side effects resolve upon

lowering the dose or discontinuing the medication

- For patients younger than 65 years, use a dopamine agonist and then add

levodopa/PDI when the dopamine agonist (with or without an MAO-B inhibitor) no

longer provides good control of motor symptoms. Dopamine agonists may provide good

symptom control for several years.

- For patients who are demented or older than 70 years (those who may be prone to

adverse effects, such as hallucinations, from dopamine agonists), and for those likely to

require treatment for only a few years, not to use a dopamine agonist and depend on

levodopa/PDI as primary symptomatic therapy

- benhexal(Anticholinergic) medications provide good tremor relief in

approximately 50% of patients but do not improve bradykinesia or rigidity

- Selegiline (MAO-B inhibitors) provide mild symptomatic benefit, have excellent

side effect profiles, and may improve long-term outcomes, a good choice as initial

treatment for many patients, delays the need for levodopa therapy in early Parkinson

disease

11.) ตอบ 1. Phenytoin

คิดถึงภาวะ generalized tonic-clonic seizures ยาหลักในการรักษาก็คือ Phenytoin

Phenobarbital Carbamazepine สวน diazepam นั้นมักเลือกใชเปนยาชนิดแรกในกรณี

status epilepticus

12.) ตอบ 1. Hypokalemia

แขนขาออนแรง ตองคิดถึง hypokalemia ซึ่งรายนี้มีประวัติได HCTZ ซึ่งทําให

hypokalemia ได

Page 4: Nle step 2_2009 si115-116 and nle_step_2_2009 nctms editors cut key

SIRIRAJ

SIRIRA

J SIRIR

AJ

13.) ตอบ 4. Ceftriaxone

เนื่องจากผูปวยมีภาวะ urosepsis กลาวคือมีอาการไขสูง หนาวสั่น ปสสาวะแสบขัด

ผล U/A พบ WBC 50-60 จึงเขาไดกับภาวะ UTI รวมกับมี sign of SIRS > 2ใน 4 ขอ ไดแก

1. T > 38 or < 36 C 2. RR > 24 3. P > 90 4. WBC > 12000 or < 4000 or band form > 10

% จึงควรใหการรักษาดวย IV quinolone / ampicillin + gentamicin / ceftriaxone

14.) ตอบ 1. urineVMA

Pheochromocytoma

อาการและอาการแสดง 1. Headache 2.Diaphoresis 3.Palpitations 4.Severe

hypertension (These 4 characteristics together are strongly suggestive of a

pheochromocytoma) 5.Tremor 6.Nausea 7.Weakness 8.Anxiety 9.Epigastric pain

10.Flank pain 11.Constipation 12.weight loss

Dx 1.Plasma metanephrine testing has the highest sensitivity (96%) for detecting a

pheochromocytoma, lower specificity (85%).

2.24-hour urinary collection for catecholamines ,vanillylmandelic acid and

metanephrines has a sensitivity of 87.5% and a specificity of 99.7%

3.CT , MRI abdomen

15.) ตอบ 3. Serum electrolyte

ในผูปวยรายนี้จากประวัติและการตรวจรางกายทําใหนึกถึงภาวะ primary

hyperaldosteronism เนื่องจากมี muscle weakness จาก potassium ต่ํา ไมมีอาการใจสั่น

เหงื่อ ออกหนาแดง อันจะพบไดใน pheochromocytoma ดังนั้นการสง Ix ที่เหมาะสมที่สุด

ในผูปวยรายนี้ นาจะเปน serum electrolyte มากที่สุด

16.) ตอบ 4. Prednisolone

ผูปวยมี nephrotic nephritic symdrome คิดถึง glomerular disease : primary or

secondary จากโจทยไมไดบอก clue ที่สงสัยวามีโรคประจําตัวใดๆ / คิดถึง primary กอน

ชวงอายุประมาณ 30-40 yrs อาจพบ IgA nepropathy, postinfectious, lupus ..... / การรักษา

ใหเปน ACEI or ARB , steroid

17.) ตอบ 5. Primary polydipsia

ขอนี้บอกตามตรงวา ขอสอบมันงงๆนะ คือถาโจทยจําตัวเลขมาถูกแลว ก็ตองมาแปล

ผล lab กันนะ คนนี้มี hyponatremia แลว urine Na ต่ํา (ปกติ 15-250 mEq/L/day) และ

urine osmol. ต่ํา (ปกติ 500-800 mOsm/kg) มาตัด choice กัน

SIADH คือ รางกายหลั่ง ADH ซึ่งทําหนาที่ดูดกลับน้ํา มากเกินไป ก็ทําให

hyponatremia ได แต urine ก็จะตอง เขมขนสิ ควรมี urine osmol และ urine Na สูง จึงไม

ตอบขอนี้

Low salt intake กินเกลือนอยไมใชสาเหตุที่ทําให hyponatremia ไดมากขนาดนี้

Renal wasting คือ เสีย Na ไปทางไต ก็ควรมี urine Na สูงสินะ

Thiazide มี side effect ทําให hyponatremia ไดจากการที่ขับ Na ทางปสสาวะเพิ่มขึ้น

ก็นาจะมี urine Na สูงดวย

Primary polydipsia เขาไดมากสุด เพราะทําใหมี hyponatremia , urine Na ต่ํา และ

urine osmol. ต่ํา ; แตก็ไมเขาใจวาเกี่ยวอะไรกับประวัติ CA lung ตอนแรกคิดวาจะให

คิดถึง SIADH ที่พบไดใน Small cell lung cancer แต lab ไมเขาอะ

18.) ตอบ 1. Calcium gluconate

เนื่องจากมีภาวะ hyperkalemia (K =6.0)ทําใหเกิด cardiac arrhythmia ได ; อาการ

ไดแก Generalized fatigue,Weakness ,Paresthesias ,Paralysis ,Palpitations หรืออาจ

asymptomatic / Risk ไดแก Acute or chronic renal failure, Trauma including crush

injuries (rhabdomyolysis) or burns ,Ingestion of foods high in potassium (eg, bananas,

oranges, high-protein diets, tomatoes, salt substitutes) ,ยา - Potassium supplements,

potassium-sparing diuretics, NSAIDs, beta-blockers, digoxin, succinylcholine, and

digitalis glycoside ,Redistribution - Metabolic acidosis (diabetic ketoacidosis [DKA]),

Page 5: Nle step 2_2009 si115-116 and nle_step_2_2009 nctms editors cut key

SIRIRAJ

SIRIRA

J SIRIR

AJ

catabolic state Rx : สงตรวจEKG,หยุดpotassium sparing drug, ให Calcium chloride or

calcium gluconate (Kalcinate)

Adult

- Calcium chloride: 5 mL of 10% sol IV over 2 min (stop infusion if bradycardia )

- Calcium gluconate: 10 mL of 10% sol IV over 2 min (stop infusion if bradycardia )

Pediatric

- Calcium chloride: 0.2 mL/kg/dose of 10% sol IV over 5 min; not to exceed 5 mL (stop

infusion if bradycardia )

- Calcium gluconate: 100 mg/kg (1 mL/kg) of 10% sol IV over 3-5 min; not to exceed 10

mL (stop infusion if bradycardia )

19.) ตอบ 1. Hypo K

ก็ขอนี้ ถาม S-E ของ HCTZนะ ไดแก

Hypo “ขนม” K Na Mg แลวก

Hyper Ca glycemia lipidemia uricemia

สําหรับอาการแขนขาออนแรง เนี่ยกcommon จาก Hypo K แหละ

Hypo Na N/V fatique ซึม ชัก

Hypo Ca Neuromuscular irritability

HypoMg N/V CNS suppression hyporeflexia (คลายๆ preeclampsiaที่ตอง ใหMg

แกเรยเนอะ ^^)

20.) ตอบ 2. Bronchiectasis

เนื่องจากไอเรื้อรังมานาน มีเสมหะเขียว และ peribronchial thickening **

Bronchiectasis เปน chronic infection ทําใหbronchus+bronchiole dilate อยางถาวร (เปน

Obstructive lung) , เกิดจากเชื้อ : H. influenzae, S. pneumo, S. aureus, P. aeruginosa;

อาการ : ไอเรื้อรัง, เสมหะเขียวปริมาณมาก (อาจมีเลือดปน); Ix: CXR, Sputum culture,

HRCT ; CXR : Cystic shadow, bronchial wall thickening (Tramline and ring shadows);

Treatment : postural drainage, ATB, Bronchodilator (ถามี asthma, COPD รวมดวย)

21.) ตอบ 1. Sputum AFB

เนื่องจาก อาการที่เขาไดกับ pulmonary TB คือ ไขตอนเย็นๆ ไอมีเสมหะ 3 wk ตรวจ

film พบ alveolar infiltration และ Cavity ที่ RUL (suggest actibe TB) การตรวจพบ AFB

positive ก็จะชวยยืนยันผลได (แตถาตรวจไมพบก็คง treat as TB อยูดี?) Regimen ที่ควร

ไดรับในการรักษาคือ 2 IRZE + 4 IR [Co-amoxiclav และ levofloxacin จริงๆแลวก็

สามารถใชในการรักษา TB ไดเชนกัน โดยใชรักษาในกลุม multidrug resistance TB ซึ่ง

ตองใหรวมกับยาอื่นๆอีก คือ Clofazimene Moxifloxacin และ Amikacin]

ในผูปวย HIV จะมีโอกาสตรวจพบเชื้อจากเสมหะไดนอย ดังนั้นในผูปวย HIV เพื่อ

การวินิจฉัยอาจตรวจโดย bronchoscopy, tissue biopsy, blood culture

22.) ตอบ 2. วินิจฉัยใหไดวาเปนมะเร็งจริง

แตสําหรับโจทยขอนี้ คิดวา เนื่องจากแคสงสัยวาเปนมะเร็งปอดกอนอื่นก็ตองวินิจฉัย

ใหไดวาเปนมะเร็งปอดจริง จึงคิดวานาจะตอบขอ 2 นะ ขางลางนี้เปนความรูที่ไปหามาให

ไมเกี่ยวกับการตอบขอนี้เลย

โรคมะเร็งปอด : มีสาเหตุจากสารเคมีอนุภาคหลายชนิดไดแก แอสเบสตอส เรดอน

คลอโรเมธินอีเธอร โพลีอะโรมาติก ไฮโดรคารบอน อารเซนิก โครเมียม นิกเกิล อยางไรก็

ตามจากประวัติการสัมผัสทําใหสามารถวินิจฉัยวาเกิดจากการทํางาน ประวัติที่ทําให

วินิจฉัยยากคือ ประวัติการสูบบุหรี่ สวนใหญเมื่อวินิจฉัยกอนจะโตมากแลว และผูปวยจะ

ถึงแกกรรมในเวลาไมนาน อาการคือมีอาการของการระคายเคืองของเยื่อหุมปอด

หลอดลม ถุงลมในปอด ทําใหมีอาการหอบเหนื่อย ไอเปนเลือด นอกจากนี้มะเร็งปอดยัง

สามารถแพรกระจายไปยังอวัยวะอื่นทําใหมีอาการของอวัยวะนั้นๆ การสูบบุหรี่ทําใหเปน

มะเร็งปอดไดและมีโอกาสที่จะเปนจากบุหรี่มากกวาการประกอบอาชีพหลายเทา

Page 6: Nle step 2_2009 si115-116 and nle_step_2_2009 nctms editors cut key

SIRIRAJ

SIRIRA

J SIRIR

AJ

23.) ตอบ 4. Bronchogenic carcinoma

อาการแนนหนาอก เหนื่อย หนาและแขนบวม มี face and upper extremity edema ,

superficial vein dilatation at chest wall เขาไดกับ SVC (ในระยะแรกอาจจะ ไมแสดง

อาการใด ๆ โดยเฉพาะเปน partial obstruction แตโดยสวนใหญจะมีอาการ และอาการ

แสดงเล็ก ๆ นอย ๆ ใหเห็นบาง แตหาก total obstruction จะแสดงอาการตางๆโดย

Dyspnea เปนอาการที่พบมากที่สุด อาการอื่นๆ ประกอบดวย facial swelling, head

fullness, cough, arm swelling, chest pain, dysphagia, orthopnea, distorted vision,

hoarseness, stridor, headache, nasal stuffiness, nausea, pleural effusions, and light-

headedness).

สาเหตุของ SVC obstruction

มากกวา 80% เกิดจาก malignant mediastinal tumors : Bronchogenic carcinomas

พบ 75-80% ในภาวะนี้ โดยสวนใหญเปน small-cell carcinomas ; Non-Hodgkin

lymphoma (โดยเฉพาะ large cell type) พบประมาณ 10-15% ; Malignant อื่นๆที่พบนอย

ประกอบดวย Hodgkin disease, metastatic cancers, primary leiomyosarcomas of the

mediastinal vessels, and plasmocytomas.

Nonmalignant อื่นๆที่เปนสาเหตุของ SVC obstruction ประกอบดวย mediastinal

fibrosis; vascular diseases เชน aortic aneurysm, vasculitis, and arteriovenous fistulas ;

infections such as histoplasmosis, tuberculosis, syphilis, and actinomycosis ; benign

mediastinal tumors เชน teratoma, cystic hygroma, thymoma, and dermoid cyst ; cardiac

causes, เชน pericarditis and atrial myxoma; and thrombosis related to the presence of

central vein catheters.

24.) ตอบ 4. ใหยาเดิม + ใส ICD

ขอนี้คิดวาเปนaspiration pneumonia เชื้อที่พบนาจะเปนพวก anaerobe จาก GI tract

การใหยา co-amoxyclav ก็สามารถคลุมเชื้อได แตควรใสICDเพื่อระบาย pleural effusion

ดวยเพราะเปน complicated pleural effusion (เปน indicationใหใสICD)

Complicated pleural effusion มีลักษณะดังนี้

1.Frank pus or empyema thoracis

2.มีorganisim (พบในผูปวยรายนี้)

3.pH<7.2

4.Sugar<60mg/dl (พบในผูปวยรายนี้)

5.Loculate parapneumonic effusion

Transudate Exudate Empyema

thoracis

pH 7.4-7.6 <7.4 <7.2

Cell <1,000

L,Mo

1,000-5,000

PMN

>5,000 PMN

Specific gravity <0.015 >0.015

LDH <200 200-1,000 >1,000

LDH ratio <0.6 >0.6

Protein ratio <0.5 >0.5

Serum pleural-albumin

gradient (serum albumin-

pleural albumin)

>1.2 <1.2

25.) ตอบ 3. Organophosphate

Organophosphate กระตุน Parasym นะ อาการกคือ SLUDGE 3B

S Sweating + Salivation

L Lacrimation

U Urination

D Diarrhea

G GI ปวดทอง ( จาก motility เยอะมั้ง)

Page 7: Nle step 2_2009 si115-116 and nle_step_2_2009 nctms editors cut key

SIRIRAJ

SIRIRA

J SIRIR

AJ

E Emesis

B Bradycardia

B Bronchorhea ( secretion เยอะ)

B Bronchospasm

+ miosis + Nicotinic effect เชน M.fasiculation Resp M paralysis +

CNS effect เชน seizure Coma CNS depression

26.) ตอบ 1. Botulinum toxin

กตอบขอนี้แหละมีอยูขอเดียว อาการเปนไดตั้งแต หายไดเอง จนถึง เสียชีวิต อาการคือ

symmetrical descending paralysis ,abd cramp ,N/V ,diarrhea ,Resp failure การรักษาคือ

admit observe cli , +-gastric lavage

27.) คือ ขอนี้ มันเปนลักษณะ typical ของ Arsenic poisoning เรยแหละ แลว antidote กคือ

Dimercaprol (BAL in Oil) หรือ Succimer (DMSA) หรือ Dimerval (DMPS) ไมเห็นมีตํา

ตอบเรยอะ งง

29.) ตอบ 1. Acute thyroiditis

ผูปวยรายนี้เขาไดกับ Thyrotoxicosis (ภาวะที่รางกายมี thyroid h. มากกวาปกติ) มี

อาการเหนื่อยงาย, ใจสั่น, sweating, tremor, diarrhea, กินเยอะแตน้ําหนักลด; PE : fine

tremor (hand), lid lag & lid retraction, onycholysis

พิจารณาลักษณะของกอน เปนได 3 แบบ คือ 1. Diffuse goiter ; 2. Solitary nodule ; 3.

Multinodular goiter รายนี้เขาไดกับ diffused goiter โดย

1. Grave’s dis. : thyrotoxicosis, exopthalmos, pretibial myxedema, คลําได thrill

2. Thyroiditis : แบงเปน acute, subacute, chronic (depend on duration and course of

disease) คิดวานาจะตอบ acute thyroiditis เพราะมี pain, neck swelling, fever (สวน

suppurative thyroiditis นาจะ Unilateral neck pain และจะบวมแดงกดเจ็บบริเวณนั้นดวย)

3. อื่นๆ เชน thyrotoxicosis factitious

กรณีที่ประวัติแยกไดไมชัดเจนจะสงตรวจ I131 ดูการ uptake เพื่อแยกโรคตอไป

31.) ตอบ 2. มีโอกาสเกิด embolic stroke มากกวาคนทั่วไป 15-20%

Diactolic rumbling murmur นึกถึง MS ซึ่งมักเกิดจาก Rheumatic heart disease

หลังจาก Rheumatic fever / MS ทําใหมีความเสี่ยงตอ embolic stroke มากขึ้น/ ขอ3ผิด

เพราะ MS ทําใหเกิด pulmonary hypertension ไดจริง แตเนื่องจากลิ้นหัวใจตีบ ไมใชรั่ว

32.) ตอบ 3. Bacterial endocarditis

ขอนี้ผูปวยมาดวย ไข + HF จึงสงสัย bacterial endocarditis โดยวินิจฉัยตาม Duke

criteria (ลองไปหาดูเองนะ) ซึ่งในรายนี้ มี valvular regurgitation, conjuctival hemorrhage,

fever, Osler’s node(ตุมดําแดงกดเจ็บที่ปลายนิ้ว) ครบ 1 major + 3 minor criteria

atrial myxoma อาจจะไมมีอาการเลย หรือ มาดวยอาการจาก mass effect หรือ จากการ

ที่ tumor ไป involve muscle ปกติ ทําใหเกิด heart failure

Acute rheumatoid carditis (= acute rheumatic fever) มีอาการไดดังตอไปนี้ carditis,

polyarthritis, chorea, erythema marginatum, and subcutaneous nodules เปนตน

Rupture chordae tendinae ก็จะมีอาการของ mitral valve regurgitation, acute left-

sided CHF, pulmonary edema, tachypnoea and coughing เปนตน

33.) ตอบ 2. Epinephrine

Pulseless Arrest Algorithm : ผูปวยไมมี pulse แตมี electrical activity จึงจัดอยูใน

PEA ขั้นตอไปจึง ให epinephrine 1 mg. IV/IO repeat every 3-5 min

35.) ตอบ 3. Eisenmenger Syndrome

ซึ่งเปนภาวะที่พบไดในผูปวยที่มี large congenital cardiac หรือ surgically created

extracardiac left-to-right shunts เปนผลใหเกิดincreased pulmonary blood flowตามมา.

ตรสจรางกายสามารถพลลักษณะตางๆไดดังนี้ Central cyanosis (differential cyanosis in

the case of a PDA) , Clubbing , Jugular venous pulse (wave may be A-wave dominant,

and, in the presence of a significant tricuspid regurgitation, the V wave may be

prominent; central venous pressure may be elevated) , Precordial palpation reveals a

right ventricular heave and, frequently, a palpable S2. , Loud P2 , High-pitched early

Page 8: Nle step 2_2009 si115-116 and nle_step_2_2009 nctms editors cut key

SIRIRAJ

SIRIRA

J SIRIR

AJ

diastolic murmur of pulmonic insufficiency , Right-sided fourth heart sound , Pulmonary

ejection click , Single S2

36.) EKG

Sinus arrest

Sinus bradycardia

Complete heart block

LBBB

Premature atrial beat

37.) ตอบ 5. C.difficile

เนื่องจากมีประวัติเขาไดกับ antibiotic associated diarrhea และมีลักษณะเปน mucous

diarrhea

38.) ตอบ 5. U/S whole abdomen

คิดวาเปน ruptured AAA with decompensated shock Ix ทํา U/S เพื่อ diagnosis and

F/U CT if plan surgery แตถา shock อยูหามทํา ดังนั้น จึงสง U/S whole abdomen

39.) ตอบ 5. NSS

ผูปวย shock (tachycardia, BP drop) แบบ hypovolemic (flat neck v.) ตอง resuscitate

ดวย isotonic solution คือ NSS

40.) ตอบ 4. Stool occult blood (จากชีทอ.กมล recommendation for Thai adults)

- grade ก.(ควรทํา) แนะนําตรวจ stool occult blood/colonoscopy ทุก 5 ปแนะนําใน

ทุกอายุ ถามีประวัติ family history of colorectal CA, colonic polyp

- grade ข. (นาทํา) แนะนําตรวจ stool occult blood ทุก 5 ป ที่อายุ> 40 ป จากที่คนนี้

ไมมีประวัติครอบครัวจึงนาจะตรวจแค stool occult blood

41.) ตอบ 5. ตรวจ HBs Ag ซ้ําอีก 6 เดือน

ขอนี้มันมีในขอสอบชุดที่ 3 สถาบันจํามา แตไมตรงกันงะ โจทยวา หญิงจะมาสมัคร

พยาบาล ตรวจเลือดพบ HBsAg + และ HBsAb – (นาจะหมายถึง anti-HBs -ve) สวนชุด

ของคณะเราเปน anti-HBc IgM –ve

การแปลผล คือ HBsAg + แสดงวา มีการติดเชื้อ HBV

anti-HBc IgM –ve แสดงวา ไมใช recent infection

anti- HBs -ve แสดงวา ไมมีภูมิ มาตัด choice กัน

ขอ 1 ให HBV vaccine ผิดแหงๆ เพราะติดเชื้อไปแลว จะมาให vaccine อะไรกัน

ขอ 2 ให IFN ผิด เพราะจากโจทยบอกวา LFT normal ซึ่งถา ALT ปกติ หรือ

เพิ่มขึ้นนอยกวา 2 เทาของคาปกติ ก็ให follow up ยังไมตองรักษา การ follow up ยังไงก็

ขึ้นกับวาHBeAg +veหรือ–ve [จะให IFN เมื่อ HBV DNA>20000 หรือ ALT > 2X ULN]

ขอ 3 และ 4 ผิดคูกัน เพราะการตรวจ AFP ,U/S liver เปนการตรวจ HCC

surveillance ใน high risk group ซึ่งไดแก ชาย >40 ป , หญิง >50 ป, มี advanced fibrosis/

cirrhosis, มี family Hx of HCC ซึ่งจะตรวจทุก 6 เดือน

Page 9: Nle step 2_2009 si115-116 and nle_step_2_2009 nctms editors cut key

SIRIRAJ

SIRIRA

J SIRIR

AJ

จึงตอบ ขอ 5 ตรวจ HBs Ag ซ้ําอีก 6 เดือน ถายังมี HBsAg +ve อยู แสดงวาเปน

HBV carrier เพราะวาสวนใหญรางกายจะสามารถกําจัดเชื้อไปไดเอง

42.) ตอบ 4. delirium tremens

เนื่องจากผูปวยมีประวัติดื่มเหลามานาน และหลังเขารักษาในรพ. แพทยไดNPOทันที

จึงเกิดอาการอาละวาดดึงสายน้ําเกลือ

อาการDT ไดแก confusion, diarrhea, disorientation and agitation, perceptual

disturbance(visual), autonomic hyperactivity (high pulse, blood pressure, and rate of

breathing)

สาเหตุ: ผูปวยมักมีประวัติดื่มalcoholมานาน และมีการหยุดดื่มทันที ทําใหเกิดอาการ

withdrawal Delirium tremens มักเกิด3-7 วัน หลังการดื่มครั้งสุดทาย หากไมไดรับการ

รักษาอาจทําใหเสียชีวิตได(mortality rate35%) สําหรับผูปวยที่มีความเสี่ยงสูงไดแก

extreme fever, fluid and electrolyte imbalance, or intercurrent illness such as occult

trauma, pneumonia, hepatitis, pancreatitis,alcoholic ketoacidosis,Wernicke-korsakoff

syndrome

การรักษา : ใหยาในกลุมbenzodiazepines

43.) ตอบ 1. bulk forming agent

ยาระบาย (laxatives) มีดังนี้

Fiber supplements or bulk laxatives eg.Metamucil เปน laxatives ที่ปลอดภัยที่สุด

กลไกคือ fiber จะดูดน้ําทําใหอุจาระขยายขนาดใหญขึ้นไปกระตุน reflex ใน colon ดังนั้น

จึงตองกินน้ําตามเยอะๆ ถากินน้ําไมพอก็จะอุดตันลําไสได ขอเสียคือใชเวลาหลายวันกวา

จะไดผล และพวก fiber จะลดการดูดซึมของยาและวิตามิน

Stimulants eg. Bisacodil (Dulcolax), Senokot กระตุน n.plexus ในลําไสทําให colon

m. หดตัว(กระตุน reflex โดยตรง) แตถาใชนานๆ n.plexus จะถูกทําลายทําใหตอมาทองผูก

ไปเลย จึงไมควรสั่งใหคนไขที่ทองผูกเปนประจําใช

Lubricant eg. Mineral oil หลอลื่น แตใขอเสีย คือ ลดการดูดซึมวิตามินที่ละลายใน

ไขมัน เพราะมันจะไหลออกมากับอุจจาระเลย หามใชรวมกับ surfactant laxatives

Note stimulants, lubricant มีการใชเพื่อลดนน. ถือเปน laxatives abuse และมีผล

รบกวน normal bowel habit

Surfactant laxatives eg. castor oil ทําให stool นุมออกงายขึ้น castor oil ใชเตรียม

bowel เพราะออกฤทธิ์เร็ว (1-3 hr) แตมีขอเสียคือ electrolyte imbalance ได

Saline and Osmotics laxatives eg. MOM ; milk of Magnesia, Unison suppository

(Na) ออกฤทธิ์เร็วใน 3 hr ทําให osmolarity in colon สูงขึ้นจึงดึงน้ําออกมาคลุกเคลากับ

อุจจาระ ซึ่งมันก็จะดึงน้ําออกมาพรอมกับ electrolyte ดวย ดังนั้นจึงมีขอเสีย คือ electrolyte

imbalance และอาจมีการดูดซึม Mg เขากระแสเลือดได เกิด SE ของ Mg, ไมควรใชใน

ผูปวยโรคไต

44.) ตอบ 1. ENT exam

neck mass ที่ นานกวา 2 wk ควร refer เพื่อตรวจ ENT exam ทุกราย

45.) ตอบ 2. Plasmodium vivax

P. falciparum P. vivax P. malariae P. ovale

Form ที่พบได

ใน peripheral

blood smear

- Ring form

(Trophozoites)

- Gametocyte

พบไดทุกระยะ พบไดทุกระยะ พบไดทุกระยะ

ลักษณะของ

RBC

- normal

size&shape

- Maurer’s dots

- ขนาดใหญ สี

จางลง

- Schuffner’s

dots

- ขนาดปกติ

หรือเล็กกวา

ปกติ

- ziemann’s

dots

- ขนาดปกติ

หรือใหญ สีจาง

ลง

- tear drop RBC

- Schuffner’s

dots

จํานวน 20 (16-32) 16 (12-24) 8 (6-12) 8 (6-12)

Page 10: Nle step 2_2009 si115-116 and nle_step_2_2009 nctms editors cut key

SIRIRAJ

SIRIRA

J SIRIR

AJ

merozoites

ใน RBC

รูปรางของ

Gametocyte

Crescent form Spherical

shape

Spherical

shape

(มี growing

trophozoites

เปน band

form)

Spherical shape

Complication cerebral malaria

Treatment

1. P. falciparum

- with complication :

- Quinine dihydrochloride 20 mg/kg IV drip in 4 hr (loading dose) then

10mg/kg IV drip in 2 hr q 8 hr for 7 days

- Artesunate 2.4 mg/kg IV then 1.2 mg/kg q 12 hr จนครบ 600 mg

- no complication :

- Quinine sulfate 600 mg oral q 8 hr + Tetracycline 250 mg oral q 6 hr for 7

days (ในเด็กให quinine sulfate 10 mg/kg oral q 8 hr ใน 4 วันแรก then 15-20 mg/kg

q 8 hr for 4 days)

- Artesunate 100 mg oral then 50 mg oral q 12 hr จนครบ 600 mg then

Mefloquine 750 mg oral then 500 mg oral หลัง dose แรก 6 hr

2. P. vivax & P. ovale

- Chloroquine + Primaquine oral for 14 days (หากเปน G-6-PD จะลด dose

primaquine)

3. P. malariae - Chloroquine 600 mg (10 mg/kg ในเด็ก) oral

46.) ตอบ 4. artisunate + mefloquine

47.) ตอบ 1. Acyclovir

ขอนี้หลัก ๆ คือใหดู CSF profile ซึ่งพบวา glucose และ protein ไมตางจากคา

ปกติมากนัก และ wbc ก็มี Lymphocytosis ถึง 95% (มันไมมีคา wbc มาให แตคิดวาคงไม

สูงมาก) G/S ก็ไมพบ จึง R/O bacteria ออกไปไดเลย สวน fungus , TB มี lymphocytosis

ได แตควรมีการเปลี่ยนแปลงของระดับ glucose และ protein ที่ชัดเจนกวานี้ จึงคิดถึงนอย

(Indian ink ก็ไมเจอ crypto , choice ก็ไมมี TB) นาจะเปน viral มากที่สุดนะจะ

48.) ตอบ ไมมีรูปอะ เลยไมรูจะเฉลยอะไรแตจะสรุปใหแลวกันนะ

ลักษณะ P.falciparum P.vivax P.malariae P.ovale

infected rbc ปกติ ใหญ ติดสีจาง ปกติหรือเล็ก ใหญ,รี.

fimbriation

ระยะของเชื้อที่

ตรวจพบ

มักพบเฉพาะ

ring form

พบทุกระยะ พบทุกระยะ พบทุกระยะ

Ring form 1/6-1/5 of rbc

วงบางมาก

- 2 chromatins

- multiple

infection

- marginal

form

1/3 of rbc

วงบาง

1/3 of rbc

วงหนา

1/3 of rbc

วงคอนขางหนา

Growing

trophozoite

เหมือน ring

form

คลายอะมีบา

Pseudopod

มาก

Vacuole ชัด

Band form

compact form

Cytoplasm เปน

วงหนา,

pseudopod

นอย. Vacuole

Page 11: Nle step 2_2009 si115-116 and nle_step_2_2009 nctms editors cut key

SIRIRAJ

SIRIRA

J SIRIR

AJ

ไมชัด

Schizont 8-32(24) 12-24(16) 6-12(8),

rosette

4-12(8)

Gametocyte Crescent กลม กลม กลม

49.) ตอบ 1. branching septate hyphae

Tinea cruris (ขาหนีบ) การติดเชื้อราในกลุม dermatophytes หรอื กลาก ลักษณะ

lesion เปน erythematous active border with central clearing, scaly (ขุยๆ) pruritic (คันๆ)

Ix คือ KOH พบ branching septate hyphae with arthrospore Rx คือ clotrimazole ทา bid

2-3 wk (Tinea ใชยาทาก็พอ ยกเวน Tinea capitis (หนังหัว) และ Tinea unguim (เล็บ) ตอง

ใหยากินดวย เชน griseofulvin, itraconazole)

Pseudohyphae with budding yeast = Candida

Short septate hyphae with oval yeast (spaghetti and meatball) = Malassezia furfur

(โรคเกลื้อน Pityriasis versicolor)

50.) ตอบ 1. CXR

จากโจทย นึกถึง Erythema nodosum (EN) มากที่สุดเนื่องจากอาการแสดงของผูปวย

เขาไดdefinitionของโรคนี้เลย คือ

Erythema nodosum (EN) is an acute, nodular, erythematous eruption that usually

is limited to the extensor aspects of the lower legs.

โดยมีclinical presentation คือ The eruptive phase of erythema nodosum begins

with flulike symptoms of fever and generalized aching ก็คือมี Febrile illness with

dermatologic findings includes abrupt onset of illness with initial fever, followed by

a painful rash within 1-2 days. ก็เหมือนในโจทยนั่นเอง

โดยโรคนี้เปน delayed hypersensitivity reaction and may occur in association with

several systemic diseases or drug therapies, or it may be idiopathic. โดยมีสาเหตุไดหลาย

สาเหตุ แต the most common cause of erythema nodosum คือ Bacterial infections

โดยเฉพาะ Streptococcal infections พบทั้งในเด็กและผูใหญ ในผุใหญที่พบบอยอีกอันคือ

sarcoidosis เพราะThe most common cutaneous manifestation of sarcoidosis is erythema

nodosumนั่นเอง นอกจากนี้ infectionsในผูใหญ(โดยเฉพาะใน developing countries อยาง

ประเทศเรา) ตองคิดถึง Tuberculosisดวย(จริงๆยังมีอีกหลายเชื้อเลย สามารถคนดูใน

หนังสือ หรือ emedicineได)

ดังนั้น investigationในรายนี้ คือการหาการติดเชื้อนั่นเอง ซึ่งอาจทําไดหลายอยาง เชน

throat culture,stool examination, blood cultures ตามเชื้อที่สงสัย ซึ่งในผูใหญเราสงสัย

Tuberculosis กับ sarcoidosis ดังนั้นตองสง film chest x-ray ดวย จึงตอบขอ 1

51.) ตอบ 4. Praziquantel (จากภาพเปนไขของ Opisthorchis viverrini)

52.) ตอบ 2. Euglobulin test

Bleeding time I/C : ผูปวยที่มีเลือดออกผิดปกติ ที่นาจะเกิดจาก primary

hemostatic defect แตมีจํานวนเกร็ดเลือดปกติ

patient preparation :

- แขนของคนไขขางที่จะตรวจ ตองไมไดอยูระหวางการได IV fluid ใดๆ ทั้งสิ้น

- คนไขที่รับประทานยา Aspirin ตองหยุดยา 7 วันขึ้นไป กอนรับการตรวจ

- ไมตรวจในผูปวยที่มี Hct ต่ํากวา 30 %

วิธีการวิเคราะห (methodology) : Modified Ivy method โดยควบคุมความดันเลือด

ของแขน ผูปวยดวย เครื่องวัดความดันโลหิต ที่ 40 มิลลิเมตรปรอท และใชมีดปลายแหลม

เจาะผิวหนังใหเปนแผลลึก 3 มิลลิเมตร กวาง 2 มิลลิเมตร จับเวลาตั้งแตเลือดเริ่มไหลจน

เลือดหยุดไหล โดยใช กระดาษกรองซับเลือด

----> ผูปวย Dengue มี thrombocytopenia อยูแลว

Euglobulin lysis time I/C : ผูปวยที่สงสัยภาวะ increased fibrinolysis

----> Hemostasis depends on the balance between coagulation and fibrinolysis

Laboratory tests for assessment of the coagulation system

Page 12: Nle step 2_2009 si115-116 and nle_step_2_2009 nctms editors cut key

SIRIRAJ

SIRIRA

J SIRIR

AJ

- Coagulation Screening tests , Activated partial thromboplastin Time ,

Prothrombin time , Thrombin time , Fibrinogen , Fibrinolysis , Screening tests ,

Euglobulin clot-lysis time , Dilute-whole-blood clot-lysis time

ผลแทรกซอนของโรค Dengue

1. Prolonged shock

2. Fluid overload

3. Massive bleeding

4. Encephalopathy จาก hepatic encephalopathy

53.) ตอบ 1. Melioidosis

เนื่องจาก fever 2wk, FBS 260 (นาจะมี U/D เปน DM), BUN/Cr สูง (อาจมี renal

impairment อยูเดิม), ตับโต มามโต ; u/s พบ hypoechoic ก็นาจะบงถึง abscess ซึ่งมี

multiple ที่มาม ; จากประวัติทั้งหมดนี้เปน typical natural Hx of “ Melioidosis ”

Hx : ผูปวยมี U/D เชน DM, renal fail, thalassemia อาชีพที่สัมผัสดิน เชนทํานา ตรวจ

พบบาดแผลที่ขา/เทา มาพบแพทยโดยไขสูง

Ix : พบ abscess ตาม organ ตางๆ ที่พบบอย : liver, spleen, skin

Rx :1. Acute : Ceftazidime iv or Imipenem or Meropenem นาน 2 wks

2. ตอมา : Co-trimoxazole + Doxycline (ในเด็กให Augmentin แทน) นานรวม

ระยะ acute = 20 wks

54.) ตอบ 3. Strongyloides stercolaris

ผูปวยที่มีการติดเชื้อ stongyloid อยูกอน เมื่อไดยา steroids ซึ่งกดภูมิคุมกันทําใหเกิด

disseminated strongyloidiasis คือแพรกระจายไปหลาย system ผูปวยอาจมาดวย

abdominal pain, distension, shock, pulmonary and neurologic complications, septicemia

ดังนั้นจึงควรตรวจอุจจาระหา larva ของมันกอนเริ่มยาเสมอ

55.) ตอบ 1. N. fowleri

เชื้อ Naegleria fowleri เขาสูรายกายทางโพรงจมูก มักจะเกิดเมื่อมีอากรสําลักน้ํา และ

เชื้อจะขึ้นสมองโดยผานทางเสนประสาทรับกลิ่น (olfactory nerve) ทําใหเกิดเยื้อหุมสมอง

อักเสบ (Primary amoebic meningo-encephalitis หรือ PAM) N. fowleri มีรูปรางลักษณะ 2

ระยะ ไดแกระยะ trophozoite ซึ่งอาจมีรูปรางแบบอะมีบา (ameboid form) กับแบบมี

หนวดสําหรับวายน้ํา (flagellate form) และระยะ cyst เชื้อเจริญเติบโตไดดีที่อุณหภูมิสูง

ระยะฟกตัวของโรค 3-7 วัน ผูปวยจะมีอาการปวดศีรษะมาก เจ็บคอ คลื่นไส อาเจียน

เพอ ไขสูง คอแข็ง งวงซึม อาการจะทรุดอยางรวดเร็ว และมักจะเสียชีวิตภายใน 10 วัน

โรคนี้สวนใหญพบในคนวัยหนุมสาวทีมีสุขภาพ

56.) ตอบ 2. Chikungunya

ติดตอโดยยุงลาย Aedes aegypti อาการไดแก ไขสูงเฉียบพลัน ผื่นแดงตามรางกาย อาจ

คันรวมดวย conjunctival injection,migratory polyarthritis โดยมักเปนที่ขอเล็กๆ เชน

ขอมือ ขอเทา อาการหายภายใน 1-2 wk ไมรุนแรงถึงshock อาจพบpetechiae หรือ

tourniquet +ได ขอแตกตางจาก Dengue คือ ไขสูงเฉียบพลันกวา และระยะไขเพียง 2 วัน,

จํานวนpetechiaeใน tourniquet testนอยกวา,ไมพบconvalescent rash,พบMP

rash,conjunctival infection,myalgia,arthalgiaไดบอยกวา,ชักจากไข15%ซึ่งมากกวา

dengue3เทา ระบาดชวงฤดูฝน เปนไดทุกกลุมอายุ รักษาโดย supportive treatment

Rickettsial infection แบงเปน3กลุมไดแก spotted fever,typhus,scrub typhus อาการ

เดนคือไขสูงหลายวัน อาการอื่นๆ เชน ปวดหัว ปวดกลามเนื้อ ออกผื่น คลื่นไส อาจมี

eschar diag โดย serology รักษาโดย doxycycline

Rheumatic heart diseaseลิ้นหัวใจถูกทําลายจากกระบวนการเกิดโรคซึ่งเริ่มตนจาก

การเจ็บคอจากการติดเชื้อ streptococcus เปนการดําเนินโรคหนึ่งของ RF acuteมักมาดวย

carditis(ไมคอยมีผลตอ hemodynamic) chronicมักมาดวยmitral stenosis

Page 13: Nle step 2_2009 si115-116 and nle_step_2_2009 nctms editors cut key

SIRIRAJ

SIRIRA

J SIRIR

AJ

57.) ตอบ 2. xertic eczema

อาจเรียก asteatotic eczema หรือคนทั่วไปเรียก winter itch พบมากในผูสูงอายุ

โดยเฉพาะชวงอากาศหนาวหรือแหงกวาปกติ สามารถพบบริเวณหนาแขงไดบอย

เนื่องจากเปนบริเวณที่มี subcutaneous tissue นอยกวาสวนอื่นๆ ของรางกาย

58.) ตอบ ?

Zinc deficiency: Abnormal Growth, Hypogonadism, Skin Problems, Weight Loss,

Diarrhea, Emotional Instability, Hair Loss

Retinol deficiency: Bitot spots, nyctalopia, Dry skin, Dry hair, Pruritus, Broken

fingernails, Keratomalacia, Xerophthalmia, Follicular hyperkeratosis

Selenium deficiency:keshan disease, Kashin-Beck disease,(myocardial necrosis,

atrophy, degeneration and necrosis of cartilage tissue), hypothyroidism

59.) ตอบ 1. thiamine

Wet beriberi (cardiac beriberi) โดยพบในผูที่ไดรับวิตามิน บี1 นอยกวา 0.2

มก./1,000 กิโลแคลอรี่ มีอาการสําคัญคือบวม ระยะแรกจะรูสึกขาหนัก เดินลําบาก บวม

เล็กนอยที่ขาและหนา เนื่องจากมี lactic acid และ pyruvic acid คั่ง เพราะ oxidative

decarboxylation นอยลง ทําใหเซลลอักเสบและมีสารน้ํารั่วออกมา และหัวใจขาดพลังงาน

เพราะใชสารทั้งสองไดลดลง

60.) ตอบ 4. PEG (Percutaneous endoscopic gastrostomy)

เรื่อง nutrition ตองประเมินกอนวา GI function ไดหรือไม ถาได ก็ดูวากินเองไดมาก

นอยแคไหน

- กินไดเองมากกวา 75% ใหกินเอง

- กินได 50-75% ใหกินเองและ oral supplement

- กินไดเองนอยกวา 50% ให enteral feeding

ซึ่ง enteral feeding ตองพิจารณาวามีความเสี่ยงตอ aspiration ไหม

‐ ถาเสี่ยง ให ND/ NJ tube

‐ ถาไมเสี่ยง ให NG หรือ PEG โดยถาตองใหนานเกิน 1 เดือน พิจารณาใหทาง

PEG

เนื่องจากรายนี้ ทางเดินอาหารใชได แตกินอาหารเองไดนอย มีความเสี่ยงต่ําในการ

aspirate ดังนั้นสามารถใหผาน NG / PEGได แตเนื่องจากรายนี้นาจะตองใหอาหารทางนี้

เกิน 1 เดือนดังนั้น นาจะใหทาง PEG

61.) ตอบ 5. ควบคุมอาหาร + ลดน้ําหนัก

เนื่องจาก primary hypertension ควรใหการรักษาดวย life-style modifications (weight

loss, smoking cessation, salt reduction) กอนหากยังไมสามารถควบคุมระดับความดัน

โลหิตไดตามเปาหมายจึงจะพิจารณาใหยา anti-hypertensive drugs ตอไป

Pediatrics

1.) ตอบ 2. พาไปพบจิตแพทย

คิดถึง ADHD (Attention deficit hyperactive disorder) มี 3 กลุมอาการหลัก กอน

อายุ 7 ป ( แตเด็กคนนี้ 8 ป อานะ) คือ 1. ซน (Hyperactive) , 2. ขาดสมาธิ (Inattentivit) ,

3. หุนหันพลันแลน (Impulsivity)

Rx. Medication (stimulant;Methylphenidate) + Behavior modification

Impact . การเรียน สังคม การปรับตัว self esteem

2.) ตอบ 3. stranger anxiety

เพราะเด็กอายุ 9 mo นาจะมี stranger anxiety เพราะฉะนั้นมักจะกลัวคนแปลก

หนา ไมไดยอมใหทุกคนอุม จึงคิดวาขอนี้เปนพัฒนาการที่ผิดปกติสําหรับเด็ก 9 mo

เด็กอายุ 9 mo : Gross&fine motor -> pull to stand,peek- a- boo (จะเอ), mature

pincer grasp, นั่งไดมั่นคง, คลาน / Language-> immature jagoning / Social-> จะเอ,

object permanent / Stranger anxiety เริ่มตอนอายุ 6-9 mo ถึง 2 yr เดนตอนอายุ 9 mo

3.) ตอบ 4. ลดนมเหลือวันละ 2 มื้อ

เด็ก 2-6 ขวบ คิดน้ําหนักโดย 2x+8 คนนี้ 2ขวบ ได 12 Kg น้ําหนักเกินนะ เพราะหนัก

Page 14: Nle step 2_2009 si115-116 and nle_step_2_2009 nctms editors cut key

SIRIRAJ

SIRIRA

J SIRIR

AJ

16 Kg สวนสูง ใช 6x+77 กอเกือบพอดีเลย -> คดิวาขอนี้นาจะตอบไปในทางลด จะลดมือ

อาหารก็ไมได เพราะเด็กจะไดรับสารอาหารไมครบ แตถาลด นมก็พอได

4.) ตอบ 3. รับ OPV, DTP

คําอธิบาย Expanded program of immunization (EPI)

5.) ตอบ 1. TORCH titer

เพราะ เด็กมี increase muscle tone , hyperreflexia , delay development (6เดือน ยังชัน

คอไมได) head circumference มากผิดปกติ ทําใหนึกถึง involve CNS จึงคิดถึง TORCH

infection

6.) ตอบ 2.ออกใบรับรองเขาเรียนไดโดยมีอาจารยพิเศษ

IQ 50-69 (60) เปน mild MR สามารถเรียนรูได (educable) สามารถเรียนจนจบชั้น

ประถมปลายได สามารถฝกทักษะดานสังคมและอาชีพ พอที่จะเลี้ยงตัวเองได เปนแรงงาน

ที่ไมตองใชทักษะฝมือ หรือกึ่งใชฝมือ แตอาจตองการคําแนะนํา และการชวยเหลือบางเมื่อ

ประสบความเครียด

IQ 35-49 moderate MR สามรถเรียนจบ ป.1-ป.2 สามารถฝกอบรมได (trainable) ใน

ทักษะการชวยเหลือ ดูแลตนเอง เรียนรูที่จะเดินทางไดดวยตนเองในสถานที่ที่คุนเคย และ

ฝกอาชีพไดบาง สามารถทํางานที่ไมตองใชทักษะฝมือ แตควรอยูภายใตการกํากับดูแล

อยางใกลชิด

IQ20-34พอจะฝกฝนทักษะการดูแล ตนเองเบื้องตนไดบางแตนอย ดํารงชีวิตอยูใน

สังคมภายใตการควบคุมดูแลอยางเต็มที่ การทํางานตองการโปรแกรมในชุมชน หรือการ

ใหความชวยเหลือที่พิเศษเปนการเฉพาะ

IQ <20 พัฒนาการลาชาอยางชัดเจนในทุกๆดาน มักมีพัฒนาการดานการเคลื่อนไหว

และฝกการชวยเหลือตนเองไดบาง มีขีดจํากัดในการเขาใจและการใชภาษาอยางมาก

ตองการความชวยเหลือ ดูแลอยางใกลชิดตลอดเวลา

7.) ตอบ 3. Amblyopia

จากโจทยผูปวยมีภาวะ misalignment ของ rt eye (internal strabismus) complication ที่

อาจเกิดขึ้นไดคือ เนื่องจากวาเด็กนั้นนารัก มีการปรับตัวอยางดีเยี่ยม พอตา 2 ขางมองได

ไมเหมือนกัน จากอะไรก็ตาม... สมองก็เลยมีการปรับใหตาที่มีปญหานั้นลดการมองเห็นลง

ไป เลยกลายเปนตาขี้เกียจ amblyopia นั่นเอง

Key อยูที่วา เราตอง detect กอนอายุ 7-8 ป แลวรีบรักษาซะ เพราะถาเปนแรวจะ รักษา

ไมได

Amblyopia หมายถึงภาวะที่ตามี visual acuity ลดลงโดยไมมี organic lesion

ถาตาขางหนึ่งมี VA ที่แกไขแลว (ดวย pinhole หรือแวน) นอยกวาตาอีกขางหนึ่ง เทากับ

2 แถวของ Snellen Chart ขึ้นไป

8.) ตอบ 1. Viral croup

Viral croup (laryngotracheobronchitis) common ในเด็กอายุ 6 เดือน – 4 ป

Birth 1mo 2mo 4mo 6mo 9mo 12mo 18mo 4-6yr >4-

6yr

BCG

HBV1 HBV2 HBV3

DTP1&

OPV1

DTP2&

OPV2

DTP3&

OPV3

DTP4&

OPV4

DTP5&

OPV5

MMR1 MMR2

JE1&2

หางกัน

1-4

สัปดาห

DT

Page 15: Nle step 2_2009 si115-116 and nle_step_2_2009 nctms editors cut key

SIRIRAJ

SIRIRA

J SIRIR

AJ

เปน Inflammation and edema of the subglottic area causing airway obstruction in the

larynx, trachea, or bronchi. ตรวจรางกายมี inspiratory stridor มี sign ของ upper airway

obstruction ; สาเหตุสวนใหญเกิดจากเชื้อไวรัส(Parainfluenza virus) แตก็อาจเกิดจาก

bacterial infection ได เชน S. aureus, Mycoplasma pneumoniae, S. pneumoniae, S.

pyogenes, Hib (bacterial croup จะ present ดวย high fever & more severe resp. distress)

acute epiglottitis มักเกิดในเด็กอายุ 2-7 ป ทําใหเกิด supraglottic stenosis มี sign ของ

upper airway obstruction ไดเชนกัน และลักษณะ stridor เปน inspiratory stridor ผูปวยจะ

มีไขสูง, muffle voice + 4Ds symptoms (drooling, dyspnea, dysphagia, dysphonia)

Whooping cough (pertussis) เปนการอักเสบของทางเดินหายใจสวน bronchus มัก

เปน expiratory stridor มีอาการไอมาก อาจตรวจพบ subconjuctival hemorrhage, rectal

prolapsed ได lung มี croase crepitation, ไมคอยตรวจพบวามี tachypnea การตรวจ CXR

อาจพบ peribronchial thickening

9.) ตอบ 3. Retropharyngeal abscess

ตรวจรางกายพบ Bulging posterior pharynx

10.) ตอบ 2. ICD

ผูปวยนาจะมี Pneumonia with parapneumonic effusion เยอะพอสมควร trachea ถึง

จะshift ไมแนใจนาจะ 2. ICD นะ เพราะถา 1. mycoplasma titer จาก mycoplasma

pneumomiae จะเปนแบบ walking pneumonia คืออาการไมคอยมาก แต CXR ดู severe อะ

ก็พบวาทําใหเกิด pleural effusionได แต rare อะ อาการของ mycoplasma จะ non-specific

onset - insidious, with fever, malaise, headache, and cough. Cough is a hallmark of M

pneumoniae infection และก็มี extrapulmonary manifestation ได เชน มีผื่น maculo

papular, erythema multiforme, bullous myringitis, musculoskeletal เชน ปวดขอ etc. [

nasopharyngeal swab ไมรูนะวาขอนี้เคาตั้งใจจะใหเปน Flu เพราะชวงนั้น Fluระบาด]

11.) ตอบ 3. CXR PA, lateral

เด็กมี inspiratory stridor ซึ่งเปนลักษณะของ upper airway obstruction และมี rhonchi

ซึ่งเปนลักษณะของ lower respiratory tract obstruction โรคที่ทําใหเกิดลักษณะเชนนิ้ คือ

Croup = Viral laryngotracheobronchitis (larynx = upper, bronchus = lower) การตรวจ X-

ray ที่ neck AP พบ pencil sign, steeple sign Lateral พบ ballooning of hypopharynx จึง

ตอบ CXR PA,lateral เพราะสามารถดูสวนคอไดดวย

12.) ตอบ 3. SVC obstruction

classic symptoms and signs become more obvious. Dyspnea is the most common

symptom , Other symptoms include facial swelling, head fullness, cough, arm swelling,

chest pain, dysphagia, orthopnea, distorted vision, hoarseness, stridor, headache, nasal

stuffiness, nausea, pleural effusions, and light-headedness.

findings : venous distension of the neck and chest wall, facial edema, upper

extremity edema, mental changes, plethora, cyanosis, papilledema, stupor, and even

coma.

13.) ตอบ 2. Acute Renal Failure

จากโจทย นาจะเปน MAHA ที่พบบอยในเด็กคือ HUS โดยจะมีอาการนําคือ มีไข

ถายปนมูกเลือด โรคนี้เปน vascular-occlusive disorder จึงมีการแตกของเม็ดเลือดแดง

เกล็ดเลือดต่ําได & จาก U/A แสดงวานาจะมี intravascular hemolysis ก็ควรระวัง Acute

Renal Failure

14.) ตอบ 4. ANA

เหมือนมี multiorgan involvement ( Hematology, mucosal involvement , kidney

involvement) เหลืออีกหนึ่งอยางจะครบ criteria SLE แลว ก็ตรวจANA ไปเลยดิจะได

วินิจฉัย

Page 16: Nle step 2_2009 si115-116 and nle_step_2_2009 nctms editors cut key

SIRIRAJ

SIRIRA

J SIRIR

AJ

15.) ตอบ 2. Hypokalemia

ตามเทาที่โจทยใหมานะ ไมรูวาอาเจียนเพราะวาอะไร ตอนแรกคิดวาถาเปนพวก

obstruction ตางๆ มันก็จะขึ้นกับตําแหนงที่มีการอุดกั้น เพราะสวนประกอบที่ออกมาก็จะ

ตางกัน แตจาก choice ที่ใหมาเนี่ย ก็คงเห็นแลววามันก็เปนไดทุกอันนั่นแหละ เอาแบบไม

คิดมาก ก็คิดวานาจะเปน hypokalemia นะ เพราะโจทยบอกวาอาเจียนคอนขางมาก เหตุผล

ก็คือ การอาเจียนหรือถายเหลวมาก ๆ จะทําใหมี volume depletion และเกิด metabolic

alkalosis (ยังกอน ๆ ) ทั้งสองอันเนี่ยมันจะไปมีผลใหมีการขับ K ทิ้งที่ไตเพื่อเก็บ Na กลับ

มาก (volume depletion ทําใหเกิด secondary hyperaldosteronism สวน metabolic alkalosis

จะไปมีผลตอ Na/K/H channel ที่ collecting tubule อะ พอมันเปน alkalosis ก็เลยตอง

พยายามเก็บทั้ง H , Na ดวยเลยตองขับ K ทิ้งไป)

จริง ๆ ในสวนประกอบของอาเจียนมันก็มี K นอยมาก 10 mEq/L แตดวยความบาบอ

ของกลไกขางตน เลยตอบขอสองนะจะ

16.) ตอบ 4. hypochloremic hypokalemic metabolic alkalosis

ภายหลังการอาเจียนรางกายจะเสีย proton และ chloride ไปโดยตรง ทําใหเกิด

hypochloremic metabolic alkalosis รางกาย compensate จากการเสีย proton โดย แลก K

กับ H+ ในเลือด ทําใหพบ hypokalemia รวมดวย

17.นาจะตอบ K diet ลดลงนะ....

- ขอนี้คิดถึง chronic renal failure เพราะมีการติดเชื้อที่ urinary tract บอยๆและ BUN

Cr ขึ้นเรื่อยๆ

วิธีการรักษา CRF

- 1. Phosphate restriction starting early in chronic kidney disease

- 2.Potassium restriction

- 3.Sodium and water restriction as needed to avoid volume overload

18.) ตอบ 2. TFT

นาจะเปนภาวะ congenital hypothyroidism ซึ่งมีอาการไดแก

1. ตัวเหลืองนาน (Prolonged Jaundice)

2. ถายอุจจาระแรกเกิดชา (Delayed passage of stools)

3. อุณหภูมิกายต่ํา (Hypothermia)

4. ความตึงตัวของกลามเนื้อลดลง (Poor muscle tone)

5. ตัวลาย (Mottled skin)

6. ดูดนมไมดี (Poor feeding)

7. มื่ออายุมากขึ้นจะพบมี สะดือจุน ลิ้นจุกปาก กระหมอมปดชา และไอคิวจะลดลงไป

เรื่อยๆ ตามวันเวลาที่เพิ่มขึ้น

19.) ตอบ 4. กลับบานไมได ให Observe

นาจะเปนเด็กคลอดครบกําหนด ดูจากน้ําหนัก กินนมไดดี เหลือง 12 mg/dl ที่วันที่ 3

คงเปน breast milk jaundice ถือเปน physiologic jaundice มี

1. ไมใชเพราะกินไดดี ไมนามี dehydration, Hct ก็ไมขนไป

2. ไมใช เพราะเหลืองไมมาก ไมควรไปหยุดนมแม เดี๋ยวเกิด breast feeding

jaundice เปลาๆ

3. ตากแดดออนๆ สู photo ไมงายกวารึ

4. เกณฑ on photo day 3 อยูที่ >20 mg/dl แถมยังไมไดตรวจวาเปน direct หรือ

indirect ดวย

20.) ตอบ 5. เหลือง

Risk factor ของ neonatal SNHL ไดแก - familial Hx , ToRCHeS infection ,

craniofacial anomalies , BW<1500g , Jx with exchange transfusion , ototoxics with

loop diuretics , viral/bact. Meningitis , APGAR 0-4@1min, 0-6@5min - associated

syndrome , on mechanical ventilator/ admit NICU > 5day

Page 17: Nle step 2_2009 si115-116 and nle_step_2_2009 nctms editors cut key

SIRIRAJ

SIRIRA

J SIRIR

AJ

-late complication ของ neonatal jaundice - - > chronic kernicterus: athetoid cerebral

palsy, delayed motor skill, mental retardation, SNHL

21.) ตอบ 4. perinatal asphyxia

มี Hx postterm, meconium stained, Apgar แย, มีชัก โดย glucose กับ Hct กอโอเค

สาเหตุชัก จึงคิดถึง perinatal asphyxia มากสุด ( meningitis นาจะเปนประวัติติดเชื้อ เด็ก

preterm, low BW / สวน IVH ควรเปน preterm premature เชนเดียวกัน )

22.) ตอบ 3. RDS

Respiratory distress syndrome (RDS), หรือ hyaline membrane disease (HMD) มัก

เกิดใน premature infants riskที่สาํคัญคือ GDM ผลทําใหขาดสารลดแรงตึงผิว เกิดภาวะถุง

ลมแฟบไมขยายตัว ผุปวยตองใชแรงการหายใจมาก ทําใหเกิด V/Q mismatchขึ้น

ตรวจรางกายพบ Tachypnea , Expiratory grunting (from partial closure of

glottis) , Subcostal and intercostal retractions , Cyanosis , Nasal flaring

CXR พบ bilateral, diffuse reticular granular or ground-glass appearances, air

bronchograms, and poor lung expansion

23.) ตอบ 2. Patau’s (midline defect ตองคิดถึงโรคนี้)

สวน Down syndrome (trisomy 21) , Turner’s (45xo) , Klinefelter’s (47xxy) ,

Edward’s(Trisomy 18) มีลักษณะเดนคือ Rocker bottom เทาคลายเกาอี้โยก ,clench hand

ผิดปกติ

24.) ตอบ 5. Pulmonary arterial hypertension

เนื่องจากทารกที่มีภาวะ meconium aspiration syndrome มีความเสี่ยงสูงที่จะเกิดภาวะ

persistent pulmonary hypertension of the newborn ตามมา

25.) ตอบ 1. early feeding (ไมแนใจวา 4. ดวยปาว)

ภาวะเลือดขนในทารก ( Neonatal Polycythemia) = ภาวะที่ทารกแรกเกิดมี Hct

>65% พบในทารกเพศชายมากกวาเพศหญิง , สาเหตุ Placenta Transfusion เชนผูกสาย

สะดือชา ,Transfusion ระหวางทารกคูแฝด,Transfusion เลือดจากมารดาสูทารก ภาวะขาด

ออกซิเจนเพราะรกทํางานไดไมดี ทารกตัวเล็กกวาอายุ,ครรภเกินกําหนด,ยาบางอยางเชน

Propanplol ? ภาวะผิดปกติของตอมไรทอ เชนมารดาเปนเบาหวาน ตอมหมวกไตทํางาน

ผิดปกติ อื่นๆเชน Trisomy 21,13 หรือ 18 ฯลฯ

อาการ อาจพบ 2-6 ชม.หลังคลอด และพบกอนอายุ 48 ชม. อาการที่พบคือตัวแดง ตัวเขียว

หายใจลําบาก สั่นหรือชักกระตุก นอกจากนี้อาจมีอาการซึม อาเจียน ตัวเหลือง รองกวน

หรือหยุดหายใจเปนตน อาการที่อาจพบรวมกับภาวะเลือดขน

การรักษา รักษาตามอาการ ถาขนมากตองทําการเปลี่ยนถายเลือด

ปริมาตรที่ใช = น้ําหนักตัว(กก.)x90(Hct เริ่มตน-Hct ที่ตองการ) Hct เริ่มตน

Hypoglycemia DTX < 40 mg%

จะทํา Partial exchange เมื่อ Hct>70 or Hct>65+มีอาการ

Screening BS in infants with risk factors : Premature infants , SGA , Infant of

diabetic mother , LGA , Sepsis , Respiratory distress , Asphyxia

Symptoms : Lethargy, apathy, and limpness , Apnea , Cyanosis , Weak or high-

pitched cry , Seizures, coma , Poor feeding, vomiting , Tremors, jitteriness, or

irritability , Seizures

Asymptomatic hypoglycemic infants

- No contraindication for feeding - Early feeding

- With contraindication for feeding - IV fluid

Symptomatic hypoglycemic infants - IV fluid start with

Minibolus : 200 mg/kg over 1 min / 10% D/W 2 ml/kg

Maintenance : Start with GPR 4-8 mg/kg/min & Increase by 2 mg/kg/min

27.) ตอบ 1. INH 9 mo

เด็ก< 5 yr ที่มีประวัติ contact TB ทุกราย ถึงแมจะไมมีอาการ ให Ix PPD, CXR (ทั้ง

เด็กและfamily)

ถา Ix neg TB prophylaxis : INH 9 mo

Page 18: Nle step 2_2009 si115-116 and nle_step_2_2009 nctms editors cut key

SIRIRAJ

SIRIRA

J SIRIR

AJ

ถา asymptomatic + Ix pos Rx INH 9 mo. If INH resistant : rifampicin 6 mo

PPD (purified protein derivative) positive when

>5 mm if ไมเคยได BCG และcontact ผูปวย TB ที่ AFB pos, ผล CXR เขาไดกับ

TB, immucocompromised host (HIV,CMT Rx)

> 10 mm if immune ต่ํา เชน DM, chronic renal failure, อายุ <4 yr, เด็กที่ contact

ผูใหญที่มีความเสี่ยงสูง

>15 mm if อายุ >4 yr, no risk

28.) ตอบ 1. OPV , DTP

ดูตามตารางวัคซีนขอ 4 สวน Hib เปน optional vaccine ถาตองการให จะให

ตอนอายุ 2,4,6 mo เพื่อปองกันเยื่อหุมสมองอักเสบ

แนะนําใหฉีดในเด็กที่อยูบานเดียวกับผูปวยที่ติดเชื้อ Hib รุนแรง และเด็กทุกคนที่

อายุนอยกวา 2 ป แมเคยเปน Hib มากอน โดยเฉพาะเด็กที่อยูในชุมชนแออัด หรือสถานรับ

เลี้ยงเด็ก เด็กที่ภูมิคุมกันบกพรองและเสี่ยงตอ invasive Hib เชน Thalassemia, asplenia, ซึ่ง

ควรใหในทุกอายุ

29.) ตอบ 2. เกิดจาก HSV 16

Roseola infantum หรือ Sixth disease เชื้อกอโรค คือ HHV6 สวนใหญมักติดเชื้อใน

เด็กอายุต่ํากวา 2 ขวบ โดยจะมีอาการ คือ ไขประมาณ 3-5 วัน หลังไขลงจะมี MP rash ขึ้น

เริ่มจากตัว ไปยังลําคอ และแขนขา (self limited disease)

The rash of roseola appears as the fever abates. It starts on the neck and trunk and

spreads to the extremities. As depicted above, it is erythematous, blanching, and macular

or maculopapular.

30.) ตอบ 1. Naegleria fowleri

ประวัติเลนน้ํา/น้ําตก + meningitis นึกถึง Naegleria fowleri ซึ่งเปน amoeba ทําใหเกิด

primary amebic meningoencephalitis ผานทางการสําลักน้ํา แลวเชื้อเขาสูรางกายทาง

olfactory epithelium ขึ้นสมองโดยตรง ผูปวยมาดวย sudden onset of bifrontal or

bitemporal headache, fever, nausea, vomiting, stiff neck, progress rapidly to lethargy,

confusion, and coma ถาไมรักษามักตายใน 48 to 72 hours

อาการแยกจาก acute pyogenic bacterial meningoencephalitis ไมได อาจนํา CSF มา

สองกลองหาเชื้อ, Rx โดยเร็วดวย high dose Amphotericin B หรือ miconazole

31.) ตอบ 1. D-Dimer

ผูปวยมี clinical ของ decompensated DIC (Plt decreased, PT/APTT prolong,

fibrinogen level decrease) with impending shock ; สาเหตุ อาจมาจากการติดเชื้อบางอยาง

(lymphocyte เดน แต WBC ต่ํา) ไมรูอะโจทยไมสมบูรณนะเธอว

Investigation d-dimer, FDP เปนสวนของ cross-link fibrin degradation

32.) ตอบ 3. Platelet

ในผูปวยรายนี้มีไขสูง (นาจะมี infection) รวมกับมี petechiae ที่แขนขา ซึ่งเปน sign ที่

บอกวานาจะมี thrombocytopenia สําหรับ BP ในผูปวยรายนี้ยังอยูในชวงปกติ เนื่องจาก

SBP ในเด็กปกติเทากับ 70 + (yr x 2)

33.) ตอบ 5. Tympanocenthesis

อาการที่ใหมาทําใหนึกถึงวาเด็กมีภาวะ AOM มากที่สุด คือ มีอาการปวดหู มีไข การ

ไดยินเสื่อมลง รอยละ 90 จะมีประวัติของ URI symptoms นํามากอน ถาตรวจ TM จะพบ

แกวหูบวมแดงหรืออาจจะโปงนูน ถามีหนองขังจะมีอาการปวดหูมากขึ้น เมื่อตรวจดวย

pneumatic otoscope พบแกวหูขยับไดนอยลง ตรวจไมพบ light reflex บนแกวหู มีคราบ

Page 19: Nle step 2_2009 si115-116 and nle_step_2_2009 nctms editors cut key

SIRIRAJ

SIRIRA

J SIRIR

AJ

เลือดหรือหนอง เมื่อหนองในหูชั้นกลางแตกออกมาแลวจะพบวาอาการปวดหูลดลง โดยที่

รูเล็กๆนี้จะสามารถปดเองไดใน 2-3 สัปดาหเมื่อหายจากการอักเสบ มักเกิดจากเชื้อ

S.pneumoniae, Hib, M.catarrhalis

การรักษา : เริ่มตน first choice ควรใหเปน amoxicillin แลว F/U 48-72 hr. ถายังไมดี

ขึ้นใหเปลี่ยนยาเปน second line drug เชน amoxicillin/clavulanate, cefuroxime,

ceftriaxzone หรืออาจใชวิธีการผาตัด ถาF/U first drug ไปแลวผูปวยไมดีขึ้นเชน ยังมีไข

สูง ปวดหูมาก ยังมีอาการไมสบายมาก มีไขซึมลง มีอาการแทรกซอนเกิดขึ้นหรือเปน

ผูปวยที่มีภูมิคุมกันบกพรอง สิ่งเหลานี้เปนindication ในการทํา เพราะ จะชวยบรรเทา

อาการปวดและ สามารถนําหนองหรือของเหลวไปเพาะเชื้อได

ในรายนี้คิดวามีอาการปวดหูมากขึ้น การรักษาโดยการเจาะเยื่อแกวหูนาจะมีประโยชน

มากกวา (อานเพิ่ม จากหนังสือ ENT ของรามา หนา 83-86 ดูเพิ่มเติมละกัน)

34.) ตอบ 3. สงเสริมใหกินเกลือผสม iodine ทั้งหมูบาน

ผูปวยมีอาการของภาวะ hypothyroidism (อาการของ hypothyroidism ในเด็ก ไดแก

prolong jaundice after birth, constipation, poor feeding, lethargy, delayed development,

failure to thrive, hypothermia, delayed closure of AF&PF, macroglossia, hypotonia,

slowly response reflex, umbilical hernia) ที่ขีดเสนใตคือที่ตรงกับโจทย

เนื่องจากมีประวัติเด็กในหมูบานเปนกันหลายคน คิดวาสาเหตุอาจเกิดจาก Acquire

hypothyroidism จากการขาด iodine ซึ่งเปนสาเหตุที่พบไดบอยที่สุด การปองกัน

(primary prevention) คือ สงเสริมใหกินเกลือผสม iodine ทั้งหมูบาน

35.) ตอบ 1. Hypothyroidism

เด็ก 2 ปควรหนัก = 2(wt)+8 = 12 kg , ยาว = 6(wt)+77 = 89 cm เด็กคนนี้หนัก 6 kg,

ยาว 55 cm = failure to thrive, PF ปกติปด 1 ½ mo, AF ปด 1 ½ yr คนนี้ปดชา ชันคอไมได

= hypotonia

อาการของ congenital hypothyroid ไดแก

- Prolonged neonatal jaundice

- Hoarse cry

- Poor feeding

- Functional constipation (ทองผูกจาก m. ไมบีบตัว)

- Hypotonia

- Delayed closure of fontanelle

- FTT

- Coarse facies (หนาหยาบหนา), macroglossia

- Umbilical hernia

- Anemia

- Hypothermia

- Bone age ชา

Rx Levothyroxine (T4)

36.) ตอบ 1. Apt test

Common Upper Gastrointestinal Tract Bleeding in Neonates มี

- Hemorrhagic disease of the newborn self-limited bleeding disorder resulting

from a deficiency in vitamin K–dependent coagulation factors.

- Swallowed maternal blood hematemesis during the first few days of life. This

diagnosis is confirmed using the Apt test,

- Stress gastritis Stress gastritis occurs in up to 20% of patients cared for in the

neonatal intensive care unit. Prematurity, neonatal distress, and mechanical ventilation

37. ตอบ Acute rheumatic fever

GAS (gr.A Strep) strep sore throat, scarlet fever พบในอายุ 5-15 ป

Criteria dx = Modified Jones criteria : 2 major or 1 major+2minor รวมกับ

supportive evidence วามี strep inf ~3 wk กอนหนานี้ or ASO titer/anti-DNaseB positive

Major 1. Carditis เปนอาการแรกที่พบ มักมี MR or AR ตอมาอาจกลายเปน stenosis แทน

Page 20: Nle step 2_2009 si115-116 and nle_step_2_2009 nctms editors cut key

SIRIRAJ

SIRIRA

J SIRIR

AJ

2. ขอ migratory polyarthritis, large jt หายไดเอง response anti-inflam. Drug ดี

3. Chorea ; unpurposeful irreg. rapid mm., m. weakness, emotional instability

4. subcutaneous nodule กอนแข็งไมเจ็บอยูตามแนว spine, extensor surface, scalp

5. ผิวแดง erythema marginatum ขอบนูนแดงหยัก ไมคัน หายไดเอง

Minor 1. Fever 2. Arthralgia 3. 1st degree AV block 4. Increased ESR. CRP

Rx 1. Eradicate strep inf : Benzathine peniciliin/PenV/erythromycin

2. Supportive –Rx heart failure

- anti-inflam. Drug ใหตาม severity of arthritis ตย.ยา prednisolone, ASA

- F/U ESR, CRP q 4-6 wk

3. Secondary prophylaxis : Benzathine penicillin 1.2 mU q 4 wk / PenV/sulfadiazine

oral - s carditis 5 yr

- c carditis 10 yr or ตลอดชีวิต

38.) ตอบ 4. ASD

เนื่องจาก ฟงได widely fixed split S2 ตําแหนงของ murmur ที่ Lt.upper sternal border

และปกติแข็งแรงดี

AS - mid-systolic ejection murmur, heard best over the “aortic area” or right second

intercostal space, with radiation into the right neck

VSD – pansystolic murmur at Lt.lower sternal border

39.) ตอบ 1. Eisenmenger

จากโจทย ตอนแรกก็เดากอนวาเด็กมันจะเปนหัวใจแบบไหน เคยเขียว หอบเหนื่อยมา

กอน และมี clubbing finger แสดงวามี chronic hypoxemia มาตลอดตั้งแต 2-3 ป แปลวา

นาจะเปนหัวใจไมเขียวที่เปนไมมาก ทําใหสามารถ compensate ไดนานขนาดนี้ คิดวาอิ

หนูนี่นาจะมี VSD คือตอนแรก ๆ เกิดมาที่มีอาการหอบเหนื่อย

นาจะเปน 2-3 เดือนมากกวานะ ยังมี pulmonary pressure สูงอยูทําใหไมมีอาการ

หลังจากเริ่มหายใจ เริ่มมี pulmonary resistance เพิ่มขี้น flow ก็เลยเขายากขึ้น VSD จะเปน

ซายขวา shunt จนกวา ความดันทาง pulmonary จะมากกวา systemic ทําใหเกิดขวาซาย

shunt ขึ้น มีเขียวและหอบเหนื่อยมากขึ้นจากการมี pulmonary hypertension ซึ่งตรวจ

รางกายได diastolic blowing murmur ที่ left second and third intercostal space

แตอิหนูนี่มันอาจจะซับซอนกวาที่คิด เพราะมันอาจเปน TOF ไดเชนกัน อาการก็ดูเขา

ได ไมรูวามันได

ผาตัดไปกอนหนานี้ปาว เพราะมันควรผาตอนอายุ 1 ป ถามันยังไมผา อิหนูคงเดดไป

แลวหละ ถาผาแลวถามวามันจะมี hypoxic spell อีกอะปาว อันนี้ไมรูอะ เพราะดูอายุปาเขา

ไปสิบขวบ ดูไมคอยจะเปนเลย อิหนูพี่ยอมแพ ตอบขอหนึ่งละกันนะ คิดไดแคนี้แหละ

40.) ตอบ (ไมแนใจอะ)

จากประวัติยังไมสามารถบอกไดชัดเจนวา dyspnea และ cyanosis เกิดจากอะไร

สาเหตุที่นึกถึงหลักๆ คือ Pulmonary cause (R/O TTNB จากประวัติ เหนื่อยแลวเขียว)

และ Cardiac cause (R/O congenital heart disease + hypoxic spell)

Mx : 1.Pul.cause : TTNB : adequate oxygenation (ตรงนี้เราวานาจะใช hood ก็พอ

PPV ไมแนใจ)

2. Cardiac : ตรงนี้เนื่องจาก congen. heart ที่เขียวแตแรกเกิดมักตองการ การผาตัดทุก

ราย จึงควรสงตอ อยางรวดเร็ว โดยการดูแลเบื้องตนคือการแกไขอาการเบื้องตนที่มีกอน

การสงตอ เราคิดวาผูปวยอาจมีปญหาที่สามารถแกไดคือความดันที่ลดลงการให NSS อาจ

ชวยได (อาจตองดูโจทยเพิ่มเติมวา ตอนนี้มีปญหาใดเปนหลักและเราคิดวา ในคําตอบขอ

อื่นไมนาจะแกไขสาเหตุไดชัดเจน)

คิดวาให O2 ไดแตไมถึงกับ PPV เพราะบาง congen heart จะ keep O2-sat ไมสูงนัก

Hypoxic spell ยังไมแนใจจากประวัติ เพราะสวนใหญพบในเด็ก 6mo-2yrs แตเด็กแรก

เกิดก็พบได การให O2 จึงนาจะเกิดประโยชนในเด็กกลุมนี้

41.) ตอบ 1. 0.9% NaCl 20 ml/kg

การใหสารน้ําเบื้องตนใน DKA พิจารณาจาก degree of dehydration

- <7% dehydration : BW ลด < 7%, HCO3 >17 mmol/L

Page 21: Nle step 2_2009 si115-116 and nle_step_2_2009 nctms editors cut key

SIRIRAJ

SIRIRA

J SIRIR

AJ

-7-10% dehydration : BW ลด 7-10%,HCO3>10 mmol/L

-10-15% dehydration: BW ลด 10-15%,HCO3<10 mmol/L

ผูปวยรายนี้มี HCO3 = 10mmol/L ~7-10% dehydration เริ่มให 0.9%NSS 10cc/kg. in

30min if shock ให bolus 0.9%NSS 20 cc/kg.

กรณี < 7% dehydration : ให 0.45%NSS 3000 ml/m2/24 hr. + K 40 mmol/L start K:

when < 6 mmol/L & urine flow ดี

42.) ตอบ 5. Bonchoscopy

นาจะเปน Foreign body obstruction เนื่องจาก...1. อาการหายใจลําบากเกิดขึ้น

ทันทีทันใด , 2. เด็ก เปนวัยที่ชอบควาสิ่งของเขาชองปาก ,3. มี stridor และ sign อื่นๆของ

partial airway obstruction (ปามาณวา ถา complete obstruct นาจะเขียว , อาการดูแยๆ และ

อาจไมไดยินเสียงการหายใจอะ)

Rx : 1.Partial obstruct : ใหไอแรงๆ , หามใชนิ้วลวง , ถาไมออกควรสงไปทํา

Bronchoscopy

2. Complete obstruct :

- เด็ก < 1 ป ใหทํา Back blow-chest thrust เพื่อเพิ่ม ความดันในปอดใหสูงขึ้น

จนสามารถดันสิ่งที่อุดตันใหหลุดออกมาจากทางเดินหายใจ ออกไปได

- เด็กโต > 1 ป ใหทํา Heimlich maneuvers หรือ sub- diaphragmetic abdominal

thrusts ดวยการดันบริเวณลิ้นปหรือใตกระบังลม เพื่อเพิ่มความดันในปอดใหสูงขึ้นจน

สามารถ ดันสิ่งที่อุดตันใหหลุดจากทางเดินหายใจ

43.) ตอบ 1. Plasma Glucose

Neonatal convulsion เกิดไดจากหลายสาเหตุดวยกัน ไดแก perinatal cause (HIE,

cerebral hemorrhage), CNS malformation, Metabolic disturbance, Infection, Toxic and

idiopathic newborn convulsion ในเด็กรายนี้พบวา APGAR ดี และมีน้ําหนักตัวที่มากเกิน

ปกติ 4200 gm คาดวาเด็กนาจะมีภาวะ macrosomia (เด็กมีน้ําหนักมากกวา 4000 gm หรือ

น้ําหนักมากกวา 90 เปอรเซ็นตไทลของอายุนั้นๆ) ที่เกิดจากมารดาเปนเบาหวาน ทําใหเกิด

โรคแทรกซอนในเด็กที่มีผลตอการชักไดคือ neonatal hypoglycemia และ neonatal

hypocalcemia ดังนั้น investigation ที่เหมาะสมควรจะทํากอนคือ bedside glucose หรือ

เจาะหา capillary blood sugar และสงตรวจ lab กลาง serum blood sugar, electrolyte เพื่อ

เปนการยืนยันนั่นเองครับ

44.) ตอบ 5. สงตอ neuro Pred.

คิดวานาจะเปน infantile spasms (West syndrome) ลักษณะอาการในโจทยก็เขา

ไดอะนะ คือมี ผงกศีรษะ เกร็งแขนขา เปนชุดๆ ไมเกิน 2-3 นาที และก็มักเปนหลังตื่นนอน

ดวย(ไมเปนตอนหลับ)

โรคนี้การรักษามีดังนี้ : First line drug : vigabatrin ตามที่อ.สอน แตบมีchoice , ยา

อื่นๆ เชน valproic acid, topiramate ก็บมีchoice อีก เลยตอบขอ 5 ละกัน สวนขอ 4 สง

กระตุนพัฒนาการก็นาสน เพราะโรคนี้มักเกิด Mental retard. ภายหลัง แตก็คิดวาไมได

เปนทุกคนเลยไมจําเปนมั้ง สรุปสงหมอเด็ก Neuro. เลยดีกวา

45.) ตอบ 3. CT brain

anterior fontanel โปง แสดงถึง sign of increased intracranial pressure เปน C/I ใน

การ LP / ดังนั้นจึงสง CT brain scan

46.) ตอบ 4. Valproic acid

คิดถึงภาวะ Childhood Absence Epilepsy ซึ่งพบไดในเด็กอายุ4-10ป พบบอยชวง5-7

ป ขณะชักมีลักษณะแบบabsence seizure เรียกไมรูตัว เกิดขึ้นทันทีนานประมาณ 4-20

วินาที อาจพบautomatismรวมดวย วินิจฉัยโดยใหผูปวยทําhyperventilation test จะกระตุน

ใหเกิดอาการ ตรวจคลื่นไฟฟาสมองจะพบ 3Hz generalized spike and wave

First line drug คือ Valproate / second line drug คือ Lamotrigine อยางนอย2ป

47.) ตอบ 1. Diazepam

เปน status epilepticus นิยามคือ ชักนาน 30 นาที หรือชัก 2 ครั้ง โดยชวงหายชักไม

full recovery การรักษาเบื้องตน ให diazepam 0.3 mg/kg IV *2 then Phenytoin 20mg/kg

Page 22: Nle step 2_2009 si115-116 and nle_step_2_2009 nctms editors cut key

SIRIRAJ

SIRIRA

J SIRIR

AJ

in 20 min ซึ่งตองใหในสารละลายที่ไมมีน้ําตาล ถาไมไดผลให Phenobarbital ซึ่งตองระวัง

ยาสามารถกดการหายใจและทําให hypotension ได

48.) ตอบ 4. Hb typing

เนื่องจากผูปวยมี hepatosplenomegaly PBS เปน hypochromic microcytic anemia,

anisocytosis poikilocytosis 2+ รวมกับพบ schistocyte,ovalocyte และ target cell

49.) ตอบ 2. Iron def

เปน hypochromicmycrocytic anemia เพราะวามีประวัติเลี้ยงดวยนมขนหวานซึ่งไมมี

ธาตุเหล็กอยู ถาหากเปนพวก thalasemia นาจะมีอาการตั้งแตแรกเกิดและมีประวัติอยาง

อื่นดวยเชนตับมามโต หรือ ประวัติครอบครัว G6PDนาจะมีประวัติอยางอื่นเพิ่มเชน

precipitating factor สวน folate def จะเปน megaloblastic anemia

51.) ตอบ 2. G6PD deficiency

คิดถึง G6PD def มากสุด โดยจากโจทย เปน เด็กชาย , เปน acute ,ไข (infection

induce ใหเกิด acute hemolysis ในผูปวยโรคนี้ได) , pale, jaundice ไมมีตับมามโด ( ทําให

คิดถึงโรคในกลุม chronic hemolytic anemia เชน Hb H disease, HS ไดนอยกวา) แลวก็

blood smear อาจพบเปน Heinz bodies, contractec RBC ได

52.) ตอบ 5. APDE

เพราะผูปวยมีแตอาการของ bleeding disorder ผล CBC พบ eosinophil predominate

จํานวน platelet ปกติ ถาดู blood smear จะพบ eosinophilia, giant platelet, pale platelet

53.) ซ้ํา ตอบ thalassemia

56.) ตอบ 1. T cell

กขอนี้ เปน TB ซึ่งเปน intracellular แลวก onset เร็ว ติดไวรัสอีกตางหาก กนึกถึง T-

cell อยูแลวอะ

ถา Phagocytosis มันก onset เร็วเหมือนกัน แตกอาจมีประวัติวา สายสะดือหลุดชา

สวน Ig หรือ B-cell เนี่ย มันจะ onset ชานะ เกิน 6 เดือน คิดวาเพราะชวงแรกเด็กนอย

ยังไดมาจากแมอยูอะ ^^ แลวกจาติดเชื้อประเภท Bronchopulmonary อะคะ พวก

S.Pneumo อาไรปามานนี้

Complement นี่ Onset varied คะ จาติดเชื่อพวก N.Gonorhea

Opsonization นี่ไมรูแฮะ แตไมเห็นเคยไดยินเทาไหรเรยอะ ไมนาออกนะ หุหุ

57.) ตอบ 1. ทา E ที่ L minora

ก็ภาวะนี้คือ Labial Adhesions หายไดเองใน 1 ป ประมาณ 80% ถามี S/S หรือ พอแม

request กทา E bid 2-4 wk จน separate แลวกทา emollient หรือ ATB oint 3-5 ครั้ง/d เพื่อ

complete healing & prevent recur คะ

OB&GYN1

1.) ตอบ 3. DMPA

ประเด็นของการคุมกําเนิดผูปวยรายนี้ คือ - ตองการใหนมบุตร >>>> ไมควรใชยาคุม

ที่มี estrogen จึงตัดขอ combined pill ออกไป

- ตองการคุม 2-3ป >>>> จึงตัด progestogen-only only pill ออกไป เนื่องจากถาใช

จะตองกินนาน ติดตอกัน 2-3 ป ทุกวัน ซึ่งจะมี SE มาก นอกจากนี้ยังตองกินทุกวันและ

ควรกินตรงเวลาจึงอาจลืมกินได จึงมีโอกาส failed สูง

- ตองการลดปวดทองนอย(endometriosis) >>>> ปกติการรักษา endometriosis นิยม

ใหฉีดDMPA จึงสนับสนุนวาตอบ DMPA

**ยาเม็ดคุมกําเนิด (oral contraceptions) แบงออกเปน

- combined pills มี estrogen+progestrogen; SE=N/V, Wt gain, spotting, breast

tenderness, melasma, gallstones, HTN, thromboembolism, CAD

- minipills (progrstrogen-only pills) มี low dose progestrogen; SE=irregular menses,

edema, hirsutism, decreased liver func., แตไมคอยมี N/V กับ breast tenderness

- postcoitol pills มี high dose progestrogen

Page 23: Nle step 2_2009 si115-116 and nle_step_2_2009 nctms editors cut key

SIRIRAJ

SIRIRA

J SIRIR

AJ

หลังหยุด oral contraceptions แลวจะ return of fertility ประมาณ 3 เดือน

**ยาฉีดคุมกําเนิด (injectable contraceptions) เปน long-acting progestogen ไดแก

- DMPA(Depo-medroxyprogesterone acetate) ฉดีIMทุก 12 wks

- NET EN(Norethindrone enanthate)

SE=missed menses, spotting, Wt gain

หลังหยุด injectable contraceptions แลวจะ return of fertility ประมาณ 6 เดือน

**ยาฝงคุมกําเนิด มี 2 ยี่หอ ไดแก

- Implanon มี etonogestrel ฝงหลอดเดียว ได 3 ป

- Norplants มี levonorgestrel ฝง6หลอด ได 5 ป

SE=spotting, missed menses

หลังถอดหลอดยาออกแลวจะ return of fertility ประมาณ 1 ป (80%)

2.) ตอบ 2. Pelvic Outlet แคบ

คือใหเราประเมิน Clinical Pelvimetry ไดดังนี้

1. Pelvic Inlet : - Obstetic conjugate (ระยะที่สั้นที่สุดระหวาง promontory of sacrum

กับ pubic symphysis) จะ > 10 cm

- Diagonal conjugate(ระหวาง POS กับขอบลาง PS) ควรจะ > 11.5 cm.

- ประเมินคราวๆโดย ถาหากคลํา sacrumpromontery ไดก็ใหสงสัยวาอาจจะแคบ

2. Midpelvic : ดูจาก interspinous diameter ควรจะยาว > 10 cm.

3. Pelvic Outlet : ดูจาก intertubulous diameter (ระยะระหวาง ischial tuberosity 2

ขาง) ควรจะ > 8 cm. & ดู subpubic angle ควรจะกวางกวา 90 องศา / ซึ่งในรายนี้มี

intertubulous diameter = 8 cm. เอง

3.) ตอบ 4. Trophoblastic disease

antepartum hemorrhage (1st trimester) - cause: abortion, blighted ovum, ectopic

pregnancy, molar pregnancy, local cause: polyps, cervicitis, ca cervix

molar pregnancy : present with painless vaginal bleeding ; uterus may be larger

than expected(uterus at pubic symphysis = 12 wk) ; hyperemesis ; Sometimes there is an

increase in blood pressure along with protein in the urine

4.) ตอบ 3. indirect Coombs test

จากโจทยครั้งนี้เปนการตั้งครรภครั้งที่2 สิ่งที่ตองระวังคือภาวะRh incompatibility ซึ่ง

ทําใหเกิดHemolytic disease of the Newborn(HNB) ได โดยปจจัยเสี่ยงคือ แมเปนRh-

สวนพอเปนRh+

ถาลูกคนแรกเปน Rh- ก็ไมมีปญหาอะไร เพราะแมวาเลือดลูกจะสัมผัสเลือดแมตอน

คลอด ก็ไมมีantigenไปกระตุน แตถาลูกคนแรกเปน Rh+ และมีการสัมผัสกันระหวาง

เลือดแมและเลื่อดลูกขณะคลอด เม็ดเลือดแดงจากลูกซึ่งมีD antigen จะสัมผัสกับimmune

system ในเลือดแมเปนครั้งแรก ทําใหเกิด primary immune response ซึ่งให IgMและ IgGที่

titerต่ําๆ ซึ่งไมกอใหเกิดอันตรายในการคลอดครั้งแรกนี้

แตถาการตั้งครรภครั้งที่2 ถาลูกเปนRh+อีก และเกิดการสัมผัสกันระหวางเลือดแม

และเลื่อดลูกอีก คราวนี้จะเกิดresensitization ก็จะเกิด IgM และIgGตอD antigen เดิม แตจะ

มีปริมาณมากและสามารถผานเขาplacentaไปสูfetusได ทําใหเกิดantigen-antibody

complex และเกิด hemolysisในเด็ก (มักเปนชนิด extravascular hemolysis) ทําให เกิดภาวะ

ซีด,tissue hypoxia,acidosis ,มีhepatosplenomegalyจากการที่รางกายพยายามชดเชยโดย

การสรางเม็ดเลือดแดงขึ้นทดแทนจากตับและมาม(ที่เรียกวา extramedullary hemopoiesis)

ถารุนแรงมากอาจทําใหเกิดการทํางานของระบบหายใจและหัวใจลมเหลว ทําใหเกิดภาวะ

บวมน้ําทั้งตัวที่เรียกวา Hydrop fetalisได หากคลอดออกมาได ภายหลังการคลอด เม็ด

เลือดแดงที่แตกในเด็ก จะเกิดเปนunconjugate bilirubinจํานวนมาก(เด็กยังไมสามารถ

เปลี่ยนเปนconiugateไดเต็มที่) ซึ่งสามารถซึมผานcell membraneของสมองได ทําใหcell

สมองเกิดภาวะanoxia และ encephalopathy เด็กจะมีอาการซึม ไมดูดนม เกร็งหลังแอน

และมีความพิการทางสมองซึ่งเรียกวาKerniterus

Page 24: Nle step 2_2009 si115-116 and nle_step_2_2009 nctms editors cut key

SIRIRAJ

SIRIRA

J SIRIR

AJ

แตโจทยไมบอกอะไรเกี่ยวกับลูกคนแรกเลย ดังนั้น สิ่งที่ควรทําคือ ตรวจดูวา แมมีการ

สรางantibody ตอ D-antigen หรือยัง? โดยทําการตรวจจากเลือดของมารดา ดวยวิธี

Antibody Screening Test ซึ่งคือ การตรวจหาunexpected antibodies ซึ่งหมายถึง antibody

ตอantigenบนRBCที่ไมใชanti-A หรือ anti-Bและมีความสําคัญทางคลินิก(ซึ่งหมายรวาม

ถึงanti-Dที่เราตองการตรวจดวย) โดยวิธีการก็คือการทํา Indirect Combs test นั่นเอง (ยอๆ

นะ ก็คือนําreagent red cell group O ที่ตรวจวามีantigenครบทุกชนิดก็คือมีD antigenดวย

มาทําปฎิกิริยากับserumของผูปวยคือของแม ที่อุณหภูมิ37องศา แลวตอดวยการใส

antihuman globulin reagent

ลงไปเพื่อดูการเกาะกลุมของRBC โดยจะเกิดการเกาะกลุมขึ้นเมื่อRBCนั้นมีantibodyมาจับ

อยูที่ผิว ซึ่งนั่นก็แสดงวาserumของผูปวย(แม)มีAnti-D โดยแปลผลเปนระดับtiter จึงตอง

มาตรวจซ้ําเพื่อดูวา มีคาสูงขึ้นหรือไม แมวาตรวจครั้งแรกจะมีtiterต่ํา(ดูเหมือนเปน

unsensitized Rh-) ก็ตาม โดย U.S. Preventive Services Task Force (USPSTF) :Screening

for Rh(D) incompatibility แนะนําใหมาตรวจซ้ําตอนGA 24-28 week ถาสูงขึ้น ก็แสดงวา

เกิดRh incompatibilityขึ้นแลว ซึ่งคาtiterที่สูงบงบอกวาเปนกรณีที่severe ดังนั้นขอนี้จึง

ตอบขอ3 indirect Coombs test นะ

ขอ1. Anti-D (Rhogram) = anti-D ที่เปน immune Ab แตในที่นี้ RhoGAM เกิดจาก

การเตรียมขึ้นมาพิเศษ RhoGAM มีหนาที่ปองกันแม ไมให develop anti-D ตอลูก โดยมัน

จะไปจับกับ Rh+ RBC ของลูก ที่เขามาใน maternal circulation แลวกระตุน complement

ใหทําลาย RBC นั้นไป RBC จึงไมทันที่จะกระตุนใหแมสราง anti-D ได ดังนั้นขอนี่ ควร

เปนขั้นตอนตอมาถาตรวจแลวพบวาแมไมมีanti-D โดยการใหจะให28th week of

pregnancy and within 72 hours of delivery นะ

ขอ2. Direct Coombs = การตรวจเม็ดเลือดแดงที่ถูก sensitized ดวยแอนติบอดีแลว

ในรางกาย ( In vivo sensitized) ซึ่งใชเปนpostnatal investigation โดยตรวจเลือดของทารก

ที่สงสัยภาวะHNB ถาpositiveก็เปนการยืนยันวามี antibody-induced hemolytic anemia

เกิดขึ้นแลว จึงไมควรตอบขอนี้(เพราะในchoice ผูปวยยังมีทางเลือกที่ควรทํากอน)

ขอ4. Cordocentesis และ5.Amniocentesis ไมควรเลยเพราะในขั้นตอนการทํามี

โอกาสที่เลือดลูกจะไปโดนเลือดแมได ซึ่งเทากับเปนการกระตุนใหเกิดการสรางAnti-Dใน

ตัวแมขึ้นมา

ในกรณีที่แมมีanti-Dแลว ลูกคนที่2 นี้อาจจะเปนRh+ การรักษามีหลายทางขึ้นกับGA

ความรุนแรงของโรคเชน การทํา Intrauterine blood transfusion of red blood cells into the

fetus’s circulationอาจจะพิจารณาใหทํา ถาทารกมีการทําลายเม็ดเลือดแดงมากเกินไป

Early delivery, if the fetus develops complications พิจารณาใหทํา ในกรณีที่

ทารกพรอมที่จะออกมาจากครรภได เนื่องจากอยูไปในครรภ ก็จะไดรับ anti-D จากแมมา

ทําลาย RBC ไปเรื่อยๆ มีโอกาสเสียชีวิตได สวนมารดาถาตองมีการไดรับเลือด ในขณะที่

ระดับanti-D ยังไมลดลง ตองใหเลือดRh-(ตองไมมีD-antigen)เทานั้น ไมเชนนั้นแลวผูปวย

อาจตายจากการที่เลือดที่ใหจะไปจับการ anti-Dในตัวผูปวยแลว ตกตะกอน

5.) ตอบ 1. ตรวจ thalassemia

thallsemia screening ถา negative แสดงวาลูกจะไมเปน thallasemia ชนิดรุนแรง

OF+ , DCIP/MCV+ , DCIP(OF ตรวจ alfa,beta thal trait สวน DCIP ตรวจ

HbE,HbH,HbC) ถา positive ไปตรวจscreening สามี หรือตรวจ Hb typing ตอ แลวถาเปน

high risk DNA testing ตอ (เมื่อได thal 1 trait กับ thal1 trait เพราะจะR/Oวาไมใช alfa thal

2 homo)

6.) ตอบ 4. Ultrasound

ใช ultrasound ในการ diagnosis -> polyhydramnios is diagnosed if the deepest

vertical pool is more than 8 cm or amniotic fluid index (AFI) is more than 95th

percentile for the corresponding gestational age

Page 25: Nle step 2_2009 si115-116 and nle_step_2_2009 nctms editors cut key

SIRIRAJ

SIRIRA

J SIRIR

AJ Graph illustrating amniotic fluid index in a normal singleton pregnancy. The solid line is the mean AFI, the lower

dotted line is the 5th percentile value, and the upper dotted line is the 95th percentile value

Polyhydramnios

Hx : abdominal swelling out of proportion to the gestational age , with

abdominal discomfort

PE : abdomen is distended out of proportion to the gestational age

Investigation : Ultrasound

เรื่องของครรภใหญเกินอายุครรภ นึกถึง...

1. แมจําวันผิด

2. twin

3. polyhydramnios

4. แมเปนเบาหวาน

5. กอนใดๆที่ทําใหมดลูกมีขนาดใหญขึ้น เชน myoma

7.) ตอบ 3. C/S

จากโจทยคิดเปน Variable DC = การลดลงของ FHR ต่ําลงกวา baseline จนถึงจุด

ต่ําสุดอยางรวดเร็ว (ภายในเวลา 30 วินาที) โดย FHR ลดลงมากกวา 15 bpm และใช

เวลานานมากกวาหรือเทากับ 15 วินาที แตไมถึง 2 นาที นับจาก onset

สาเหตุจาก Umbilical cord compression (most often 2º to oligohydramnios)

Mx Supportive

1. เปลี่ยนทามารดาเปน trendelenberg

2. หยุดให oxytocin และใหสารน้ําใหเพียงพอ

3. ตรวจภายในเช็คสายสะดือยอย

4. ให O2 100% ดวย face mask

5. ± amnioinfusion

6. ± tocolysis ในรายที่จะผาตัดทําคลอดถาไมไดผล

8.) ตอบ 2. Oral tinidazole

เอาที่เดนๆนะ foul smell d/c เนี่ยกเกิดจาก Tricho BV ถาคันก Tricho Candida อันนี้

แลวขอนี้กให specific มามากๆ กคือ ตกขาวสีเขียวๆ มีฟอง มี strawberry Cx อีกตางหาก

เพื่อนๆกตองนึกถึง Tricho นะ สวนการรักษา มันเปน protozoa กให Metro หรือ

Tinidazole กได เหมือนกันคะ

9.) ตอบ 2. threatened rupture uterus

threatened rupture uterus = อาการคือ มดลูกหดตัวตลอดเวลา (tetanic contraction) ,

ปวดทองนอยบริเวณเหนือหัวหนาวอยางรุนแรง , Suprapubic tenderness , พบ Bandl’s

(pathological retraction) ring คือการตรวจพบรอยคอดที่มดลูก เปนรอยคอดที่อยูระหวาง

upper และ lower segment of uterus โดยจะเห็นเดนชัดเมื่อมีการคลอดยาก มดลูกไม

สามารถบีบไลทารกใหเคลื่อนตัวผานชองเชิงกรานลงไปได ทําใหสวนของ lower

segment ซึ่งบางกวาขยายตัวยืดออก ในขณะที่สวนบนหดตัวแข็งมากขึ้น [ สาเหตุที่ทําให

เกิดในผูปวยรายนี้คือ multiparity และการได oxytocin ]

10.) ตอบ 3. Ultrasound

โจทยขอนี้เปน Antepartum hemorrhage = abnormal vaginal bleeding เมื่อ GA

>28wk /20 wk ซึ่งทําใหคิดถึงภาวะดังตอไปนี้ , 1.Placenta previa Diag จากultrasound –

ดูpositionของplacenta , 2. Placental Abruption =Painful vaginal bleeding , Diagnosis –

clinical dx , อยางอื่นแคสนับสนุนUltrasound อาจพบเลือดออกใตเนื้อรกได 25% ภาวะทั้ง

2 เปน emergencyทั้งคู

Page 26: Nle step 2_2009 si115-116 and nle_step_2_2009 nctms editors cut key

SIRIRAJ

SIRIRA

J SIRIR

AJ

11.) ตอบ 2. ขนาด 100 2 tab หางกัน 12 ชม.

จากการสังเกตจากขนาดของยาขางลางนี้จะเห็นวา ethinyl estradiol ตอง≥ 100

microgram/ ครั้ง

12.) ตอบ 1. VDRL

ในขอนี้คิดถึงวา ผูปวยนาจะเปน hard chancre เพราะเปนแผลขอบนูน คิดวานาจะเปน

Syphilisไมเจ็บ การinvestigationจึงนาจะสง VDRL

13.) ตอบ 1. Herpetic vulva

Genital herpes พบ Lesions usually are vesicular or ulcerative on an erythematous

base,shallow ulcers form after the vesicles rupture on mucous membranes

Molluscum Contagiosum พบ Firm, smooth, umbilicated papules, usually 2-6 mm in

diameter (range 1-15 mm), may be present in groups or may be widely disseminated on

the skin and mucosal surfaces.

vulvovaginal candidiasis พบ Itching, soreness and/or burning discomfort in the

vagina and vulva ; Heavy white curd-like vaginal discharge ; Bright red rash affecting

inner and outer parts of the vulva, sometimes spreading widely in the groin to include

pubic areas, inguinal areas and thighs.

Primary syphilis พบ painless chancre at the site of transmission after an incubation

period of 3-6 weeks. The lesion has a punched-out base and rolled edges

secondary syphilis พบ condylomata latum and patchy alopecia. Condylomata latum

are painless, highly infectious gray-white lesions that develop in warm, moist sites.

14.) ตอบ 1. anovulation

รายนี้ อวน มี estrone มาก ซึ่งมาจาก peripheral conversion ที่ไขมัน ทําให hormone

สูงตลอดเวลา ไมมี surge จึงไมมีการตกไข รบกวน HPO axis คดิวารายนี้ ผิดปกติจากการ

anovulation

OB-GYNE 15(4) เนื่องจากคิดถึง threaten abortion ที่ GA 7 wks จึงทํา transvaginal u/s

ซึ่งเห็นไดชัดกวา transabdominal u/s

สวน B HCG ไมชวยเนื่องจาก urine preg test pos

15.) ตอบ 4. Transvaginal U/S

16.) ตอบ 3. F&C

กคนนี้นึกถึง E.CA เนอะ เพราะวา หมดประจําเดือนแลว มี Vg bleed ยังไงกตอง R/O

CA กตองไดชิ้นเนื้อมาตรวจ ที่ทํากันเนี่ย กไมได Bx นะ จะใชวิธี F&C คะ

Surgery

1.) ตอบ 2. Cholangio CA

จากความคิดของเรานะ ผูปวยนาจะมีการติดเชื้อที่บริเวณ Hepatobiliary system

รวมกับ U/S พบวา CBD dilate และ มี multihypoechoic mass ที่ตับ เราจึงคิดวา โรคนั้น

จะตองสามารถแสดง lesion ได 2 ที่ คือที ่CBD และ ที่เนื้อตับ ดวย จึงตัดสินใจตอบขอนี้

ที่ตัดขอแรกเลยคือ Amebic liver abscess เนื่องจากเกิดจากเชื้อ E.his ซึ่งคนกินเขาไป

เพราะฉะนั้น อาการที่จะเกิดรวมจะเปนอาการทองเสียนํามากอน ถายมีมูกเลือด แลวพวก

ที่เปนฝ ๆ เนี่ย กอนมันจะโต เพราะฉะนั้น ตับตองโตแลวก็กดเจ็บดวย

สวนขอตอมาที่ตัดคือ Pyogenic liver abscess เนื่องจากมักเกิดจากเชื้อ E.coli ซึ่งอาการ

ไข จะสูงกวา amebic abscess กออยางวาแหละ พยาธิ มีหรอวาจะสู bacteria ได แถมกอ

เปน abscess เหมือนกัน ตับโต กดเจ็บ เบื่ออาหาร คลื่นไสอาเจียน เหงื่อออกตอน

กลางคืน

สวน Infectious Cyst มันอยูเฉพาะในสวนของตับ มันจะโตชา ๆ กอคลาย ๆ กับซีส

ที่ไต มี 2 แบบ มีตั้งแตแบบ Solitary และ แบบ Polycystic มักมีแคไข ถาติดเชื้อ อาจ

ปวดทองถากอนซีสมีการแตก

Page 27: Nle step 2_2009 si115-116 and nle_step_2_2009 nctms editors cut key

SIRIRAJ

SIRIRA

J SIRIR

AJ

Hepatoma มักมีน้ําหนักลด ออนเพลีย ปวดทองตื้อที่ใตชายโครงขาว ราวไปที่ไหล

เนื่องจากตัวกอนไปดันกระบังลม อาจมีภาวะ portal hypertension รวมดวยได คลําตับ

จะโต แตกดไมเจ็บ เหมือน 3ขอขางบน

ขอที่เราตอบคือ cholangiocarcinoma เนื่องมาจาก สามารถมี lesion ได 2 ที่คือ

extrahepatic and intrahepatic โดยจะมาพบแพทยดวยอาการที่แตกตางกัน ถาอยูที่ทอ

น้ําดี กอตองดูวาอดุสมบูรณปาว ถาอุดสมบูรณกอจะทําใหเหลือง แตถาอุดไมสมบูรณกอ

จะไมเหลือง รวมกับอาจมีภาวะแทรกซอน เปน cholangitis ได และตัวlesion ที่อยูในตับ

นั้น อาจเปน ฝได จากการที่มี ascending infection จากทอน้ําดีเขาไปในตับได

2.) ตอบ 2. amylase หากระดับสูงมากกวา 1000 IU/ml

โรคนี้ผูปวยมีอาการ acute pancreatitis อาการปวดทองบริเวณลิ้นป อาการปวดมักราว

ทะลุหลัง รวมกับมีอาการคลื่นไส อาเจียน มีไข สาเหตุมักมาจาก การดื่ม alcohol มาก และ

นิ่วในในถุงน้ําดี

การตรวจacute abdomen series อาจพบลักษณะ sentinel loop of small bowel จากการ

มีlocalized ileus หรือ colon cut off sign จากการเกิด spasm ของ trasverse colon

หลักการรักษา- 1. การลดการอักเสบของตับออน โดย NPO เพื่อหลีกเลี่ยงการกระตุน

การหลั่งเอนไซม ใสNG tube ในรายที่ทองอืดมาก - 2. ปองกันโรคแทรกซอน โดย

พิจารณาให antibiotics - 3. supportive treatment โดยเฉพาะตอ circulatory และ

respiratory systems

Indication for surgery

1. Doubtful diagnosis

2. มี gallstones

3. Treatment of complications

3.) ตอบ 4. AFP

INCREASED ALPHA FETOPROTEIN – hepatoma(hepatocellular

carcinoma),testicular tumor(embryonal carcinoma, malignant teratoma), neural tube

defects( in mother’s serum[ spinabifida, anencephaly, myelomeningocele), fetal death,

multiple gestations, ataxiatelangiecsia, some case of benign hepatic disease( alcoholic

cirrhosis,hepatitis,necrosis)

CEA increased : Carcinoma(colon, pancreas, lungs, stomach), smokers, non

neoplastic liver disease, crohn disease, ulcerative colitis

CA 19-9 : GI cancer eg. Pancreas, stomach, liver, colorectal, hepatobiliary, some

case of lungs and prostrate,pancreatitis

Beta- hCG : PRG, some testicular tumor (nonseminomatous germ cell tumors, but

not seminoma), trophoblastic disease (hydatiform mole, choriocarcinoma levels usually

> 100,000 mlU/ml)

ALP increased : biliary obstruction and infiltration liver disease, increased calcium

deposition in bone eg. Childhood, healing fracture

4.) ตอบ 5. Stricture esophagus

Caustic or corrosive ingestion : Alkaline ingestions = liquefactive necrosis

Acid ingestions = coagulation necrosis

1- 2% of caustic ingestions results in stricture formation , Mx : - ABC, airway

protection (กัน aspirate) , หามทําใหอาเจียน, ไมควร lavage, เนน supportive care

7) ตอบ 4. Incarcerated hernia

Rx - กอนการเกิด incarceration แนะนําใหรีบรักษาโดยการผาตัด ไมควรรอนาน

เกินไปอาจเกิด incarceration และ strangulation ได

- ในกรณีที่เกิด incarceration ในเด็กแนะนําใหพยายาม reduce ลําไสหรืออวัยวะที่

ออกมาใหกลับเขาไป ถาดันกลับเขาไปไมไดควรให sedative เสียกอนแลวพยายามดันให

กลับเขาไป ในผูปวยเด็กสวนใหญมักสามารถที่จะดันใหกลับเขาไปไดในที่สุด ถาสําเร็จก็

จะรอทําผาตัดในเวลา 48 ชั่วโมงหรือนานกวานั้นหลังจากดันกลับเขาไปแลว แตถาให

sedative แลวยังไมสามารถดันกอนกลับเขาไปไดก็จําเปนตองทําผาตัดแบบเรงดวน

Page 28: Nle step 2_2009 si115-116 and nle_step_2_2009 nctms editors cut key

SIRIRAJ

SIRIRA

J SIRIR

AJ

8.) ตอบ 4. Acute pancreatitis

จากอาการปวดทองบริเวณลิ้นป แนนมากขึ้นเวลานอน มีปวดราวไปหลัง อาเจียนเปน

อาหาร แสดงถึง Cardinal sign ของ acute pancreatitis อีกทั้ง ผูปวย เคยเปน Cholecystitis

โดยที่ไมไดรับการรักษา ซึ่งเปนไปไดวาผูปวยอาจเคยมี

ภาวะ Gall stone ซึ่งก็ถือวาเปนสาเหตุสําคัญทีทําใหเกิด acute pancreatitisได

1. acute cholecystitis มักจะมีอาการปวดทองบริเวณ RUQ หรือไมก็ radiate ไปที่ tip

of right scapular และอาการมักจะเกิดขึ้นกับผูหญิงที่มีอายุนอย ถาเกิดในผูหญิงอายุมาก

มักจะไมมีอาการ

2. acute appendicitis มักจะปวดทองที่ RLQ หรือไมก็ RUQ พบนอยมากนะที่จะปวด

ที่ลิ้นป อีกอยางคือถาเปนที่ไสติ่งจริงคนไขไมนาจะทนไดมาจนถึงวันที่ 4 นะ นาจะมา

ตั้งแตวันแรกๆ

3. peptic perforation จะมีอาการ severe sharp abdominal pain นอนขดเปนกุงอยูนิ่งๆ

และจะทนไมไดในวันแรกๆเชนกัน นอกจากนี้ vital sign ก็มักจะไม stable (แนวๆ shock)

4. Small bowel obstruction คนไขจะมีทองอืด abdominal distention และลักษณะ

อาเจียนอาจมีน้ําดีปน

9.) ตอบ 1. Hyperstimulation uterus

ในเพศหญิงจะมี breast bud ซึ่งเกิดในชวง 8 – 12 ป และเปนลักษณะสําคัญของ sexual

maturity rating stage 2 ในเพศหญิง อาจมีอาการเจ็บตึงได หามทําการผาตัดเอา mass ออก

โดยเด็ดขาด เพราะจะทําใหเด็กหญิงจะขาด breast tissue เมื่อโตขึ้นได breast 2 ขางไม

เทากันได โดย menstruation จะเกิดขึ้นหลังจากนั้นประมาณ 2 - 2½ ป (9 – 16 ป) ในชวงที่

มี peak height velocity ซึ่งภายในมีการเปลี่ยนแปลง เชน การขยายขนาดของ ovaries,

uterus, labia, และ clitoris, และ มีการหนาตัวขึ้นของ endometrium และ vaginal mucosa

11.) ตอบ 4. F/U 6 mo

ผูปวยอายุนอย Mass เปน cystic consistency นึกถึง benign มากที่สุด นัด F/U ได ถา

กอนโตขึ้น, เจ็บ ก็ aspirate ได

Breast mass

Age Signs and symptoms Diagnosis management

< 30 yr Firm กอนชัด ขางเดียว

ประวัตินาน เปนป

Fibroadenoma Elective excision

30-50 yr Cyst เจ็บ โตยุบตามรอบ

เดือน

Fibrocystic

change

Follow up

Aspirate

Advice recur

>50 yr Risk CA: ไมมีลูก ลูก

นอย อวน กินยาคุม

menopause ชา มีประวัติ

ครอบครัว

CA breast Local: -mastectomy,BCT

-Sentinel, axillary

dissection

-reconstruction

-radiation

Systemic:-

chemo/hormome

12.) ตอบ 3. 70% alcohol

การแกพิษแมงกะพรุนเบื้องตน

เมื่อ ขึ้นจากน้ําแลว หากมีหนวดของแมงกะพรุนขาดติดอยูใหคีบเอาออกให

หมดกอน หามไปขัดถู เพราะจะทําใหเข็มพิษที่อยูที่หนวดปลอยออกมามากขึ้น การดูแล

รักษาเพื่อลดอาการพิษ ของแมงกะพรุนนั้นมีหลายวิธี เชน มีการแนะนําใหแชหรือลางดวย

น้ําสมสายชูอยางออน ประมาณ 15-30 นาที แตหากไมมีใหลางดวยน้ําทะเล หรือ

แอลกอฮอลลางแผล หามลางดวยน้ําจืดเพราะจะทําใหถุงพิษของเข็มพิษ (Nematocyst)

แตกออกมามากขึ้น และหามขัดถูบริเวณที่โดนแมงกะพรุน หลังจากนั้นใชน้ําแข็งหรือ

น้ําอุนประคบ อีก วิธีที่แนะนํา คือการใชครีมโกนหนวดทาลงบริเวณที่โดนแมงกะพรุน

แลวใชมีดโกนโกนออกหรือใชบัตรพลาสติก เชน บัตรเครดิต ขูดออกเพื่อกําจัดเอาเข็มพิษ

ที่ติดอยูออกไป แลวจึงใชน้ําสมสายชูอยางออนหรือแอลกอฮอลลางตอไป

Page 29: Nle step 2_2009 si115-116 and nle_step_2_2009 nctms editors cut key

SIRIRAJ

SIRIRA

J SIRIR

AJ

13.) ตอบ 2. trachea shift

Indication ในการใส ICD ในภาวะ emergency

1. Pneumothorax - pneumothorax ขนาดใหญ

- pneumothorax ในผูปวยที่ clinical ไม stable

- tension pneumothorax ที่ทํา needle decompression แลว

- recurrent หรือ persistent pneumothorax

- iatrogenic pneumothorax ขนาดใหญ

2.Hemotharax

3.Esophageal rupture รวมกับมี gastric leak ใน pleural space

จากโจทยนึกถึงวาผูปวยนาจะมีภาวะ Massive hemotharax ซึ่งก็เปน indication ในการ

ใส ICD อยูแลว แต sign ที่สําคัญที่คิดวานาจะเปน indication ในการใส ICD มากที่สุด ก็

นาจะตอบ trachea shift เพราะ เปน sign ที่บอกวามีลม หรือเลือด อยูในปอดปริมาณมาก

จนทําใหเกิดการ shift ของ mediastinum ควรจะ release ออกโดยเร็ว

14.) ตอบ 5. ประเมินความพรอมและสภาพจิตใจของผูปวย

ตามหลัก 6 stages of change : pre-comtemplation>contemplation>determination>

action>maintainance>relapse ดังนั้นกอนอื่นควรตองประเมินความพรอมของผูปวยกอน

เปนอยางแรก หากผูปวยไมทราบหรือลังเล ตองใหขอมูล ใหบอกขอดีเสีย จนผูปวย

ตัดสินใจเลิกเอง (ไมบังคับนะ) แลวคอยเสนอวิธีหยุดให สวนในโจทยอะ naltrexone เปน

pure opioid antagonist ที่ไป block brain pleasure ที่เกิดจากการดื่มเหลา (ก็เลยลดความ

อยากลงได) มักใชใน relapse prevention

15.) ตอบ 2. Steroid

Mx Scorpion sting - Supportive treatment , Cold pack ประคบ , ใหยาแกปวด ,

± antihistamine รวมกับ oral steroid ถาแพและบวมมาก

16.) ตอบ 1. ใช clamp แหวกแผล แลว explore (ขอนี้คิดวานาจะเปน zone II)

Neck แบงเปน 3 zones

- ZONE III : above angle of mandible

- ZONE II : between cricoid cartilage and angle of mandible

- ZONE I : below cricoid cartilage

Evaluation

Blunt : ไมตอง explore

Penetration : local explore wound กอน

- depth of wound

- local anesthesia at ER

ถา ลึกกวา platysma ใหไปทํา neck explore

Neck exploration/surgical exploration

- treatment general anesthesia in OR

17.) ตอบ 2. explor. lap

ผูปวยคนนี้นาจะมี sign of peritonitis แลว รวมกับมีประวัติ blunt injury และ vital sign ดี

การเลือก investigation อื่นๆมีขอหามดังตอไปนี้

1.DPL มี indication ดังนี้

- มี blunt injury และมีอาการที่ไมชัดเจนพอที่จะระบุไดวามีการบาดเจ็บของ

อวัยวะในชองทอง

- Unconscious or spinal injury ที่ทําใหอาการและอาการแสดงหนาทองเปลี่ยนไป

- ไมสามารถติดตามอาการได เชน ตองไดรับการดมยานานเพื่อรักษาการบาดเจ็บ

ตําแหนงอื่น

- Shock ที่หาสาเหตุของการเสยีเลือดในบริเวณอื่นไมได

Absolute C/I: มีอาการและอาการแสดงของการบาดเจ็บภายในชองทองชัดเจน เชน

peritonitis , มีความจําเปนตองผาตัดชองทองอยูแลว , Pregnancy or previous abdominal

surgery

Neck wound 

Page 30: Nle step 2_2009 si115-116 and nle_step_2_2009 nctms editors cut key

SIRIRAJ

SIRIRA

J SIRIR

AJ

2. Observe สําหรับการบาดเจ็บจาก blunt injury จะปฏิบัติไดก็ตอเมื่อ Vital sign stable

, มี CT scan ประเมินการบาดเจ็บในชองทอง , ไมมีอาการหรืออาการแสดงของการ

บาดเจ็บของอวัยวะในชองทอง เชน ทองอืด กดเจ็บ guarding rigidity or rebound

tenderness , สามารถ observe อยางใกลชิดได และพรอมที่จะผาตัดชองทองไดทันทีถามี

ความจําเปน

18.) ตอบ 3. Hypovolumic shock

ผูปวยมี blunt thoracic injury หลังจากนั้น BP drop คิดวาผูปวยนาจะมี massive blood

loss จนเกิด hypovolemic shock สวน hemopericardium, tension PTX อาการและตรวจ

รางกายไมเขา เพราะทั้ง 2 ภาวะนี้ ควรมี engorged neck vein

19.) ตอบ 1. Urgency endoscope removal

ถาแบตเตอรี่นั้นอยูที่ esophagus ใหเอาออกเลยเพราะเสี่ยงที่จะเกิด voltage burn and

corrosive effect เนื่องจากผนังหนาหลังของesophagus จะประกบกันเขาขั้วโดยสมบูรณ

ถาอยูเลย esophagus ลงไปในกระเพาะแลว paper สวนใหญ(ที่updateหนอย)จะบอกวารอ

ได และให F/U film ถา 2-3 วันไมออกก็คอยเอาออก เนื่องจากเขาเชื่อวา มีแค case report

เทานั้นที่จะเกิด systemic absorption และถาเกิด burn ก็ไมเปนปญหามากเพราะ กระเพาะมี

พื้นที่มาก แตในทางปฏิบัติเพื่อความปลอดภัยตอปญหาฟองรอง ก็แนะนําใหเอาออกเลย

ดีกวา เพราะถาเกิด peritonitis แลวจะไมคุม

ขอนี้ โจทยใหมาวาอยูตรง EG junction แตไมวาอยูตรงไหนก็คิดวาใหเอาออกเลยดีกวา

20.) ตอบ 2. Needle thoracocentesis

จากโจทย คิดถึง tension pneumothorax หรือ อาจเปน hemothorax มั้ง เนื่องจาก

decrease breathsound ขวา กับ trachea shift ซาย ตองตรวจรางกายใหละเอียดกอนอีกที จึง

จะแยกได แลวก็มี shock bp drop

การรักษา massive hemothorax 1.IV resuscitate or blood transfuse 2.ICD

การรักษา tension pneumothorax 1.Needle thoracocentesis ICS 2,3 MCL 2.ICD

21.) ตอบ 1. revise ICD

22.) ตอบ 3. Peak flow

งู “จง เหา สาม คลา เปน neurotoxin” ตอง monitor respiration โดยเปา peak flow

23.) ตอบ 4. Plain abdomen X-ray

เนื่องจากเด็กมีอาการอาเจียนเร็ว อายุ 3 วันหลังคลอด และลักษณะของอาเจียนเปน

bilious vomiting ทําใหนึกถึง duodenal atresia มากที่สุด การวินิจฉัย ดูจาก Plain abdomen

X-ray พบ double bubble sign

24.) ตอบ 3. D 0,3

เพราะเคยไดรับการฉีด Rabies vaccine แลว หากยังไมเคยฉีด ตองฉีด d 0,3,7,14,28

การฉีด d 0,3,7 เปนการฉีดกอนถูกกัด ใชในกลุมเสี่ยง เชน บุรุษไปรษณีย

25.) ตอบ 1. Diaphragmatic hernia

ที่เลือกคําตอบนี้ เราดูจาก ภาวะที่ทําใหCyanosis มีแค 2 อันคือ Diaphragmatic

hernia and tetralogy of falot จุดสําคัญของขอนี้คือ Gurgling มันเปนเสียงของการ

เคลื่อนไหวของของเหลววะ 555 แลวที่สําคัญ heart ก็ปกติดี ก็เลยตัดสินใจตอบขอนี้ งั้น

เรามาอธิบายตัวโรคนี้งาย ๆ กันนะ (เราวาโจทย นาจะบอกมากกวานี้ ไมงั้นกอคงตอบขอ

นี้ไมไดเหมือนกัน)

ตําแหนงที่พบไดบอยที่สุด คือ บริเวณ posterolateral ที่เรียกวา Bochdalek’s hernia

ขางซาย มันทําใหทารกแรกคลอดมีการหายใจลําบาก ทําใหปอดขางที่โดนพวกอวัยวะใน

ชองทองแทรกจะไมเจริญเติบโต โดยอาการจะเปน progressive dyspnea, cyanosis ,

apparent dextrocardia ซึ่งเกิดจากหัวใจถูกกระเพาะและลําไสดันไปขางขวา อาการอาจ

เกิดขึ้นหลังแรกคลอดประมาณ 2-3 วัน โดยจะเห็นวาหนาทองแฟบ ทรวงอกพองโตกวา

ปกติ พอฟงทรวงอกจะไดยินเสียงหัวใจทางดานขวา

26.) ตอบ 4. Chest tube thoracotomy

เนื่องจากจากประวัติ และการตรวจรางกายพบBP drop ประกอบกับการตรวจรางกาย

จึงคิดวาผูปวยเปน tension pneumothorax และเนื่องจากภาวะนี้เปนภาวะ emergency และ

Page 31: Nle step 2_2009 si115-116 and nle_step_2_2009 nctms editors cut key

SIRIRAJ

SIRIRA

J SIRIR

AJ

ผูปวยก็ shock ดวย การทํา ICD จึงไมทันเวลา เราตองทําหลังจาก save life ผูปวยใหได

กอน ซึ่งนั้นก็คือการทํา needle thoracotomy นั้นเอง

27.) ตอบ 1. ตรวจการไดยิน

ขอนี้เราไมแนใจนะ แตจากวิเคราะหทางclinical นาจะตอบ 1.ตรวจการไดยินเนื่องจาก

วัยนี้เปนวัยที่ตองมีการหัดพูด เมื่อมีปญหาการไดยินก็อาจเกิดปญหาในอนาคตได

ในขอนี้เราคิดวาเด็กในโจทยนาจะเปนโรค Fronto-ethmoidal encephalocele ซึ่งเปน

โรคที่พบบอยในแถบ south east asia ผูปวยมักจะเปนMR ดวย

จุดประสงคในการรักษา คือ cosmetic,prevent infection, เพื่อรักษา function จากการที่

กอนกดตาและจมูก

หลักการรักษา คือ การผาตัดปดซอม defect ที่เกิดขึ้น ในกรณีที่มีruptured

encephalocele จาํเปนตองผาตัดภายใน 24 – 48 ชั่วโมง เพื่อปองกันการติดเชื้อ ถากอน

ใหญยื่นรบกวนการมองเห็น ควรรักษากอนอายุประมาณ 6 เดือน กอนมี binocular vision

Orthopedics

1.) ตอบ 3. IIIA

Gustilo’s classification of open fractures

Grade Wound Soft-tissue injury Bone injury

I Less than 1 cm long Minimal Simple

II Greater than 1 cm

long

Moderate, some muscle

damage

Moderate comminution

IIIA Usually greater than

1 cm long

Severe deep contusion;

± compartment syndrome

High-energy fracture

patterns; comminuted

but soft-tissue cover

possible

IIIB Usually greater than

10 cm long

Severe loss of soft-tissue

cover

Requires soft-tissue

reconstruction for

cover

IIIC Usually greater than

10 cm long

As IIIB, with need for

vascular repair

Requires soft-tissue

reconstruction for

cover

2.) ตอบ 1. Wrist extension test

ในผูปวยรายนี้คิดถึง Tennis elbow (lateral epicondylitis)

Symptoms - Pain on the outer part of elbow (lateral epicondyle).

- Point tenderness over the lateral epicondyle – a prominent part of the bone on the

outside of the elbow.

- Gripping and movements of the wrist hurt, especially wrist extension and lifting

movements.

- Activities that use the muscles that extend the wrist (e.g. pouring a pitcher or

gallon of milk, lifting with the palm down) are characteristically painful.

- Morning stiffness.

Physical examination and tests

The diagnosis is made by clinical signs and symptoms : point of tenderness over the

origin of the extensor carpi radialis brevis muscle from the lateral epicondyle (ECRB

origin). There should also be pain with passive wrist flexion and also with resisted wrist

extension (Cozen's test), both tested with the elbow extended.[7]

- Yergason’s test ใชตรวจ Biceps tendon pathology (Bicipital tendonitis)

3.) ตอบ 4. steroid injection

Carpal tunnel syndrome : most common nerve entrapment, awaken night pain

below elbow ปวดชานิ้วโปง ชี้ กลาง ถาเปนนานๆ thenar muscle จะลีบ

Associated medical disease ที่พบรวม Median Trap, Myxedema, Edema, DM, Idiopathic,

Acromegaly, Neoplasm, Trauma, Rheumatoid ,Amyloidosis, Pregnancy

Page 32: Nle step 2_2009 si115-116 and nle_step_2_2009 nctms editors cut key

SIRIRAJ

SIRIRA

J SIRIR

AJ

Test: phalen’s test 60 sec-pain,numbness, Tinel’s sign เคาะที่ volar side of wrist

joint จะเสียวไปตาม nerve distribution , EMG and NCV help to confirm the diagnosis of

CTS(r/o c spondylosis)

Rx:Conservative treatment :if onset < 1mo,no thenar muscle atrophy : มักเริ่มดวย

กินNSAIDS PO ถาไมไดผลคอยใช Steroid injection, สําคํญคือ Rest ,immobilization

splintดามขอมือเวลานอน

Surgical decompression of flexor retinaculum: if อาการนานกวา 1 mo, thenar

muscle atrophy

มันสัมพันธกับเบาหวานดวยก็อาจตองสง FBS แตมันไมใชการรักษาอาการที่นําผูปวย

มา เลยคิดวานาจะตอบ steroid injection ไปเลย

4.) ตอบ 2. External fixator

นาจะเปน Gustilo’s IIIB - IIIC

5.) ตอบ 2. Short wave diathermy + stretch test

อาการปวดสะโพกราวลงขาซาย อาจคิดถึง Herniated nucleus pulposus, spinal

stenosis, lumbar muscular strain, Spondylolysis หรือ spondylolisthesis แตจากการตรวจ

รางกายพบวา SLRT negative ไมมีความผิดปกติของระบบประสาทสวนปลาย (sciatica)

จึงไมคิดถึง Herniated nucleus pulposus, spinal stenosis นอกจากนี้ผลรังสีพบวา loss of

lordotic curve จึงทําใหนึกถึง lumbar muscular strain มากที่สุด เพราะ Spondylolysis หรือ

spondylolisthesis จะทําใหมี exaggerated lordosis ดังนั้นการรักษาที่ดีที่สุดคือตามขอ 2.

สวน Trigger point injection มักทําในคนที่เปน fibromyalgia

สวน NSAIDs ก็ยังไมมีความจําเปนเพราะคนไขไมไดมีภาวะกลามเนื้ออักเสบ

6.) ตอบ 2. AVN

AVN (avascular necrosis) เกิดจากการขาดเลือดไปเลี้ยง bone ทําใหเกิด cell ตาย

สาเหตุที่พบบอยไดแก 1. Traumatic interruption of arteries : fracture, dislocation , 2.

Hemoglobinopathies: sickle cell anemia , 3.Exogenous or endogenous hypercorticsolism

: steroid medication, Cushing’s disease , 4. Alcoholism

ตําแหนงที่พบบอยไดแก Femoral head (most common), humeral head, scaphoid and

talus , อาการของผูปวยมีไดตั้งแต ไมมีอาการในระยะแรกๆ, อาการปวดจะเริ่มมีเล็กนอย

และจะคอยๆปวดมากขึ้นเรื่อยๆ, อาการปวดจะมากขึ้นเมื่อใชงานหรือตองรับน้ําหนัก

ภาพรังสี ในระยะแรกๆจะไมมีลักษณะผิดปกติใดๆ ตอมาอาจพบมี sclerosis ถาเปน

มากขึ้นอาจพบ bone deformities เชน flattening, subchondral radiolucent lines (crescent

sign), การยุบตัวลงของกระดูก

ในขณะที่ OA hip จะมีอาการปวดสะโพก, morning stiffness, limit ROM

7.) ตอบ 5. ประเมินสุขภาพกอนพิจารณาการรักษา

- Intertrochanteric fracture: เปน extracapsular fracture, PE พบ great trochanter

tenderness, พบ ecchymosis,ขาสั้น และ external rotate ชัดเจนกวา fracture neck of femur

เพราะไมมี capsule รั้งไว

Rx - หากสภาพรางกายแข็งแรง ไมมีขอหามในการผาตัด นิยมรักษา โดยการผาตัด ทํา

internal fixation เชน dynamic hip screw, intramedullary nail เพื่อใหผูปวยสามารถนั่งหรือ

ยืนไดในชวงสัปดาหแรกหลังการผาตัด

- conservative treatment โดยการทํา skeletal traction 6 สัปดาห

I/C 1. มีอัตราเสี่ยงมากในการผาตัด

2. ยืนเดินไมไดตั้งแตกอนกระดูกหักและมีอาการเจ็บปวดที่รอยหักไมมาก

3. ผูปวยอยูในระยะสุดทายของโรครายแรง

- ตองระวัง venous thrombosis, pressure sore, pneumonia, UTI

8.) ตอบ 1. Spondylosis

Most common cause of low back pain คือ overuse ทําใหเกิด muscle strain, ligament

sprain ซึ่งmuscle ที่ spasm จะทําให loss of lordotic curve แตไมมีใน choice ที่เจอบอยรอง

มาก็นาจะเปน พวก degenerative change

Page 33: Nle step 2_2009 si115-116 and nle_step_2_2009 nctms editors cut key

SIRIRAJ

SIRIRA

J SIRIR

AJ

9.) ตอบ 4. Forearm pronation

ผูปวยมีอาการปวดแขนและขอมือ ตรวจรางกายพบ Abductor pollicis brevis m. grade

IV (Recurrent branch of Median n.) & Loss sensation ที่นิ้วโปงชี้ กลาง (Median

n.) = ไมนาใช AIN syndrome

Tinel test พบวา negative ที่บริเวณขอมือ แสดงไมนาใช Carpal Tunnel Syndrome

ก. Cozen’s test ใชตรวจ Tennis elbow (ไมเขากับ clinical)

ข. Vermitt test ??? (หาไมเจอจริงๆ...โทษที)

ค. Cervical traction ตรวจ lesion ที่ nerve root (นาจะมีอาการปวดขอ มีอาการ

Radiculopathy, myelopathy รวมดวย)

ง. Forearm pronation ตรวจการทํางานของ Pronator teres เพื่อ R/O Pronator

teres syndrome (Rare condition)

10.) ตอบ 4. Axillary crutches

เนื่องจากเปนเครื่องค้ํายันที่สามารถรับน้ําหนักหรือ share weight ไดมากที่สุดถึง 80 %

ของน้ําหนักตัว และมีจุดที่สัมผัสรางกาย 2 จุด จึงชวยในการทรงตัวไดดี เหมาะกับผูที่

พิการ ที่สําคัญ คือ เวลาใชตองสัมผัสดานขางของทรวงอกต่ํากวารักแรเล็กนอย อยาใหกด

รักแรเพราะจะทําใหกด radial nerve เกิดเปน crutch palsy ได

สาเหตุที่ไมเลือกขออื่นเนื่องจากผูปวยรายนี้เกิดขาซายขาด ไมไดเปนเพียงขาซายเจ็บ

หรือออนแรง ทําใหเสียแรงพยุงลําตัวของขาขางที่ขาดไป อุปกรณที่ควรแนะนําจึงนาจะ

เปนอุปกรณที่ share weight ไดมาก และทรงตัวไดดี

11. ) ตอบ 3. Back strengthening exercise

จากโจทยผูปวยมีอาการเดินแลวปวดฝาเทา ที่จริงตอนแรกก็คิดวาอาจจะเปนพวก

Plantar fasciitis แตก็ไมtypical เพราะโรคนี้มักจะปวดฝาเทาตอนเดินกาวแรกหลังตื่นนอน

หรือหลังจากนั่งพักนานๆ แตพออานโจทยตอ ก็บอกวานั่งพักแลวดีขึ้น และ SLRT neg ซึ่ง

ก็ทําใหนึกถึงพวก Low back pain ที่จะมีปวดราวลงขาได แตก็แปลกๆที่วาปวดแตฝาเทา

อยางเดียว ไมปวดหลัง และไมบอกวาราวลงมาดวย แตถาใชจริงๆ จากประวัติก็คิดถึง

Spinal stenosis มากที่สุด เพราะเปนประวัติที่เขากับ Intermittent claudication และ SLRT

neg ทําใหคิดถึง Herniated disc นอยลง ซึ่งการexerciseที่นาจะชวยในโรคนี้ ก็มีทั้งการ

บริหารหลังและทอง แตคิดวานาจะเปนการบริหารหลังมากกวานะ

ENT & EYE

1. ) ตอบ 3. Miotic drug

จากประวัติเขาไดกับภาวะ acute angle closure glaucoma การรักษาดวยยาแบงเปน

1. CAI : เชน Acetazolamide : หามใชในผูปวยที่แพยา sulfa

2. Beta-blocker : เชน Timolol : ระวังการใชในผูปวย hypotension, asthma (เพราะไป

block ทั้ง beta 1,2)

3. Miotic : เชน Pilocarpine : หลังใชจะตามัวระยะนึง,ระวังอยาไปขับรถหลังหยอดยา

4. Alpha- adrenergic : หลังใชอาจมี allergic conjunctivitis ได

5. Prostaglandin : เชน Latanoprost : ระวังการใชในผูปวย keratitis

: จากประวัติผูปวยเปน asthma และแพ sulfa จึงไมใช CAI , Beta-blocker

: ไมใช atropine เพราะจะยิ่งขยายไปปด angle มากไปอีก

: โดยสรุปแลวจึงใช Miotic นั่นเอง

4.) ตอบ 2. pressure patch

จากโจทยขอนี้บอกชัดเจนวาผูปวยคนนี้ มีภาวะ corneal abrasion ที่ตาขวา ซึ่งภาวะ

corneal abrasion ก็คือการที่ cornea มีinjury ที่ยังจํากัดอยูในชั้น epithelium โดยสาเหตุสวน

ใหญมาจาก อุบัติเหตุ foreign body มาทิ่มตา ที่พบบอย ก็คือ เล็บมือ และ คอนแทคเลนสที่

ใสนานเกิน(แหง) อาการก็จะมี รูสึกมีอะไรในตา(แตเอาไมออก), เคือง, น้ําตาไหล, ตาแดง

การวินิจฉัย ก็ไดแกการยอม fluorescine ที่ตา แลวเอาไปสอง blue light lamp สวนการ

รักษา ก็คือ ปาย ATB + pressure patch 24hr ยกเวนมีสงสัยการติดเชื้อ ก็ไมตอง pressure

patch ใหรักษาโดยการหยอด ATB บอยๆ

Page 34: Nle step 2_2009 si115-116 and nle_step_2_2009 nctms editors cut key

SIRIRAJ

SIRIRA

J SIRIR

AJ

5.) ตอบ 2. refraction

-ภาวะ physiologic myopia เปนสาเหตุของ visual loss ที่พบไดบอย เกิดจากการที่

ขนาดของ eye globeเจริญ ไมสัมพันธกับ lens ทําใหเกิด refractive error มักเริ่มเปนตอนที่

เด็กเขาโรงเรียน และจะหยุดเมื่ออายุ 20 ป โดยจะ < -5.00 (แยกจาก pathologic myopia)

ดังนั้นจึงควรประเมิน refractive error โดยตรวจ VA

- ไมแนใจวาขอ1. หมายถึง orthoptist (ชางวัดสายตา) หรือไม แตถาพูดถึง orthoptics

จะใชตรวจเกี่ยวกับ EOM disorder เชน strabismus

6.) ตอบ 2. Glaucoma

จาก Blunt eye injury เปนเหตุใหผูปวยมี Hyphema (คือการที่มี blood in the anterior

chamber) ซึ่งตอมาในระยะยาวจะทําใหเกิดเปน traumatic glaucoma ขึ้นได จากการที่มี

old red blood cells ที่มีรูปรางผิดปกติไปเนื่องจากสูญเสีย hemoglobin และจะไปรวมตัว

เปน clot จนอุด trabecular meshwork เปนเหตุใหไมสามารถ drain aqueous humor ออกไป

ได ทําใหมีการเพิ่มขึ้นของ intraocular pressure จนกลายเปน Glaucoma ในที่สุด

Complication ของ hyphema คือ corneal blood staining, secondary hemorrhage,

uncontrolled intraocular hypertension, prolonged duration of anterior chamber clot,

vitreous hemorrhage, retinal detachment, ocular rupture

7.) ตอบ 3. amblyopia

คือภาวะการมองเห็นลดลงโดยไมมีความผิดปกติทางกายภาพหรือสมองสวนการ

มองเห็น เกิดจากตาขางนั้นไมไดรับการกระตุนอยางเหมาะสม ในชวงdevelopment of

visual function (8-10ปแรก)

8.) ตอบ 5. incision & curettage

รายนี้ hordeolum เปน abscess แลว เนื่องจากมี pus formation จําเปนตอง I&C

9.) ตอบ 3. Laser Photocoagulation

เนื่องจากประวัติผูปวยเปน PDR และมี vitreous hemorrhage ดังนั้นการรักษาคือทํา

PRP เพื่อปองกัน neovascularization ไมใหมากขึ้น

10.) ตอบ 2. ENT exam

เมื่อพบกอนที่คอ ควรทํา ENT exam เพื่อหา primary lesion อาจทํารวมกับFNA กอน

ที่คอได แตไมควรทํา LN biopsy แบบexcision incision เพราะหากเปนcervical

lymphadenopathy จากmetastatic CA จะทําใหstagingเปลี่ยนได

11.) ตอบ 2. Corneal ulcer

เนื่องจากผูปวยมีประวัติ trauma (ถูกไสดินสอเขาตา) มีอาการตามัว ตาแดง และมี

ciliary injection แตมี normal pupil จึงทําใหนึกถึง acute glaucoma (fixed dilated pupil)

และ uveitis (constricted pupil)ลดลง สวน dull corneal reflex นี้ไมแนใจวาหมายถึง

irregular light reflex หรือวา corneal reflex ที่เปนของ CN V ถาเปน irregular light reflex

ก็จะนึกถึง corneal abrasion มากขึ้น แตนาจะมีอาการระคายเคือง เจ็บตา น้ําตาไหล แพแสง

มากรวมดวย แตผูปวยรายนี้มีอาการปวดตา ตามัว ตาแดง จึงเขาไดกับ corneal ulcer

มากกวา

12.) ตอบ 5. Excisional biopsy

โรคที่นาจะนึกถึงไดในผูปวยรายนี้คือ Chalazion ซึ่งเปนการอักเสบเรื้อรังของตอม

meibomian โดยเกิดจากการอุดตันของตอมนี้ ลักษณะที่พบจะเปนกอนที่หนังตาบนหรือ

หนังตาลาง กอนจะโตขึ้นชาๆไมเจ็บ ไมมีการอักเสบ ถาใหญขึ้นอาจจะกดลูกตาทําให

ความโคงของกระจกตาผิดไป เกิดภาวะสายตาเอียงได ในบางรายอาจจะมีการติดเชื้อรวม

ดวย โดยจะมีอาการแบบเดียวกับ internal hordeolum

การรักษา ถาขนาดเล็กและไมมีอาการไมตองรักษาก็ได ถาโตขึ้นหรือมีการอักเสบติด

เชื้อ ใหการรักษาแบบเดียวกับ internal hordeolum โดยการเจาะหนอง รวมกับการใหATB

อาจฉีด steroid เขาที่กอนในรายที่ไมมีการอักเสบแลว แตอาจทําใหสีของผิวหนังจางลงได

ในผูปวยรายนี้กลับเปนซ้ําตรงตําแหนงเดิมบอยๆ ควรตัดชิ้นเนื้อตรวจเพราะอาจเปนมะเร็ง

ของตอม meibomianได

Page 35: Nle step 2_2009 si115-116 and nle_step_2_2009 nctms editors cut key

SIRIRAJ

SIRIRA

J SIRIR

AJ

13.) ตอบ malignant otitis externa

ในขอนี้จะเห็นวามีประวัติในเรื่องของ เบาหวานที่คุมไดไมดี ซึ่งถือไดวาอยูในภาวะที่

เปน immunocompromise host และก็ตรวจพบมี granulation tissue ที่บริเวณหูชั้นนอก

เปนลักษณะที่เขาไดกับโรคนี้มากที่สุด เนื่องจากมักพบในผูปวยสูงอายุที่มีภูมิตานทานต่ํา

โดยเฉพาะในโรคเบาหวาน โรคนี้เปนการอักเสบของหูชั้นนอกและกระดูกตรง skull base

ที่เกิดจากเชื้อ P. aeruginosa ไดบอยที่สุด นอกจากจะพบวามี granulation tissue ที่หู

ชั้นนอกแลว จะมีอาการปวดหูมาก(severe otalgia) และอาจพบมีอาการหูอื้อ/หูตึงไดถามี

การอักเสบบวมอุดกั้นในชองหู และอาจมี cranial nerve palsy ไดดวยจากการอักเสบติด

เชื้อกระจายไปตาม skull base (ที่ใชคําวา malignant ไมไดหมายถึงเปนมะเร็ง แตเปนโรคที่

มีอัตราตายคอนขางสูง อาจเรียกโรคนี้อีกอยางวา”skull base osteomyelitis” )

นอกจากนี้จะตองแยกโรควาเปน tumor หรือไม เนื่องจากก็สามารถพบ granulation

tissue ไดเชนกัน แต tumor นั้นมักจะมีอาการปวดมากโดยไมสัมพันธกับรอยโรค ซึ่งมะเร็ง

ที่มักจะพบมักเปน squamous cell carcinoma สวนใหญ ซึ่งมักจะพบ lesion บริเวณ

posterosuperior ของใบหู/concha ดวย และอาจตรวจพบมีตอมน้ําเหลืองบริเวณหนาหูโต

ดวย

สวน otitis externa นั้นตางจาก malignant otitis externa โดยจะพบมีอาการปวดหู

โดยเฉพาะเมื่อถูกกดที่ tragus และขยับใบหู และมีอาการคัน บวมแดงของผิวหนังที่บุชองหู

ได อาจจะพบลักษณะของ exudate หรือฝหนองได แตจะไมพบลักษณะของ granulation

tissue ในชองหู และโรคนี้มักจะเจอในผูที่มีประวัติแคะหู ปนหู มีความชื้นหรือน้ําเขาหู

บอยๆ ดวย ทําใหมีการเปลี่ยนแปลงสภาพในชองหู เกิดการอักเสบของผิวหนังที่บุชองหูได

14.) ตอบ 1. Acoustic trauma

อาการ มีการสูญเสียการไดยิน มี2 แบบ คือ เปนชั่วคราวและถาวร

1. เปนชั่วคราว (TTS : Temporary Threshold Shift) มักมีอาการอื่นรวมดวย เชน เสียง

อื้อในหู ทนเสียงดังๆไมได หู2ขางไดยินเสียงไมเทากัน การหายจาก TTS ขึ้นกับระดับ

เสียงที่ไดรับ อาจใชเวลานานเปนนาทีถึงชั่วโมง

2. เปนถาวร (PTS : Permanent Threshold Shift) มีการเสียหายที่ cochlea ดวย มี2 แบบ

1) acoustic trauma เกิดจากไดยินเสียงที่ดังมากเพียงครั้งเดียวในชวงเวลาสั้นๆ ทํา

ใหสูญเสียการไดยินทันที มักมีอาการเจ็บหูดวย

2) NIHL (Noise-induced hearing loss) เกิดจากไดยินเสียงดังปานกลางเปน

เวลานาน (>85dB >8hr/d) ทําใหสูญเสียการไดยินแบบคอยเปนคอยไปในชวง 5-10 ป

การสูญเสียการไดยินมักเสียที่ความถี่ 3,000-8,000 Hz กอน แลวเสียที่ 2,000 Hz และ

จะเสียมากที่ 4,000 Hz มักเปนเทาๆกัน 2 ขาง แตก็อาจไมเทากันได เชน ยิงปน

แหลงกําเนิดเสียงอยูหางหู 2 ขางไมเทากัน

การตรวจเพื่อ early detection ใหทํา OAE (OtoAcoustic Emission) วัดสัญญาณที่สง

มาจาก outer hair cell ของ cochlea แม audiogram จะปกติก็ตาม

สวน acoustic neuritis เปนการอักเสบของ cochlear division ของ CN 8 เชื่อวาเกิดจาก

viral, vascular cause อาการมักคอนขางเร็ว(sudden) หรือรูตัวตอนตื่นนอน อาจเสียการได

ยินที่ความถี่ต่ําหรือสูงก็ได อาจมีอาการtinnitus, vertigo รวมดวย สามารถหายไดเองภายใน

14 วัน

16.) ตอบ ?? (ตามชีท ENT อ สมุทร,si 115 survivor’s guide)

นึกถึง Bell’s palsy or Ramsay Hunt syndrome(โจทยไมครบ??!! ไมรูมี vesicleมั้ย )

- Bell’s palsy ตอบ 1 prednisolone ละกัน :disease of exclusion ,r/o โรคอื่น

ออกไปแลว ex Malignant otitis externa,ramsay hunt สพ กับ viral infection(HSV1)

- Acute paresis เปนมา < 2 wk : ให Prednisolone 1 MKD for 14 day ,F/U day 5

if paresis then f/u 1 mo

- Other med: vit b complex ,articificial tear,eye oint ดวย

- Acute paresis เปนมา เกิน 2 wk ให observe อยางเดียวไมตองใหยา

• Acute paralysis เปนมา < 2 wk : ทํา EnoG

• if <90% degen ให Prednisolone 1 MKD for 14 day

• if >90% degen : surgical decompression เลย

Page 36: Nle step 2_2009 si115-116 and nle_step_2_2009 nctms editors cut key

SIRIRAJ

SIRIRA

J SIRIR

AJ

• Acute paralysis เปนมา เกิน 2 wk ทํา EMG f/u 5 mo

Good prognosis,almost recovery in 1 mo.แต complete paralysis prog แยกวา paresis

• If Ramsay’s hunt

• Tx:1 acyclovir 800mg po x 5 for 7-10 day

• 2 prednisolone 10 mg po bid for 7-10 day

• Pain control :analgesia

• For post herpetic neuralgia :amitriptyline 10 mg po hs

18.) ตอบ 3. bullous myringitis

Hx : มี URI นํามากอน จากนั้นตื้อๆ หนักในหู ปวดหูรุนแรง

PE : herpes like bleb ที่ Tympanic membrane, TM สีแดงเขมหรือคอนขางมวง

เชื้อมักเปนพวก virus เชน influenza อาจเปน M. pneumonia

รักษา: -symptomatic treatment : pain control

-ถาปวดมากใหเจาะ bleb + ยาแกปวด ear drop

-ถาสงสัยเชื้อ M. pneumonia ให erythromycin or tetracyclin

19.) ตอบ 1. Benign positional vertigo

ขอ 4, 5 ตัดออกกอนเพราะเปน neuro อาการนาจะรุนแรง นามีแขนขาออนแรง แต

โจทยไมครบคือเวลาที่เปนอะ แลวก็ไมรูมีอาการอื่นรวมดวยปาว เชน tinnitus hearing loss

ถาไมมีก็ตัดขอ 3 ออก แลวก็ไมมีไข ไมมี URI นํา เลยตัดขอ 2 ออก นาจะตอบขอ 1 มาก

ที่สุดนะเพราะอาการสัมพันธกับทาทาง

20.) ตอบ 1. Vocal cord paralysis

Vocal cord paralysis มักเกิดตามหลังผาตัดตอมไทรอยด!!

- Unilat – low-pitch dysphonia, aspiration

- Bilat – อาจnormal ไปจน biphasic stridor , aspiration

Subglottic edema ไมนาใช นาจะเหนื่อยเยอะ

Arytenoid dislocation เกิดตามหลังการใส ET tube พบไมบอย อาการเหมือน Vocal

cord paralysis ตองแยกกันใหได เนื่องจาก ภาวะนี้ ควร early detect and reduction แลวก

หาย แต Vocal cord paralysis กทําแค observe กับ voice therapy

Intubation granuloma – hoarseness , globus , + dyspnea เกิดจากchronic irritation

และไมนาจาเกิดทันทีหลังผาตัด ไมนาตอบขอนี้

Vocal cord edema – ไมคอยเกิดจาก ET tube อะ เกิดจาก Anaphylaxis , Quinsy

/ Peritonsil abscess , Retropharyngeal abscess , Serum sickness ดูchoiceมันไมคอย

specificอะ ไมนาตอบขอนี้เหมือนกัน

21.) ตอบ 3. decongestant

ผูปวยมีหูขวาไดยินลดลง + test เปน right conductive hearing loss ตรวจหู พบ

ลักษณะของ blood in the middle ear cavity และ negative pressure in the middle ear

retracts the ear drum and fills the middle ear with fluid นึกถึง middle ear barotrauma ซึ่ง

แบงไดเปน 4 type คือ

Type I--congestion of umbra , Type II--entire drum red , Type III--hemorrhage and

fluid , TypeIV-- perforation ;

Treatment with systemic decongestants (Sudafed, 60 mg orally three times a day).

Topical nasal decongestants should be tried, (no longer than three days to avoid rhinitis

medicamentosa).

Antibiotics are used only when there is fluid in the middle ear and there is the

possibility of infection. If perforation is present, oral antibiotics definitely should be

used. Eardrops are definitely contraindicated if there is perforation of the eardrum.

22.) ตอบ 2. ใชเครื่องปองกันเสียงดังขณะทํางาน

เพราะโจทยถามวาดูแลทุกราย ก็คือคนอื่นยังไมเปน ก็เลยตอง prevention

Page 37: Nle step 2_2009 si115-116 and nle_step_2_2009 nctms editors cut key

SIRIRAJ

SIRIRA

J SIRIR

AJ

23.) ตอบ 2. subglottic stenosis

Approximately 90% of all cases of acquired subglottic stenosis in children and

adults result from endotracheal intubation. Duration of intubation is the most important

factor in the development of stenosis. Patients with acquired stenosis are diagnosed from

a few days to 10 years or more following initial injury. The majority of cases are

diagnosed within a year. Symptoms include the following :

Dyspnea (may be on exertion or with rest, depending on severity of stenosis) ,

Stridor , Hoarseness , Brassy cough , Recurrent pneumonitis , Cyanosis

intubation granuloma : caused by trauma during intratracheal intubation for general

anesthesia. S&S: change in voice or speech, distinct hoarseness, difficulties in breathing,

or even dyspnea แตยังไมมี stridor

unilateral vocal cord paralysis อาการสําคัญที่ผูปวยมักมา คือ เสียงแหบ

Reinke edema สวนใหญผูปวยจะมีประวัติ เปนผูหญิงวัยกลางคน สูบบุหรี่ low-

pitched voice ; Reinke’s edema, also called ‘polypoid corditis,’ is a swelling of the

entire layer of the superficial lamina propria. It occurs exclusively in smokers. Reinke’s

edema causes a very characteristic gravelly, low-pitched voice.

foreign body ผูปวยนาจะมีประวัติแบบ acute onset

24.) ตอบ 2. Laryngomalacia

Laryngomalacia เปน congenital malformation of larynx ที่พบมากที่สุด เกิดจาก

โครงสราง supraglottis ออนตัว ไมสามารถคงรูปอยูไดขณะหายใจเขา ทําใหมีการยุบตัว

ของฝาปดกลองเสียงเขาไปในทางเดินหายใจ เด็กมักมี inspiratory stridor ตั้งแตอายุ 2-3

สัปดาห ดีขึ้นเมื่อนอนคว่ํา เลวลงเมื่อนอนหงาย,มักไมเขียว,เสียงรองปกติ ควรแนะนําให

เด็กนอนทาตะแคง อาการมักดีขึ้นเองตอนอายุ 2 ป

►Congenital /w stridor

DDx -Laryngomalacia most common

-Vocal cord paralysis สวนใหญผิดปกติขางเดียว มักพบในทารกคลอดยาก, กิน

แลวสําลัก, stridor ชัดตอนรองไห นอนหลับจะเบาลง, มักหายเองตอน 6-9 เดือน

-subglottic hemagioma congenital tumor, อาจมีcyanosis dyspnea, 50% มักพบ

รวมกับ skin lesion , มักหายเองใน 1-2 ป ถาไมหายรักษาโดยผาตัดหรือฉายรังสี

►Reinke’s edema การบวมของ vocal cord ที่ superficial layer of lamina propria

มักเกิดจาก chronic irritation พบในคนที่สูบบุหรี่จัดมากๆ อาการมักมาดวยเสียงแหบ ถา

เปนมากอาจมีการอุดกั้นทางเดินหายใจได

25.) ตอบ 1. Oral Amoxycullin + Clavulanic acid

Mx of acute otitis media :

1. Medical : 1st line : Amoxycillin แลว F/O 48-72 hr ถายังไมดีขึ้นหรือแยลง

เปลี่ยนให 2nd line แตถาดีขึ้นใหตอจนครบ 7-10 วัน

2nd line : Amoy-clavulanic acid, Cefotaxime, Ceftriaxone

2. Surgery : ทํา tympanocentesis

I/C : 1. อาการไมดีขึ้นหลังได ATB ที่เหมาะสมแลว (ก็คือได 2nd line ไปแลว)48-72hr

2. ซึมลงหรือมีภาวะแทรกซอน

3. immonocompromised host

Psychi

1.) ตอบ 2. Delirium tremens

เริ่มจากผูปวยมีประวัติหยุดกินเหลามา 2 วัน แลวมีอาการดังกลาว อาการเปนอยาง

รวดเร็ว ไมไดใหประวัติวามีโรคประจําตัวใดๆ

- ที่ตัดไดขอแรก นาจะเปน dementia เพราะ acute ไปและโจทยไมไดกลาวอางใดๆ

เกี่ยวกับการเสียทางดาน memory ซึ่งเปนขอเดนขอ dementia ถาไมมีไมครบ criteria

diagnosis (ตองเสียความจํา + 1ขอของ aphasia or apraxia or agnosia or executive function

+ กระทบชีวิตประจําวัน)

Page 38: Nle step 2_2009 si115-116 and nle_step_2_2009 nctms editors cut key

SIRIRAJ

SIRIRA

J SIRIR

AJ

- Alcohol abuse or dependence แมวาผูปวยจะดื่มมากจนหยุดแลวมีอาการ แตโจทย

ไมไดใหรายละเอียดการดื่มของผูปวย ซึ่งถาไมมีก็ วินิจฉัยภาวะขางตนไมไดเชนกัน

Abuse คือการใชสารในทางที่ผิด สวน dependence คือการติดสาร 2 อันนี้ แยกกันไดโดย

dependence ตองมี 1.การดื้อยา เชน ตองการเสพมากขึ้น หรือ เสพเทาเดิมผลของยาลดลง

2. withdrawal symptom 3. อยากเลิกเสพแตเลิกไมได รูวาไมดีแตก็ใชเวลาหมดไปกับการ

หา เสพ และถอนยา

- ขอที่นาจะถูกตองมากที่สุด คือ Delirium tremens ผูปวยจะมีอาการเหมือน Delirium

คือ cloudy of consciousness , confusion (hallucination ที่เปนมักเปน visual) ,attention

deficit, fluctuation ,Deliriumมักเกิดจาก organic causes Delirium tremens มักมีประวัติ

ดื่มสุรา แลวมีการหยุดดื่ม และที่สําคัญคือ จะมี autonomic symptom รวมดวย เชน ความ

ดันสูง ชีพจรเร็ว มือสั่น มีไข อาจถึงชักได อาการเปนไดใน 48 ชม.หลังหยุดดื่ม แตมักเปน

มากวันที่ 4-5 ที่ตองแยกกันเพราะ Delirium tremens รักษาดวย benzodiazepine ที่มี half

life ยาว เชน diazepam แต ใช haloperidol ซึ่งทําให threshold การชักต่ําลง

2.) ตอบ 3. Haloperidol

จากโจทย มีประวัติการผาตัดสําไล เปน major operation หลังจากผาตัด มีอาการ

ผิดปกติ มี visual hallucination and perceptual disturbance จึงนึกถึง delirium มากที่สุด

A. มีความผิดปกติไปของระดับความรูสึกตัว

B. มีความเปลี่ยนแปลงใน cognition หรือมีความผิดปกติในการรับรู perceptual

disturbance

C. ความผิดปกติในชวงสั้นๆ และ มักจะ fluctuation ระหวางวัน

D. มีโรคทางกาย -> การรักษา Haloperidol

3.) ตอบ 2. Delirium tremens

เริ่มจากผูปวยมีประวัติหยุดกินเหลามา 2 วัน แลวมีอาการดังกลาว อาการเปนอยาง

รวดเร็ว - ขอที่นาจะถูกตองมากที่สุด คือ Delirium tremens ผูปวยจะมีอาการเหมือน

Delirium คือ cloudy of consciousness , confusion (hallucination ที่เปนมักเปน visual)

,attention deficit, fluctuation ,Deliriumมักเกิดจาก organic causes Delirium tremens มัก

มีประวัติดื่มสุรา แลวมีการหยุดดื่ม และที่สําคัญคือ จะมี autonomic symptom รวมดวย เชน

ความดันสูง ชีพจรเร็ว มือสั่น มีไข อาจถึงชักได อาการเปนไดใน 48 ชม.หลังหยุดดื่ม แต

มักเปนมากวันที่ 4-5 ที่ตองแยกกันเพราะ Delirium tremens รักษาดวย benzodiazepine ที่

มี half life ยาว เชน diazepam แต ใช haloperidol ซึ่งทําให threshold การชักต่ําลง

- เพิ่มเติม บางคนอาจคิดวาถาตัวเลือกมี alcohol withdrawal ละ alcohol withdrawal

ก็ถูกนะแตกวางไป มันเปน spectrum ละ Delirium tremens ก็รวมอยูในนั้นดวย

- Wernicke encephalopathy ตองมี 3 อยางคือ 1)oculomotor disturbance 2)ataxia

3)confusion

4.) ตอบ 1. Hyperventilation

อธิบาย อาการมือจีบ เกิดจาก hypocalcemia ทําให cell ตางๆ มี excitability เพิ่มขึ้น

(เพราะ Ca++ ชวย stabilize พวก voltage-gated ion channel ถา Ca++ ลด voltage-gated

ion จะเปดไดงายขึ้น)ที่ neuromuscular junction ก็จะมี excitability เพิ่มเชนกัน กลามเนื้อที่

จะถูก affect กอน คือ กลามเนื้อขนาดเล็ก เพราะมีปริมาตรนอย depolarization ไมตองมาก

ก็ตอบสนองไดแลว ดังนั้นกลามเนื้อเล็กๆ จึงเปนกอน เชน กลามเนื้อปลายมือปลายเทา

โดยเฉพาะ extensor digitorum ทั้งหลาย เลยเกิดเปนมือจีบ แตกลามเนื้อเล็กที่ถา contract

แลวจะตายไดคือ laryngeal muscle ถาเกิด laryngeal spasm คนไขจะ asphyxia ตาย นี่คือ

สาเหตุที่วาทําไมตองรีบ treat คนไขพวกนี้ สําหรับ hyperventilation ทําใหเกิด hypoCa

เนื่องจากเกิด respi. alkalosis ทําให โปรตีนตางๆ ในเลือดแตกตัวให H+ มากขึ้น Ca จึงไป

จับกับโปรตีนแทน ทําให free Ca2+ ลดลง

5.) ตอบ 1. Delirium

ตาม hierarchy principle ผูปวยรายนี้มีอาการเกิดขึ้นหลังจากไดรับอุบัติเหตุที่ศรีษะ ทํา

ใหคิดถึงภาวะ organic mental disorder กอนโรคอื่น

Page 39: Nle step 2_2009 si115-116 and nle_step_2_2009 nctms editors cut key

SIRIRAJ

SIRIRA

J SIRIR

AJ

6.) ตอบ 2. IV ceftazidine

คิดถึงภาวะ agranulocytosis ซึ่งเปนผลมาจาก clozapine การรักษาคือ specific

antibiotic therapy โดยในกรณีผูที่มีไขมาตั้งแต3-5วันคิดถึง กลุมhigh risk ซึ่งผูปวยมีไข

มากกวา 39 C ในกลุมlow riskอาจใหcipro+amoxi-clav ครอบคลุม pseudomonas

7.) ตอบ 2. Delirium Tremens

รายนี้ มีภาวะ alcohol withdrawal มีอาการเอะอะโวยวาย คิดถึงภาวะ delirium tremens

อาการจะเดนเรื่อง sympathetic hyperactivity ยังไมใช Wernicke encephalopathy เนื่องจาก

ภาวะนี้จะมีลักษณะเขาไดกับ CAN คือ Confusion, CN6palsy, Ataxia ,Nystagmus

8.) ตอบ 2. Wernicke encephalopathy

มีอาการและอาการแสดงครบ CAN triad ไดแก Confusion Ataxia และ Nystagmus

9.) ตอบ 3. Delirium tremens

เนื่องจากในผูปวยรายนี้มีอาการconfusion, visual hallucination, illusion, agitation,

tachycardia,hypertension เกิดขึ้นหลังจากที่หยุดกินเหลาเปนเวลา 2 วัน จึงคิดถึงภาวะ

delirium tremens มากที่สุด

Delirium tremens มักจะเกิดในชวง 48-72 ชั่วโมงหลังหยุดหรือลดแอลกอฮอล อาการ

จะเปนมากวันที่ 4-5 หลังหยุดดื่ม โดยมีอาการสูงสุดอยูไดนานประมาณ 72 ชั่วโมง จากนั้น

คอยๆลดลงใน 5-10 วัน โดยผูปวยจะมีอาการ disorientation, confusion, hallucination,

tremor, insomnia, fever, tachycardia, hypertension, agitation ได ผูปวยมักจะมี medical

conditions อื่นๆรวมดวย เชน liver failure, pneumonia, GI bleeding, electrolyte imbalance

เปนตน

Note! ขอสอบอาจไมใหประวัติหยุดหรือลดแอลกอฮอลมาก็ได ใหพิจารณาจาก sign

of ANS hyperactivity

การรักษา 1. ใหยาในกลุม Benzodiazepines เชน Diazepam 10 mg IV( เนนวา IV)

กรณีที่มี liver impairment มากหรือผูสูงอายุ พิจารณาให lorazepam แทน

Note! การรักษาตางจาก delirium ที่ให haloperidol ใน delirium tremens หามให

haloperidol เนื่องจากวาจะทําใหผูปวยชักได)

2. ใหยาเพื่อปองกันการเกิด Wernicke-Korsakoff syndrome ดังนี้ 1. Thiamine 100

mg IV or IM วันละครั้งนาน 3 วัน 2. Thiamine 100 mg กินวันละ 3 ครั้ง 3. Folic acid 1

mg กินวันละครั้ง

10.) ตอบ 3. Diazepam

จากประวัติและอาการของผูปวย เขาไดกับ Delirium tremens ซึ่งเปนอาการของ

Alcohol withdrawal โดยจะมีอาการ disorientation, ประสาทหลอนเหมือนจริง, นอนไม

หลับ, confusion, มีไข จะเปนมากในวันที่ 4-5 หลังหยุดเหลา รักษาโดยการใหยาในกลุม

Benzodiazepine (Diazepam, Chlordiazepoxide)

**Delirium tremens ไมใช Delirium หามใหการรักษาดวยยา Haloperidol**

11.) ตอบ 2. Autistic

มีพฤติกรรมที่อยูในโลกของตนเอง ไมสนใจบุคคลอื่น ไมมองสบตา ไมแสดงอารมณ

โตตอบหรือสรางความสัมพันธกับผูอื่น สนใจวัตถุมากกวาบุคคล สนใจเฉพาะบางสวน

หรือบางดานของวัตถุ มักมาพบแพทยดวยพัฒนาการดานภาษาลาชา มี 3 ดานหลักๆคือ

   1. Impair social interaction   2. Language  &  3. Repetitive behavior 

12.) ตอบ 1. Normal grief

เหตุผล ผูปวยมีอาการซึมเศรา กินไมได นอนไมหลับ เรียนไมได ซึ่งเกิดหลังจากแม

เสียชีวิต และอาการยังเปนไมเกิน 2 เดือน ถือวา เปน Normal reaction

- MDD: ผูปวยจะตองมีอาการคงอยูตอหลังจากพนระยะเวลาของ normal reaction ไป

แลว หรือมีความคิดอยากตาย โดยการวินิจฉัยตองมีอาการ 5 ขอจาก 9 ขอ (โดยมีขอ 1 หรือ

ขอ 2 อยู 1 ขอ) เปนเวลา 2 สัปดาห – 1.ซึมเศรา 2. ความสนใจในสิ่งตางๆลดลงอยางมาก

3. เบื่ออาหาร หรือน้ําหนักลดมากกวา 5% ใน 1 เดือน 4. นอนไมหลับ หรือนอนมากกวา

ปกติ 5. psychomotor agitation or retardation 6. ออนเพลีย ไมมีเรี่ยวแรง 7. รูสกึตนเอง

Page 40: Nle step 2_2009 si115-116 and nle_step_2_2009 nctms editors cut key

SIRIRAJ

SIRIRA

J SIRIR

AJ

ไรคา หรือรูสึกผิด 8. สมาธิลดลง ลังเลใจ 9. คิดอยากตาย รวมทั้งไมเคยมีประวัติของ

mania or hypomania

- Anxiety disorder: จะมีอาการเดนดานวิตกกังวล ตางจาก depression ที่เดนเรื่อง

อารมณเบื่อหนาย

- Adjustment disorder: จะวินิจฉัยเมื่อผูปวยไมเขาเกณฑการวินิจฉัยภาวะอื่น

- Dysthymia: อาการคลาย MDD แตรุนแรงนอยกวา โดนเกณฑการวินิจฉัยจะตองมี

อาการซึมเศรานานกวา 2 ป (ในเด็กและวัยรุนอาจมีอารมณหงุดหงิดนานอยางนอย 1 ป)

โดยตองไมมีชวงที่หายนานเกินกวา 2 เดือน รวมกับมีอาการตอไปนี้ 2 อาการขึ้นไป คือ 1.

เบื่ออาหาร หรือกินจุ 2.นอนไมหลับ หรือนอนหลับมาก 3.ออนเพลีย ไมมีแรง 4.low self

esteem 5.สมาธิไมดี หรือตัดสินใจลําบาก 6.รูสึกทอแท

13.) ตอบ 3. sleep hygiene

จากโจทยจะเห็นวาผูปวยเปน primary insomnia คือมีอาการนอนไมหลับมานานกวา 1

เดือน โดยไมมีสาเหตุชัดเจน โดยอาการครั้งแรกเกิดขึ้นในชวงที่มีความเครียดอยางหนึ่ง

(ในผูปวยรายนี้คือการยายงาน) แลวทําใหผูปวยมี over concern ยิ่งกังวล ยิ่งเครียด ทําใหยิ่ง

นอนไมหลับ กลายเปนวงจรของการนอนไมหลับ โดยอาการมักเกิดขึ้นในชวงวัย 20 – 40

ป และเปนติดตอกันไดนานเปนสิบป ยิ่งถาใหประวัติวาผูปวยมักออนเพลีย ไมมีสมาธิ

หรือหลับไดงายถาไมไดตั้งใจจะนอนหลับจะยิ่งชวยสนับสนุน

หลักการรักษา ขั้นแรกตองแยกโรคอื่นๆไปกอน เชนพวก anxiety, mood d/o, drug ซึ่ง

โจทยขอนี้ไมไดใหประวัติมา และการรักษาตองเริ่มดวยการปรับพฤติกรรมการนอน

(sleep hygiene)เปนหลัก รวมกับการรักษาดวยยา

สําหรับการใชยานอนหลับใหใชในระยะสั้น(ไมมากกวา 2 สัปดาห) ถาตองใชเปน

เวลานานตองติดตามอาการผูปวย ยาที่นิยมจะเปน benzodiazepines(ที่นิยมคือ flurazepam,

temazepam, triazolam, midazolam, flunitrazepam) กับ zolpidem(imidazopyridine

สําหรับ sleep hygiene ก็ทั่วๆไป เชนพวกไมดื่มกาแฟหลังเที่ยง ไมนอนกลางวัน ออก

กําลังกาย ไมกินอาหารหนักๆกอนนอน ไมคิดอะไรเครียดๆกอนเขานอน

ในการศึกษาผลของการใชยา และการรักษาโดยไมใชยา พบวาทั้งสองวิธี สามารถลด

อาการนอนไมหลับได พอ ๆ กัน ถึงแมวาการใชยาจะไดผลรวดเร็วกวา แตการไมใชยา จะ

ไดผลนานกวา ; มีการศึกษาเปรียบเทียบ ระหวางการใช triazolam กับการใชวิธี stimulus

control instruction รวมกับ relaxation training พบวา triazolam ไดผลทันที สวนการไมใช

ยา ไดผลเมื่อ 3 สัปดาหตอมา อยางไรก็ตาม เมื่อดูผลที่ 1 เดือนตอมา การไมใชยา มี

ประสิทธิภาพมากกวา triazolam ในการคงผลการรักษาใหดีเหมือนเดิม

14.) ตอบ 4. ไมตองทําอะไรเปนอาการปกติ

ผูปวยเปน Hypnagogic hallucination โดยเปนอาการประสาทหลอนที่เกิดขึ้นขณะจะ

เคลิ้มหลับ อาจเปน visual,auditoryหรือtactile hallucination ก็ได พบไดในคนปกติ ไม

ตองทําอะไร

15.) ตอบ 4. crisis intervention

จากประวัติของผูปวยเขาไดกับภาวะ Major depressive disorder (MDD) ซึ่งในผูปวย

รายนี้มารพ.ดวยเรื่องกินยานอนหลับเกินขนาด ซึ่งอาจเกิดจาก suicidal attempt หรือสาเหตุ

อื่นก็ได ซึ่งเราตองทําการคนหาสาเหตุตอไปในภายหลัง แมวาอันที่จริงแลว MDD ควร

จะตองใหการรักษาโดยใหยา Antidepressants ซึ่งในขอนี้ก็มีchoice คือ Fluoxetine (ยาใน

กลุม SSRIs) ซึ่งเปน first choice ในการรักษา MDD ในปจจุบัน แตเมื่อพิจารณา setting ใน

โจทยขอนี้ เปนภาวะ Emergency คิดวาคําตอบที่นาจะเหมาะสมกวาคือ crisis intervention

Crisis คือ ภาวะชั่วคราวที่เกิดขึ้นเมื่อบุคคลรับรูวามีสิ่งคุกคามตอตัวเขา ภาพพจนหรือ

เปาหมายในชีวิตของเขา ความสูญเสียหรือความเปลี่ยนแปลงทําใหบุคคลนั้นเกิดการ

เปลี่ยนแปลงโดยที่วิธีการแกปญหาในลักษณะเดิมๆไมเพียงพอที่จะลดความตึงเครียดนั้น

ทําใหเขาตองเผชิญกับปญหาซึ่งเขาแกไขไมได จนทําใหบุคคลนั้นเสียความสมดุลไป

Crisis intervention เปน psychotherapy แบบหนึ่งที่มีเปาหมายอยูที่การแกไข

immediate crisis โดยจะชวยปรับcoping ของผูปวยและทําใหผูปวยกลับเขาสูสมดุลเพื่อให

มี function ไดดีกวาหรือเทากับชวงกอนมีcrisis

Page 41: Nle step 2_2009 si115-116 and nle_step_2_2009 nctms editors cut key

SIRIRAJ

SIRIRA

J SIRIR

AJ

Forensic

2.) ตอบ 5. ไมมีความพิการ

chart นี้ อานเทียบบัญญัติไตรยางศกับ snellen chart เลย OD6/60 OS6/6

นิยามความพิการทางการเห็น “จะใชสายตาขางดีที่สุดเมื่อใชแวนตาธรรมดา”

โดยถาเปน 1. Visual impairment (สายตาเลือนราง) 6/18 ถึง 3/60

2. blindness (ตาบอด) นอยกวา 3/60

3.) ตอบ 5. เก็บเลือดผูปวย + คราบเลือดรอบแผล + ผมรอบแผล + ผมหรือขนตามตัว +

เนื้อที่ซอกเล็บ

เพราะเสนผมกับเนื้อที่ซอกเล็บ ที่พบในสถานที่เกิดเหตุอาจไดจากมือผูตายที่เกิดการ

ตอสูกันกอนตาย เสนผมถามีรากผมก็ตรวจ DNAได ถาไมมีรากผมก็ตรวจไดแต blood

group สวนเลือดผูตายก็เอาไวตรวจสอบผูตายได

4.) ตอบ 1. ผานการรวมเพศมา

เนื่องจากเมื่อดูจากสิ่งที่โจทยใหมาไมสามารถระบุลงไปไดวาผูปวยยินยอมหรือไมจึง

ไมเลือกขอ4 สวนขออื่นๆมีคําวา “กระทําชําเรา” ซึ่งเปนคําในกฎหมายจึงไมเลือก

5.) ตอบ 1. blast injury

การบาดเจ็บจากระเบิดจะแบงเปน

- Primary blast injury (PBI) เปนผลโดยตรงจากการเปลี่ยนแปลงแรงกดของอากาศซึ่ง

เกิดจากคลื่นระเบิด (Blast wave)

- Secondary blast injury เกิดจากวัตถุที่ปลิวจากแรงระเบิดหรือสะเก็ดระเบิดปะทะ

รางกายทําใหเกิด blunt หรือ Penetration injury

- Tertiary blast injury เกิดจากรางกายผูบาดเจ็บกระเด็นหรือปลิวจากแรงระเบิดไป

ปะทะกับวัตถุอื่นๆ

นอกจากนี้อาจมีการบาดเจ็บอื่นๆ จากการระเบิดไดแก การสําลักฝุน ควัน หรือสารเคมี

แผลพองไหมจากกาซรอนจากระเบิดและ Crush injury จากวัตถุหรือสิ่งกอสรางลมทับ

ซึ่งจากโจทย เปนmultiple injury จึงคิดวา นาจะเปน blast injury

Miscellaneous

1.) ตอบ 1. smoking กับ CA lung ไมมีความสําคัญทางสถิติ

เนื่องจากคา p value > 0.05

2.) ตอบ 1. Lateral view

Conventional sinus X-rays consist of three views:

1. Waters view -- used to visualize maxillary sinuses

2. Caldwell view -- used to visualize frontal and ethmoid sinuses

3. Lateral view --- used to visualize sphenoid sinuses

3.) ตอบ 1. Lt. Mento-posterior

ขอนี้ใชเรื่องของ Fetal Position ซึ่งเราจะบอกอางอิงสวนนําของทารกเทียบกับชอง

คลอดของมารดา โดยขึ้นกับpresentationของทารกดวย ซึ่งในขอนี้จากโจทยนาจะหมายถึง

ทารกที่เปน Face presentation ซึ่งสวนนําหรือ Denominator จะเปนสวนคาง เมื่อคางอยู

ดานหลังซายของแมจึงเรียกวาเปน Lt. Mento-posterior นั่นเอง

4.) ตอบ 2. AST/ALT

- การวินิจฉัย Paracetamol toxicity อาศัยการตรวจระดับยาในเลือด โดยแปลผลดวย

Matthew-Rumack Normogram โดยมีการมีพิจารณารวมกับคา AT/ALT ดวย

: AST and ALT levels are normal (< 50 IU/L) and the acetaminophen level is < 10

μg/mL, significant hepatotoxicity is very unlikely.

: If AST and ALT levels are normal but the acetaminophen level is ≥ 10 μg/mL,

significant hepatotoxicity is possible; AST and ALT levels are remeasured after 24 h. If

repeat AST and ALT levels are normal, significant hepatotoxicity is unlikely; if the levels

are high, significant hepatotoxicity is assumed.

Page 42: Nle step 2_2009 si115-116 and nle_step_2_2009 nctms editors cut key

SIRIRAJ

SIRIRA

J SIRIR

AJ

: If initial AST and ALT levels are high, regardless of the acetaminophen level,

significant hepatotoxicity is assumed.

5.) คําตอบไมแนใจอะ พิจารณาจากขอมูลที่หามาใหนะ

Prescription drugs

• Central acting agents:

o Sedative hypnotics (for example, benzodiazepines).

o Anticonvulsants (for example, barbiturates).

o Antiparkinsonian agents (for example, benztropine, trihexyphenidyl).

• Analgesics:

o Narcotics (NB. meperidine*).

o Non-steroidal anti-inflammatory drugs*.

• Antihistamines (first generation—for example, hydroxyzine).

• Gastrointestinal agents:

o Antispasmodics.

o H2-blockers*.

• Antinauseants:

o Scopolamine.

o Dimenhydrinate.

• Antibiotics:

o Fluoroquinolones*.

• Psychotropic medications:

o Tricyclic antidepressants.

o Lithium*.

• Cardiac medications:

o Antiarrhythmics.

o Digitalis*.

o Antihypertensives (β-blockers, methyldopa)

• Miscellaneous:

o Skeletal muscle relaxants., Steroids.

Page 43: Nle step 2_2009 si115-116 and nle_step_2_2009 nctms editors cut key

SIRIRAJ

SIRIRA

J SIRIR

AJ

ชุดที่ 2

Medicine

1.) ตอบ A. Naloxone

ผูปวยมีอาการที่เขาไดกับภาวะ opioid toxicity กลาวคือมีภาวะ decreased

consciousness, respiratory depression, constricted pupil(miosis) ซึ่ง naloxone เปนยาที่

ออกฤทธิ์ตาน opioid

ภาวะ hypoglycemia ผูปวยจะมีอาการออนเพลีย วิงเวียน หนามืด ตาลาย ใจหวิว ใจ

สั่น มือสั่น เหงื่อออก รูสึกหิว บางคนอาจมีอาการปวดศีรษะ ซึม กระสับกระสาย พูดออแอ

แขนขาออนแรง ปากชา มือชา พูดเพอ เอะอะ โวยวาย กาวราว ลืมตัว หรือทําอะไรแปลก ๆ

(คลายคนเมาเหลา) ถาเปนรุนแรง อาจมีอาการชักหมดสติ ในรายที่เกิดจากการดื่มเหลา

ผูปวยอาจมีอาการตัวเย็นชืด แขนขาเกร็ง ขากรรไกรแข็ง สิ่งตรวจพบ คือ เหงื่อออก มือเทา

เย็น อาจมีอาการชักหรือหมดสติ ชีพจรมักเบาเร็ว และความดันเลือดต่ํา (แตก็อาจพบวา

ปกติก็ได) รูมานตามักจะมีขนาดปกต ิและหดลงเมื่อถูกแสง

สวน atropine ใชรักษาภาวะพิษจาก organophosphate ซึ่งจะมีอาการ SLUDGE

(Salivation, Lacrimation, Urination, Diaphoresis, Gastrointestinal motility, Emesis) หรือ

DUMBBELSS (Diarrhea, Urination, Miosis, Bradycardia, Bronchoconstriction,

Excitation (as of muscle in the form of fasciculations and CNS), Lacrimation, Salivation,

and Sweating (only sympathetic innervation using Muscarinic receptors)) ซึ่งฤทธิ์ของ

atropine เปน competitive antagonist ตอ muscarinic acetylcholine receptor

5.) ตอบ A. N.menigitides

ซึ่งมักพบในเด็กและyoung adult และพบpetechiae or purpura รวมดวยไดเปน

ลักษณะสําคัญ

6.) ตอบ จากโจทยและจาก choice ที่มีตัวเลือกเดียวจึงคิดวา ตอบ DHF

7.) ตอบ A. Strongyloides

เนื่องจากผูปวยมีประวัติ on steroid มาเปนระยะเวลานาน

Acute infection อาจพบรอยแดงเปนทางคดเคี้ยวที่ผิวหนัง หรือพบ urticarial rash มี

อาการไอ หรือปอดอักเสบ

Chronic infection อาจมีอาการปวดทอง คลื่นไส อาเจียน แนนทอง ถายเหลว

ออนเพลีย น้ําหนักลด หรือมีภาวะ malnutrition

การวินิจฉัยอาศัยการตรวจพบ rhabditiform larva ปนอยูในอุจจาระ duodenal fluid

หรือ เสมหะ

8.) ตอบ 3. Chicukunya fever

ผูหญิงรายนี้ไปเที่ยวที่จ.ทางภาคใต มีอาการไข ปวดขอ ผลเลือดดังกลาวจึงคิดถึงโรค

Chicunkunya fever มากที่สุด

9.) ตอบ A. Prednisolone

เนื่องจาก ประวัติของผูปวยรายนี้ ทําใหนึกถึง ITP ซึ่ง เปนโรค autoimmune ตอ Plt ที่

พบไดบอยในผูหญิงชวงอายุ 20-40ป การรักษา ขั้นแรกควรใหเปน corticosteroids ถาการ

รักษาไมไดผลจึงจะพิจารณาให IV immunoglobulin หรือทําsplenectomy

10.) ตอบ E. ET tube

Primary survey- A: airway คนนี้มี inspiratory stridor จึงควรใส endotracheal tube

กอนในการรักษาเบื้องตน

B: breathing ดูการหายใจ

C: circulation ดู pulse,bleeding

D: EVM ประเมิน neuro ดู pupil, concious

E: exposure&environmental ดูบาดแผลทั้งตัว หลัง,perineum

11.) ตอบ A. Prednisolone

ถาเปน Bell’s palsy จริง จะใหการรักษาดวย Prednisolone +/- acyclovir และ

ปองกัน exposure keratitis ; Bell’s palsy จะเปน acute onset , unilateral ผูปวยจะมี

Page 44: Nle step 2_2009 si115-116 and nle_step_2_2009 nctms editors cut key

SIRIRAJ

SIRIRA

J SIRIR

AJ

อาการหลับตาไมสนิท มุมปากตกและขยับใบหนาซีกนั้นไมได ; เปนไดทุกเพศทุกวัย แต

พบไดบอยขึ้นในผูสูงอายุ , 3 rd trimester pregnancy , DM

ประวัติไมนาจะมี vesicular rash ที่หูนะ ไมงั้นจะคิดถึง Ramsey-Hunt syndrome

(Herpes zoster oticus)

12.) ตอบ ??

จากการตรวจรางกายคิดถึง Distal median nerve dysfunction ตามdistribution

Cause : - Dysfunction of one nerve group is called a mononeuropathy. although systemic

disorders may cause isolated nerve damage ex. mononeuritis multiplex

- Distal median nerve dysfunction occurs when the nerve is inflamed, trapped, or

injured by trauma. Carpal Tunnel Syndrome most common

- Inflammation of the tendons ( tendonitis) or joints ( arthritis) can also cause nerve

compression.

-Conditions that affect connective tissue or cause deposits to form in tissue can

block blood flow and lead to nerve compression

- can be remembered using the mnemonic, MEDIAN TRAP for Myxoedema,

Edema, Diabetes mellitus, Idiopathic, Acromegaly, Neoplasm, Trauma, Rheumatoid

arthritis, Amyloidosis and Pregnancy.

Symptoms : Sensation changes in the thumb and first two fingers, such as: Burning

feeling ,Decreased sensation ,Numbness ,Tingling / Weakness of the hand

Ix: - Electromyogram (EMG) : DDx mononeuropathy from polyneuropathy

,radiculopathy ,myopathy

- Nerve conduction tests : confirm nerve dysfunction

การที่Tinel’s sign neg.ไมไดบอกวาคนไขไมไดเปน Carpal Tunnel Syndromeเพราะ

sens+specต่ํา ; ขอ C กับ D ดูเหมือนจะไมใชinvestigationนะ= =’’ สวนขอ A กับ B ดู

เหมือนจะใหหาสาเหตุของ Distal median nerve dysfunctionซึ่งก็ไมไดเขากับสาเหตุของ

Distal median nerve dysfunctionเลย

13.) ตอบ C. Prednisolone

Ramsay Hunt syndrome is defined as an acute peripheral facial neuropathy

associated with erythematous vesicular rash of the skin of the ear canal, auricle (also

termed herpes zoster oticus), and/or mucous membrane of the oropharynx.

Treatment : Corticosteroids and oral acyclovir are commonly used in the treatment

of Ramsay Hunt syndrome.

15.) ตอบ C. thyroid function test

Levels of urine VMA ไดแก Pheochromocytoma, Neuroblastoma, Ganglioma ซึ่ง

นาจะพบบอยกวาในเด็ก

16.) ตอบ B. Bronchial asthma

Bronchiectasis(โรคหลอดลมโปงพอง)

พยาธิสรีรวิทยา : เกิดพยาธิสภาพของทั้ง bronchi และ bronchiole จากภาวะติดเชื้อหรือการ

อักเสบซ้ําๆ ของหลอดลมทําใหเกิดแผลเปนบางสวนหลังการอักเสบ ผนังหลอดลมโปง

พองขึ้น มีการเพิ่มเซลลตอมเมือกผิดปกติในบางบริเวณ มีการงอกของหลอดเลือดแดง

อยางผิดปกติเกิดพยาธิสภาพที่ถาวร เกิดการติดเชื้อ ณ บริเวณนี้ไดบอยๆ และเกิดการ

ทําลายหลอมลมมากขึ้นเปวงจร

สาเหตุ : 1 ) การติดเชื้อ 2 ) โรคทางพันธุกรรมหรือโรคทีเ่ปนแตกําเนิด

3 ) ภาวะภูมิคุมกันบกพรอง 4 ) การทําลายของหลอดลมและปอดหลังการสูดสําลัก

5 ) โรคขออักเสบเรื้อรังและโรคออโตอิมมูน

6 ) อื่น เชน inflammatory bowel disease แตมักไมสัมพันธกับการสูบบุหรี่

ลักษณะทางคลินิก : มักมีอาการไอเรื้อรัง ปริมาณเสมหะมักมีปริมาณมากอาจถึงครึ่งถึง 1

แกวน้ําตอวัน เสมหะมักเหนียวขนหรือเปนเมือก มักมีอาการเหนื่อยหอบเฉพาะเวลาออก

Page 45: Nle step 2_2009 si115-116 and nle_step_2_2009 nctms editors cut key

SIRIRAJ

SIRIRA

J SIRIR

AJ

แรงหรือเหนื่อยเรื้อรังก็ได ตรวจรางกายมักพบ อัตราการหายใจเร็วกวาปกติ ฟงเสียงปอด

มักพบเสียง crackles

Bronchial asthma

พยาธิสรีรวิทยา : 1 ) การอักเสบของหลอมลม 2 ) การหนาตัวของผนังหลอดลม

สาเหตุ : 1 ) ปจจัยทางพันธุกรรม

2 ) ปจจัยทางสิ่งแวดลอม- ภาวะภูมิแพ , การติดเชื้อ , การสูบบุหรี่ , มลภาวะทาง

อากาศ , โภชนาการ

ลักษณะทางคลินิก : มักพบในคนอายุนอยกวา 40 ป หายใจดังเสียงวี้ด แนนหนาอก หอบ

เหนื่อยและไอ มักมีอาการเปนพักๆและอาการเปลี่ยนกลับไปกลับมาอยางรวดเร็ว อาจ

เกิดขึ้นเองหรือจากการกระตุนดวยปจจัยชักนํา เชน อาการเย็น ชวงเวลากลางคืน การออก

กําลังกาย ถาหากเกิดการกระตุนเซลลมาสตที่แทรกตัวอยูที่ผนังหลอมลมแตเดิมกอาจทําให

เกิดการกระตุนกลามเนื้อเรียบใหเกิดการหดตัวอยางรุนแรงเกิดอาการหอบแบบเฉียบพลัน

ได ( asthma exacerbation )

C ) Chronic bronchitis

พยาธิสรีรวิทยา 1 ) ภาวะหลอมลมอุดกั้นและคางในปอด

2 ) ความผิดปกติในการแลกเปลี่ยนกาซ

3 ) ภาวการณหลั่งเมือกมากขึ้น

4 ) ภาวะแรงดันในหลอดเลือดแดงของปอดสูง

สาเหตุ : 1 ) การสูบบุหรี่ ( 80-90 % )

2 ) การสัมผัสฝุนและสารเคมีจากการประกอบอาชีพ ( 20% ) 3 ) มลพิษอื่นๆ

ลักษณะทางคลินิก : มีอาการไอแบบมีเสมหะเปนระยะเวลาอยางนอย 3 เดือนใน 1 ป เปน

เวลา 2 ป ติดตอกัน โดยไมพบสาเหตุอื่น มักพบในผูสูงอายุ ในระยะแรกๆมักจะไมมีอาการ

ใดๆ แตในระยะหลังๆ จะมีอาการเหนื่อย ไอเรื้อรัง และมีเสมหะ อาการเหนื่อยพบไดบอย

และมักจะมีลักษณะคอยเปนคอยไปและเพิ่มขึ้นเรื่อยๆ ยกเวนมีภาวะ acute exacerbation

การตรวจรางกายมักจะพบลักษณะผิดปกติในชวงที่มีการดําเนินโรคมานานแลวโดยอาจ

พบ barrel-shaped chest , คลํา apex ของหัวใจไมได , hyperresonance , wheezing

D ) Pulmonary TB

พยาธิสรีรวิทยา : จากการตอบสนองของรางกายตอเชื้อวัณโรค

สาเหตุ : จากการติดเชื้อวัณโรคซึ่งมักพบในผูที่มีภูมิคุมกันบกพรอง

ลักษณะทางคลินิก : มักพบอาการไอเรื้อรังบอยที่สุดตั้งแตในระยะแรกของโรค อาจไมมี

เสมหะตอมาในระยะหลังเมื่อมีการทําลายเนื้อปอดมากขึ้น อาการไอก็จะเปนแบบมีเสมหะ

มักไมมีอาการหอบเหนื่อย เนื่องจากมีอากาคอยเปนคอยไป และอาจพบอาการทางระบบ

อื่นๆ เชน ไข ออนเพลีย น้ําหนักลด เบื่ออาหาร เปนตน

E ) CA bronchus

สาเหตุ : 1 ) Smoking 2 ) Radon gas 3 ) Asbestos 4 ) Viruses 5 ) Particulate

Matter

ลักษณะทางคลินิก : dyspnea , hemoptysis , chronic coughing or change in regular

coughing pattern , wheezing , chest pain or pain in the abdomen , cachexia , fatigue and

loss of appetite , dysphonia , clubbing , dysphagia

17.) ตอบ C. Mycloplasma pneumoniae

ลักษณะไขไมรุนแรง ไอ หอบ รวมกับ ฟงปอดมี crepitation ดานขวา CXR: RLL

infiltration เขาไดกับ atypical pneumonia (CXR แบบ patchy infiltration บางครั้งไม

สามารถแยกระหวางภาวะ typical pneumonia กับ atypical pneumonia ที่เกิดจาก bacteria

ได) โดย atypical pneumonia เชื้อกอในเด็กวัยรุนถึงผูใหญ คิดถึง Mycoplasma

pneumoniae มากที่สุด สวน Chlamydia pneumoniae จะพบในเด็ก infant มากกวา; สวน

typical pneumonia นั้นจะมีอาการไขสูง รุนแรงมากกวานี้ เชน Streptococcus pneumoniae

พบไดบอยที่สุดในทุกชวงอายุ, Haemophilus influenza พบในเด็กและนาจะมีประวัติขาด

Hib vaccine, Staphylococcus aureus จะพบในเด็ก infant หรือมีประวัติ pneumatoceles

รวมดวย

Page 46: Nle step 2_2009 si115-116 and nle_step_2_2009 nctms editors cut key

SIRIRAJ

SIRIRA

J SIRIR

AJ

18.) ตอบ D. Mycoplasma pneumonia

Bullous myringitis เปนการอักเสบของหูจาก virus มักเกิดรวมกับหวัด หรือ

mycoplasma / S&S: ตื้อๆในหู-severe otalgia โดยเฉพาะเมื่อมีการเคลื่อนไหวของ

tympanic membrane, ตรวจพบ herpetic-like bleb ที่ดานนอกของ tympanic membrane,

tympanic membrane เปนสีคล้ํามวงแดง

Rx: เจาะ bleb, ใหยาแกปวด, ยาหยอดหูที่มียาแกปวดผสม, อาจให erythromycin หรือ

tetracycline

19.) ตอบ C. Tzanck smear

จากlesion คิดถึง bullous pemphigoid จึงคิดวานาจะทําTzanck smear

21.) ตอบ D. ใหยาเดิม แตเจาะเขา (aspirate) หนอง ออกใหมากที่สุด

จากประวัติและการตรวจรางกายของผูปวยรายนี้ เขาไดกับ septic arthritis มากที่สุด

เจาะหนองออกมายอมแลว คิดวาเกิดจากเชื้อ S.aureus ไดยา cloxa ไปแลวไมดีขึ้น

กอนอื่นขอพูดถึง septic arthritis โดยยอๆกอนนะ

septic arthritis = หมายถึงภาวะการอักเสบของขอที่เกิดจากการติดเชื้อ ซึ่งสวนมากมักจะ

หมายถึงเชื้อ bacteria

อาการ : - monoarticular involvement

- ในadult joint ที่เปนมากคือ knee รองมาไดแก hip และ ankle joint ในขณะที่

neonate มักจะเปนมากที่ hip joint

- acute onset ของ joint pain, swelling, warm และ tender ผูปวยจะปวดมากไมวา

จะเปน active หรือ passive movement

เชื้อกอโรค : S.aureus มากสุด, Streptococcus

ในเด็ก < 6 mo. : E.coli , เด็ก 6mo-2yr : H.influ

การรักษา : Ix : H/C, joint aspiration>>>G/S, C/S, cell count, cell diff

Antibiotics : cloxacillin for S.aureus (IVประมาณ 2wk oral จนครบ 6-8wk)

Adequate drainage :**ขอนี้สําคัญอยูแลวในทุกๆ infection

- Arthrotomy เปดขอไปลาง

- Arthroscope สองกลองไปลาง >>> Ortho ชอบทํา

- Needle aspiration เรื่อยๆ ตามสไตล หมอ Med

สรุปก็คือ รายนี้ ที่ไขไมลงเพราะวายังไมมีการทํา drain เลย ยาที่ใหก็นาจะฆาเชื้อไดอยู

แลว จึงขอตอบ ขอ D จา

23.) ตอบ E. Endotracheal intubation

: จากประวัติมีกอนที่คอ มีการโตขึ้นภายใน 3 M (ยัง + benign or CA) จน 1wk PTA

หายใจลําบากและตรวจรางกาย RR 24 และมี inspiratory stridor คดิวากอนนาจะโตจน

เบียดหรือบังทางผานของอากาศตามปกติ ก็คือเกิด airway obstruction (นึกถึง

Upper>lower)

Mx od UAO :

1. Basic : position (head tilt, chin lift), suction for clear airway

: if not improve : oropharyngeal airway or nasopharyngeal airway

2. Advance: ET-tube, Surgical airway (cricothyroidotomy, tracheostomy)

: ในผูปวยรายนี้ประเมินการอุดตันจากกอนไปเบียดเปนหลัก ดังนั้น basic life support

ไมนาเพียงพอ (เพราะกอนก็ยังกดอยูดี) ดังนั้นควร advance โดยเริ่มจากการใช ET-tube

กอนไดเลย

25.) ตอบ B. ATB เดิม แลวทํา drainage ใหมากที่สุด

การระบายหนอง หรือ drainage มีขอบงชี้ในผูปวยเด็กเล็ก, มีการติดเชื้อขอสะโพก, มี

การเจาะดูดหนองไดขนจํานวนมาก แตในผูปวยเด็กโตหรือผูใหญซึ่งมีการติดเชื้อของขอที่

อยูตื้น(ไมใชขอสะโพก) และมีอาการในระยะเริ่มแรก(นอยกวา 3 วัน ) อาจเจาะดูดหนอง

ออกจากขอใหมากที่สุดและใหยาปฏิชีวนะ และใหเจาะดูดซ้ําๆได แตถาอาการไมดีขึ้น

ภายใน 48 ชั่วโมง ก็ควรระบายหนองดวยวิธีผาตัด ในปจจุบันมีการใชกลอง arthroscope

เพื่อชวยระบายหนองจากขอที่อยูตื้น เชน ขอเขา ขอเทา ซึ่งสามารถระบายหนองไดอยางมี

ประสิทธิภาพ และยังสามารถลดการยึดติดของขอไดดี

Page 47: Nle step 2_2009 si115-116 and nle_step_2_2009 nctms editors cut key

SIRIRAJ

SIRIRA

J SIRIR

AJ

26.) ตอบ E. steroid

เพราะวาวาคนนี้เปน atopic dermatitis ซึ่ง เปนโรคผิวหนังอักเสบเรื้อรังชนิดหนึ่ง ซึ่ง

พบบอยในเด็ก เกิดจากปฏิกิริยาภูมิแพในรางกาย ไมใชโรคติดตอ (ผูปวยมักมีอาการแพ

อื่นๆดวยและมักมีประวัติครอบครัว) โดย ถาผิวหนังอักเสบในลักษณะแบบเฉียบพลัน

จะพบเปนvesicle +ozing โดยมีอาการคันมาก

ตําแหนงของโรคจะแตกตางตามวัย กลาวคือ วัยทารก จะพบผื่นแดงบริเวณแกม ดาน

นอกของแขนขา ขอมือและขอเทา / วัยเด็กโต และผูใหญ ผื่นจะพบบริเวณขอพับตางๆ

เชน ขอพับแขนและขาทั้งสองขาง ที่คอ บางครั้งเกาจนเปนปนมีขุยดํา หรือแหงหนา

บางครั้งถาเปนมากผื่นอาจลามขึ้นไดทั่วรางกาย

การรักษา+ปองกัน : - การปองกันความแหงของผิว >>ควรทาโลชั่น ครีมบํารุงผิวชนิดไม

กอผิวแพ หรือ สารเคลือบผิวตางๆ ภายในระยะเวลา 3- 5 นาทีหลังซับผิวหรือเช็ดตัวให

แหง โดยหามเช็ด ขัดหรือถูตัวแรงๆ

- ในกรณีทีมีอาการคันมาก แพทยจะพิจารณาการใหยาตานภูมิแพ หรือยาแกคัน บาง

ชนิดอาจมีผลขางเคียงทําใหงวงได, – การใชยาทาเฉพาะที่เพื่อทาผื่น มีการใชยาหลายชนิด

เชนการใชยาสเตียรอยดครีม ซึ่งจะใหผลการรักษาที่ทําใหผื่นยุบไดเร็วไดผลดีมาก แตอาจ

กลับเปนซ้ําขึ้นไดเร็วเชนกัน

27.) ตอบ C. prednisolone

อาการของผูปวยรายนี้เขาไดกับภาวะ Acute nephritic syndrome เพราะมี

hypertension, hematuria, red blood cell casts, pyuria, and mild to moderate proteinuria1

Note2: urine dipstick 1+ ≈ 30 mg/dl

Urine dipstick 2+ ≈ 100 mg/dl

Urine dipstick 3+ ≈ 300 mg/dl

Urine dipstick 4+ > 2 g/dl

จากการที่ผูปวยมีอาการ acute onset เกิดขึ้นภานในเวลา 2 วัน จึงคิดถึงภาวะ IgA

nephropathy มากที่สุด

Rx : There is no agreement on optimal treatment.

อาจให ACEI เพื่อรักษา proteinuria, steroid, fish oil1

28.) ตอบ C. Chronic myeloid leukemia

จากโจทย ก็คือ ชายคนหนึ่ง เพลีย มีมามโต bl.smear พบ granulocyte ทุก series และ

platelet สูง ซึ่งเขาไดกับ chronic myeloid leukemia(CML) มากที่สุด ทีนี้จะมาพูดถึงแต

ละชอยส อยางยอๆนะ

1. Acute myelofibrosis เปนโรคในกลุม Idiopathic myelofibrosis (ภาวะที่เซลลตาง

ๆ ในไขกระดูก โดยเฉพาะอยางยิ่ง fibroblasts เจริญเติบโตมากผิดปกติจนทําให

ไขกระดูกเกิด fibrosis) ประกอบดวยโรคที่สําคัญ 3 ชนิด คือ

1. Agnogenic myeloid metaplasia and myelofibrosis (AMMM) ขอไมกลาวถึง

2. Acute myelofibrosis (AMF): มีอาการรุนแรง(ซีด ออนเพลีย น้ําหนักลด)

ผูปวยเสียชีวิตอยางรวดเร็ว ไขกระดูกมี fibrosis แตเซลลไขกระดูกมีรูปรางปกติ

ยกเวน megakaryocytes และ megakaryoblasts รูปรางแปลก คลายผูปวย acute

megakaryoblastic leukemia (M7)

3. Childhood acute myelofibrosis (C-AMF) ขอไมกลาวถึง

ลักษณะ bl.smear : พบลักษณะ leukoerythroblastosis

2. Acute myeloid leukemia - อาการ : pancytopenia , Blood smear : blast มากมาย

3. CML เปนโรคมะเร็งเม็ดเลือดขาวชนิดหนึ่ง ซึ่งจะเห็น proliferation ของ

granulocyte ที่โตเต็มที่ (neutrophil, eosinophil และ basophil) และเซลลตน

กําเนิดของเซลลเหลานี้เปนจํานวนมาก

CML เปนผลจาก neoplastic transformation ของ pluripotential stem cell ซึ่งแม

จะเปนตนกําเนิดของเซลลหลายชนิด แตมักจะแสดงออกในรูปของ excessive

granulopoiesis อยางเดียว หรือมีเกล็ดเลือดสูงรวมดวย

สาเหตุของ CML ที่แทจริงยังไมแนนอน ประมาณรอยละ 90 ของผูปวย CML จะ

Page 48: Nle step 2_2009 si115-116 and nle_step_2_2009 nctms editors cut key

SIRIRAJ

SIRIRA

J SIRIR

AJ

พบ Philadelphia chromosome (Ph) รวมดวย ซึ่งถือเปน hallmark ของ CML

Blood smear ก็ตามโจทยเลย

4. Malignant lymphoma ไมเกี่ยวที่สุด

อาการเริ่มตนของมะเร็งตอมน้ําเหลืองที่พบไดบอย

- การพบกอนที่บริเวณตางๆ ของรางกาย เชน ที่คอ รักแร หรือ ขาหนีบ โดยกอน

ที่เปนมะเร็งตอมน้ําเหลืองนั้นมักไมเจ็บ ซึ่งตางจากการติดเชื้อที่มักมีอาการเจ็บที่

กอน

- ไข หนาวสั่น- มีเหงื่อออกมากตอนกลางคืน

- เบื่ออาหาร น้ําหนักลด ออนเพลียโดยไมทราบสาเหตุ

- อาการคันทั่วรางกาย

- ปวดศีรษะ (พบในมะเร็งตอมน้ําเหลืองในระบบประสาท)

อาการของมะเร็งตอมน้ําเหลืองในระยะลุกลาม

ในผูปวยบางราย จะพบอาการปวดที่ตอมน้ําเหลืองหลังการดื่มเครื่องดื่มที่มี

แอลกอฮอล

ในรายที่เปนมะเร็งตอมน้ําเหลืองภายในชองทอง จะพบอาการแนนทอง อาหาร

ไมยอย คลําไดกอนในชองทอง อมน้ําเหลืองที่โตจะมีผลกดเบียดอวัยวะขางเคียง

เชน หลอดเลือด หรือ เสนประสาท อาจทําใหเกิดอาการชา หรือบวมตามแขนขา

ได

29.) ตอบ A. life style modification นาจะใหผลระยะยาวดีที่สุด

30.) ตอบ B. ควบคุมอาหารและออกกําลังกาย / รายนี้จัดวาเปน DM เพราะ FBS 2 ครั้ง

เกิน 126 แตสามารถให lifestyle modification กอนได

Pediatrics

1.) ตอบ B. aminoplylline infusion

เนื่องจากใชในการรักษาภาวะ severe asthmatic attack ที่ไมตอบสนองตอ B agonist

3.) ตอบ B. atropine

จากการตรวจรางกายเขาไดกับ organophosphate poisoning ซึ่งไดแก Salivation,

Lacrimation, profund sweating, miosis, vomiting, diarrhea, paralysis และ decreased

consciousness / สําหรับ antidote คือ Atropine +/- Pralidoxime

Antidote ในตัวเลือกอื่นๆ มีรายละเอียดดังนี้

Antidotes substance Sign & Symptom

Naloxone Opioids pinpoint pupils, respiratory depression,

unconsciousness

Methanol Headache, visual changes, dizziness Ethanol

Ethylene glycol Ataxia, hallucinations, seizures, sweet breath

Flumazenil Benzodiazepines Sedation, respiratory depression

4.) ตอบ B. Beri beri

:three or more months of thiamine-deficient diet แบงไดเปน

Wet beri beri : cardiovascular system แบงเปน acute กับ chronic เกิดจากภาวะที่

heart ไมสามารถที่จะmaintain function ได อาการจะมี tachycardia, low diastolic pressure,

pulmonary edema ภาวะนี้จะมี lactic acid เพิ่มมากขึ้น ซึ่งภาวะนี้ reverseไดดวย thiamine

injection

- Chronic wet beriberi with high-output cardiac failure 3 stages.

1. peripheral vasodilatation ,high-output cardiac failure.

2. Then, the progression of vasodilatation . kidney as a relative loss of volume.

Renin angiotensin system produces greater salt and water retention.

3. Fluid overload results in peripheral edema and pulmonary effusions.

Dry beri beri : neuromuscular เปนลักษณะของ polyneuritis and symmetrical,

ascending paralysis of peripheral nerve system โดย sensory system จะ involve กอน

Page 49: Nle step 2_2009 si115-116 and nle_step_2_2009 nctms editors cut key

SIRIRAJ

SIRIRA

J SIRIR

AJ

ตามมาดวย motorและANS อาการจะเริ่มที่ lower extremitiesกอนแลวจึงไป involve ที่

upper extremities

Muscular dystrophy มี 2 โรคเดนคือ Duchenne muscular dystrophy(DMD) กับ

Becker muscular dystrophy เปน X-linked recessive disorder

- DMD เปนความผิดปกติจาก deficiency of dystrophin , CK elevate มีผลตอ axial

and proximal muscle > distal muscle, pseudohypertrophy of gastrocnemius, waddling

gait,scapular wing / มักจะมาดวย progressive weakness ,onset at 3-5 ป, mental

retardation is common / ซึ่งตางจาก Becker ตรงที่ Becker จะมี onset ที่ 5-15 ป และ

dystrophin level จะปกติ แต protein จะผิดปกติ

5.) ตอบ E. Acute lymphoblastic leukemia

เนื่องจาก อาการเขาไดกับ ALL ซึ่งเปน hematologic malignancy ที่พบบอยสุดในเด็ก

ผูปวยมาดวย pancytopenia ซึ่งบอกถึงการมี bone marrow failure (โรคนี้จํานวนเม็ดเลือด

ขาวอาจจะสูงหรือต่ําก็ได) %Lสูง มีตับโต

สวน Wilm’s tumor มักมาดวยบังเอิญคลําพบกอนที่ทอง ไมขาม midline อาจมาดวย

ปสสาวะสีแดง

Hepatoblastoma ก็ซีดได แตไมไดมี leukopenia และ thrombocytopenia ถามีอาจพบ

thrombocytosis และจะมี AFP สูง

Non Hodgkin lymphoma ก็นาจะมาดวย lymph node โต มีไข นน.ลดนะ แลวก็มัก

พบในคนสูงอายุและweaked immune

6.) ตอบ A. เปนภาวะปกติเพราะเปนผลจากพันธุกรรม

คนนี้เปน failure to thrive จากคา weight or height นอยกวา Percentile ที่ 3 ก็ตอง

คิดถึงวาเปน normal variant หาโรคทาง genetic หรือ endocrine ที่อาจเกี่ยวของแตดูจากพอ

แมแลวนาจะคิดถึงเปน familial มากที่สุด

9.) ตอบ A. laryngomalacia

advice วาสามารถ improve ภายในอายุ 2 ป

10.) ตอบ A. sinusitis

Sinusitis คือ การอักเสบของ paranasal sinus รวมทั้ง mucosa ของเยื่อบุจมูกเอง รวม

เรียกวา rhinosinusitis ซึ่งจะมีอาการนานตั้งแต 7-10 วันขึ้นไป แบงตามเวลา

1. Acute phase หรือ acute sinusitis เปนมานาน 7-10 วัน แตไมเกิน 4 สัปดาห

หรือ 30 วัน

2. Subacute sinusitis ระยะเวลานานประมาณ 4-10 สัปดาห

3. Chronic sinusitis คือระยะเวลามากกวา 2 สัปดาห

อาการและอาการแสดงของ acute sinusitis ไดแก 1. Nasal discharge, post nasal

drip มักจะมีอาการรุนแรง/มากชวงกลางคืน ทําใหไอบอย ๆ ตอนกลางคืน

2. ลมหายใจมีกลิ่นเหม็น

3. ไขต่ํา ๆ

4. บางคนอาจจะมี facial pain หรือปวดศีรษะได

PE : Inflammed mucosa . turbinate hypertrophy Mucopus d/c , mucopus in middle

meatus Tenderness at maxillary or frontal sinus

- จากประวัติและตรวจรางกายเขาไดกับ sinusitisมากที่สุด

Bronchiolitis = เปนกลุมอาการที่เกิดจากการอักเสบของหลอดลมฝอย (bronchioles)

ซึ่งเกิดจากการติดเชื้อ พบในผูปวยเด็กอายุนอยกวา 2 ป โดยเฉพาะอยางยิ่งในชวงอายุ

ระหวาง 2-6 เดือน / สาเหตุสวนใหญเกิดจากไวรัส โดยเฉพาะอยางยิ่ง respiratory

syncytial virus (RSV)

อาการและอาการแสดง : ผูปวยมักมีประวัติน้ํามูกใสๆและไอนํามากอน บางรายมีไขต่ําๆ

รวมดวย หลังจากนั้นประมาณ 1-2 วันจะมีอาการหายใจเร็ว อกบุมและหายใจมีเสียงหวีด

(wheezing) ตรวจรางกายพบวามีหายใจเร็ว บางรายอาจไมมีไขหรือมีไขสูงถึง 41 เซลเซียส

ได และมักตรวจไดยินเสียง wheezing จากการฟงปอด อาการทางคลินิกแยกไมไดระหวาง

acute bronchiolitis ที่เกิดจาก RSV, hMPV หรือไวรัสตัวอื่นๆ ผูปวยสวนใหญจะมีอาการดี

ขึ้นภายใน 3-4 วันภายหลังจากไดรับการรักษาแบบประคับประคอง ในรายที่มีภาวะการ

Page 50: Nle step 2_2009 si115-116 and nle_step_2_2009 nctms editors cut key

SIRIRAJ

SIRIRA

J SIRIR

AJ

หายใจลมเหลวและตองใชเครื่องชวยหายใจ พบวาระยะเฉลี่ยของการใชเครื่องชวยหายใจ

ประมาณ 5 วัน ผูปวยสวนใหญจะมีอัตราการหายใจและระดับคารบอนไดออกไซดใน

เลือดเปนปกติภายใน 2 สัปดาห

11.) ตอบ A. Choanal atresia

Choanal atresia หรือการอุดตันของโพรงหลังจมูก เปนความผิดปกติแตกําเนิด เด็กจะ

มีอาการเขียวเนื่องจากเด็กแรกเกิดไมสามารถหายใจทางปากได เด็กจึงมีอาการตัวเขียวแต

อาการจะดีขึ้นเมื่อรองเนื่องจากทําใหหายใจทางปากได

TE fistula คือการที่มี tract เชื่อมระหวางหลอดลมกับหลอดอาหาร เด็กจะหายใจได

ตามปกติ แตจะมีอาการสําลักขณะดูดนม อาเจียน สามารถทดสอบไดโดยการรใส NG tube

แลวพบวามีการอุดตัน

PDA เด็กจะมาดวยอาการหัวใจวาย ตรวจพบมี bounding pulse

12.) ตอบ C. แยกจากพอแม แลวคอยซักประวัติ

Management (child abuse) - 1.Admit

2.Holisticรางกาย จิตใจ Hx PE film + ตรวจ genitalia

3.ไมควรโทษผูเลี้ยงดู ควรถามปญหา ชวยเหลือปญหาสุขภาพกายและจิต

4. รายงานหนวยงาน

5.Consult จิตเวชเด็กทุกราย

เราวาอะ ควรไดประวัติที่ถูกตองกอนจาDxอะ แลวคอยไปดูเรื่องจิตเวชครอบครัว

13.) ตอบ A. insulinเพิ่ม

เพราะacanthrosis nigricanจะพบในคนไขที่hyperinsulinemia

14.) ตอบ A. arthrocentesis

สงสัย septic arthritis Rt. Knee (มีLNโต) สง arthrocentesis เลยจะไดเพาะเชื้อดูเพื่อให

ATB

An aggressive workup is indicated whenever signs and symptoms suggest septic

arthritis. This evaluation must include blood cultures and joint aspiration.

- ESR is elevated in septic arthritis; it returns to normal levels with resolution of the

infection. นาจะเอาไวเพื่อ follow up มากกวา

- Radiographs are not helpful in diagnosing acute septic arthritis.

15.) ตอบ c. Athrocentesis

ไมแนใจนะ แตคิดวานาจะตอบ Athrocentesis เพราะคิดถึง septic arthritis จึงควรเจาะ

น้ําจากขอมาเพาะเชื้อ สวนserum uric acid นั้นไมนาใช เพราะอายุแค 17 ป

17.) ตอบ C. OPV4, DPT4, JE3

ไปเปดดูตารางวัคซีนที่ใหในแตละชวงอายุดูนะ

18.) ตอบ B. atropine

อาการจากโจทยเปนกลุมอาการที่ไดรับสาร Cholinergic ซึ่งจะมีอาการ คือ

Hypertension and tachycardia (early) hypotension and bradycardia (late) ,alteration of

conscious ,diaphoresis ,increased secretion ,diarrhea ,bronchospasm

การดูแลรักษา ตรวจ Cholinesterase activity และให atropine and pralidoxime

19.) ตอบ D. Plain film abdomen

Plain film abdomen เพื่อดู sign ของภาวะ gut obstruction – ตําแหนง , พยาธิสภาพ ,

ลักษณะของsmall bowel obstruction , -dilatation of small bowel , -no air in colon and

rectum , -double bubbles sign

20.) ตอบ A. pericardiocentesis

จากผลการตรวจรางกายพบมี hypotension, engored neck vein, distant heart sound ->

ทั้งหมดเขาไดกับ Beck’s triad ที่เปนสิ่งที่ชวยวินิจฉัยวาผูปวยมี cardiac tamponade จึงควร

ใหการรักษาโดยการทํา pericardiocentesis

สวนสาเหตุของการเกิด tamponade นั้น คิดวานาจะเกิดจาก เชื้อ bacteria พวก strep,

staph จากบริเวณแผลที่หกลม แลวมี hematogenous spreading ทําใหเชื้อไปที่หัวใจ จนเกิด

pancarditis และ มี pericardial effusion ตามมาได

23.) ตอบ A. viral load ไมแนใจ

Page 51: Nle step 2_2009 si115-116 and nle_step_2_2009 nctms editors cut key

SIRIRAJ

SIRIRA

J SIRIR

AJ

OB GYN

1.) ตอบ B. left mento-posterior

2.) ตอบ A. emergency C/S

คนนี้ คิดวาอยูในภาวะ Threaten ruptured uterus หรือคือมดลูกใกลแตก ซึ่งจะพบมดลูกหด

ตัว มีรอยคอด ที่เรียกวา Pathological band of bandle ซึ่งจะอยูสูงกวา รอยคอดของมดลูก

ปกติ ซึ่งถาคนนี้ ไมไดรับการดูแลที่เหมาะสม มดลูกก็แตกได โดย sequence จะเปนอยางนี้

1. Hyperstimulation uterus = มีการหดตัวนานมากกวา 2min , มากกวาหรือเทากับ 5

ครั้งใน10นาที

2. Threaten ruptured uterus เห็น Pathological band of bandle (แนวขวางอยูต่ํากวา

สะดือเล็กนอย) รักษาโดย emergency C/S

3. Uterine rupture มักเกิดใน late pregnancy, previous c/s, augmentation, high

parity, คลอดยาก

S&S : ปวดทอง มีเลือดออกทางvagina, FHRลดลง***, shock, ในที่สุดมดลูกไมหดตัว

รักษา : hysterectomy, repair +/- tubal sterilization

3.) ตอบ A. Ut atony

ขอนี้ เปน Early post partum bleeding (ทางVg >500cc, c/s >1000 cc)ซึ่งมี ddx ดังนี้

Tone : Uterine atony อาการก็ตามโจทยแหละ

Trauma : รกครบ เห็นแผลฉีกขาด มดลูกหดตัวดี

Tissue : Retained placenta ไมคลอดรกหลัง 30นาที, มดลูกหดตัวดี

Thrombin : coagulopathy

รักษา uterine atony : call for help, resuscitation, bimanual compression, empty

bladder, Med: oxytocin>>methergin>>prostaglandins>>>Sx

4.) ตอบ A. Ultrasound

ในผูปวยรายนี้มีภาวะ antepartum hemorrhage โดยเปน painless bleeding จึงคิดถึง

ภาวะ placenta previa จึงพิจารณาทํา ultrasonography

5.) ตอบ E. ฝากครรภตามปกติ

โรคที่มีความรุนแรงตอทารกจนตองทํา prenatal diagnosis ไดแก Hb Bart’s hydrops

fetalis , B thal homozygous , B thal /Hb E จึงตองหา gene ที่มีความเสี่ยง คือ α thal 1 , B

thal , Hb E จากโจทย มารดามี Hb typing AE ซึ่งอาจแปลได 3 ลักษณะคือ

Hb A + E (25-30%) = Hb E trait

Hb A + E (< 25%) และ MCV < 80 fl ถา Hb ปรกติ = HbE trait รวมกับ αthal1หรือ2 trait

Hb A + E (< 25%) และ MCV < 80 fl ถา Hb ต่ํากวาปรกติ = HbE trait รวมกับ iron

deficiency anemia ดั้งนั้นอาจมี gene ที่มีความเสี่ยง คือ α thal 1 , Hb E ซึ่งอาจตรวจสามี

วามี gene เหลานี้หรือไม โดยสามีของผูปวยรายนี้ มี Hb typing อยูในเกณฑปกติ คือ Hb

A+A2 (2-3.5%) ,F~1% ดี ดังนั้นในผูปวยรายนี้เปน non risk couple จึงฝากครรภปกติได

แตถาโจทยบอกวามารดามี anemia อาจตอบ ใหเหล็กกินวันละ 3 ครั้งได

6.) ตอบ B. herpetic vulvitis

จากประวัติและการตรวจรางกายที่เปน vesicle & shallow ulcer at valva คิดถึง

herpetic vulvitis

7.) ตอบ A. ตรวจ chromosome

8.) ตอบ A. Ascending urethral transmission

เหตุผล เพราะผูปวยมีประวัติมีเพศสัมพันธมากอนมีอาการ จึงนาจะทําใหเกิด

Ascending infection ได ทั้ง PID และ UTI

9.) ตอบ ไมแนใจ

เหตุผล เพราะตองพิจารณาโจทยเพิ่มเติม เพราะมารดามีอายุมากแลว การตั้งครรภก็มี

ความเสี่ยงสูงมาก โอกาสเกิดความผิดปกติของโครโมโซมก็มาก เพราะฉะนั้นมีผลเสียทั้ง

Page 52: Nle step 2_2009 si115-116 and nle_step_2_2009 nctms editors cut key

SIRIRAJ

SIRIRA

J SIRIR

AJ

ตอตัวมารดาและบุตร แตถาความจําเปนในการตองการมีบุตรสูงมาก ก็อยางขอ 10 ก็นาจะ

คลาย ๆ กัน คือ อาจใหตั้งครรภ โดยทํา amniotomy เปนprenatal diagnosis

สรุป ควรอานโจทยใหครบถวน และใชวิจารณญาณเองดวย

10.) ตอบ ???

Down’s syndrome

- Trisomy 21 (47,XX,+21) พบได 95% ของผูปวย down’s syndrome

- Robertsonian translocation [45,XX,der(14;21)(q10;q10)] พบได 2-3%

- Mosaicism (46,XX/47,XX,+21) พบได 1-2%

11.) ตอบ A. manual rotation

Mento-posterior

- Second stage:

- Wait for long anterior rotation of the mentum 3/8 circle and the head will be

delivered as mento-anterior. During this period oxytocin is used to compete inertia

which is common in such conditions as long as there is no contraindication. Failure

of this long rotation is more common than in occipito-posterior position so earlier

interference is usually indicated.

- Failure of long anterior rotation 3/8 circle or development of foetal or

maternal distress at any time, is managed by:

- Caesarean section: which is the safest and the current alternative in modern

obstetrics.

- Manual rotation and forceps extraction as mento-anterior, or

- Rotation and extraction by Kielland forceps.

- In the last 2 methods the head should be engaged but they are hazardous to

both the mother and foetus so they are nearly out of modern obstetrics.

- Craniotomy: if the foetus is dead

ในขอนี้คิดวา station ยังไมพรอมจะใช F/E

12.) ตอบ ???

ขอนี้ติดเชื้อแน ๆ หนังสือบอกตองตัดไหม เปดแผลระบายหนอง ในระยะแรกให

ATB+ยาแกปวด เลยไมแนใจวาขอไหนดี B/C ตองรวมกันนะ

13.) ตอบ B. ตรวจพบหลักฐานวาผูปวยถูกกระทําชําเรา

-บาดแผลที่พบไดบอย คือที่ตําแหนง posterior fornix,hymnen at 4-8 o’clock,fossa

navicularis,labia minora

-old tear ที่ 6 o’clock หมายถึง มี evidence of penetration มานานกวา 3 วัน (

ของแข็งไมมีคม ) โดยปกติทั่วไปบาดแผลจะหายเองใน 3-5 วันมักไมเกิน 7 วันดังนั้น old

tear อาจเกิดจากผูปวยมาพบแพทยชาก็เปนได

-มี sperm ไมมีหาง แสดงวา sperm อยูใน vagina มานานกวา 16 hr. หางจึงหลุดเหลือ

แตหัว

-kiss bite( suction lesion ) at neck หมายถึง มี sexual arousment

Choice A ผิดเพราะหามบอกวาโดนขมขืน เพราะ ขมขืน ตองมีองคประกอบดังนี้

โดยขูเข็ญดวยประการใดๆ -ขมขูกรรโชก ขูวาจะเปดเผยความลับ ขูวาจะทําราย , ใชอาวุธ

ขูโดยใชกําลังประทุษราย , ทํารายรางกาย ใชอาวุธทําราย , โดยผูอื่นนั้นไมสามารถขัดขืน

ได , สภาวะบางอยางเชน ถูกมอมดวยยา , โดยทําใหเขาใจผิดวาตนเปนบุคคลอื่นทําให

เขาใจวาเปนบุคคลที่จะยอมมีเพศสัมพันธดวย

Choice C การกระทําชําเราหมายความวา การกระทําเพื่อสนองความใครของผูกระทํา

โดยการใช อวัยวะเพศของผูกระทํากระทํากับอวัยวะเพศ ทวารหนัก หรือชองปากของผูอื่น

หรือการใชสิ่งอื่นใดกระทํากับอวัยวะเพศหรือทวารหนักของผูอื่น

14.) ตอบ C. ไมทําใหเนื่องจากไมมีขอบงชี้ในการยุติการตั้งครรภทางการแพทย

การยุติการตั้งครรภทางการแพทย (therapeutic abortion)มี indication คือ

- มีคําสั่งจากศาลใหยุติการตั้งครรภ เชน กรณีโดน rape

- มีผลตอสุขภาพของแมทางรางกายหรือจิตใจ

Page 53: Nle step 2_2009 si115-116 and nle_step_2_2009 nctms editors cut key

SIRIRAJ

SIRIRA

J SIRIR

AJ

- ลูกมีความพิการ (ในไทยไมใชขอนี้)

15.) ตอบ A. Uterine rupture

รอยแยกระหวางสวนบนกับสวนลางของมดลูกเรียกวา Physiologic retraction ring ซึ่ง

จะพบที่สันดานในของโพรงมดลูก ในกรณีที่ตําแหนงของ retraction ring เลื่อนขึ้นไปสูง

กวาปกติและอาจสังเกตเห็นเสนนี้ไดชัดเจนทางหนาทอง เรียกวา Pathological ring of

bandle เปนสัญญาณเตือนวาอาจเกิดภาวะ Uterine rupture ตามมาได

16.) ตอบ B. set C/S emergency ไมงั้นแตกแน

SURGERY

1.) ตอบ C. FNA

ในรายนี้สงสัยวาเปน Fibrocystic change ซึ่งเกิดการเปลี่ยนแปลงไดหลายอยางตาม

รอบเดือน ในรายนี้เปนลักษณะของ cyst ดังนั้นจึงแนะนําใหทําFNA เจาะน้ําออก หากกอน

ยุบหมดและไมมีเลือดปน ก็ไมจําเปนตองผาตัด แตถามีเลือดปนหรือเจาะไดเลือดหรือกอน

ยุบไมหมดหรือพบเซลลนาสงสัยในน้ําที่เจาะออกมาหรือกอนrecurบอย ควรตัดเอากอน

ออกมาตรวจ

2.) ตอบ A. acute appendicitis

ตอนแรกเปน visceral pain ตาม dermatome ที่ 10 จึงมีอาการปวดทองรอบๆสะดือ ไม

สามารถบอกตําแหนงไดชัดเจน

เมื่อเริ่มมีการ irritate peritoneum (parietal pain) ก็จะมีอาการปวดที่ RLQ ที่ตําแหนง

McBurney’s point

5.) ตอบ A. Hidradenitis suppurativa

ลักษณะตรงกับคนไขตามที่ขีดเสนใต ดังนี้

• The patient may present in considerable pain, with multiple red, hard, raised

nodules in areas where apocrine glands are concentrated.

• Affected areas may include the axillae, periareolar, intermammary zones, pubic

area, infraumbilical midline, gluteal folds, genitofemoral areas (top of the

thighs in genital area), and the perianal region.

• As suppuration progresses, surrounding cellulitis may be present. Chronic

recurrences result in palpable thick sinus tracts under the skin, which may turn

into draining fistulas.

• The patient may present with a chronic condition in which the multiple nodules

have coalesced and are surrounded by a fibrous reaction. This results in scarred

and unsightly appearance of the area.

• Hidradenitis suppurativa may resemble recurrent bacterial folliculitis and

furunculosis

แต Hidradenitis suppurativa มักไมเปนตุมเดียวจะเปนทีละเยอะๆนําเปน area มักไมมี

รูเปดรูเดียว

จริงๆ คิดวานาจะตอบวาผูปวยเปน Fistula in ano มากกวา เพราะ Chronic เกินกวาจะ

เปน perianal abscess แตอาจมีภาวะดังกลาวที่ไดรับการรักษาไม adequate นํามากอน ทํา

ใหเกิดเปน fistula ที่มีรูเปดเดี่ยว บริเวณเดิมที่มีหนองแตกภายหลัง และมีหนองและเลือด

ไหลเปนครั้งคราวได แตตัวเลือกไมมีใหเลือก เพื่อนๆพิจารณาเลือกเอาละกัน

ORTHO

1.) ตอบ B. steroid injection

จากประวัติและการตรวจรางกายคิดถึง carpal tunnel syndrome

Treatment : Local steroid injection or splinting is suggested when treating patients

with carpal tunnel syndrome, before considering surgery

Page 54: Nle step 2_2009 si115-116 and nle_step_2_2009 nctms editors cut key

SIRIRAJ

SIRIRA

J SIRIR

AJ

2.) ตอบ A. ใสเฝอกแขนทอนบนและลาง (Hanging Cast)

จากผูปวยมาดวยเรื่อง fracture shaft of humerus สิ่งที่ควรทราบคือ radial nerve จะอยู

ชิดกับกระดูก humerus โดยวิ่งออมจากดานหลังมาดานหนาตามแนว spiral groove ดังนั้น

เมื่อมี fracture ของ humerus จึงมีโอกาสเกิด radial nerve palsy ได โดยจะตรวจพบ wrist

drop ,ชาหลังมือบริเวณ first web space , finger extension ไมไดซึ่งเขาไดกับผูปวยรายนี้

การรักษาสวนมากจะใชวิธี conservative ซึ่งไดผลดี กระดูกติดใน 6-8 สัปดาห โดยจะทํา

Coaptation splint (Sugar tong, U-slap) แตมักทําในกรณีกระดูกหักหลายชิ้น หรือมี

distraction ใสนาน 4-6 สัปดาห หรือเลือกทํา Hanging cast ซึ่งเปนการใสเฝอกเหนือ

รอยหัก 1 นิ้วจนถึงขอมือ ใชในกรณีที่มีการเกยกันของกระดูก ดังรูป

โดยไมจําเปนตองทํา EMG เนื่องจากสวนใหญ 75-90 % ของ radial nerve palsy จะ

เปน neurapraxia ,สามารถหายไดเองใน 3-4 เดือน

I/C for surgery

1. Vasucular injury

2. Multiple injury มีกระดูหักหลายแหง

3. Open fracture

4. Bilateral humeral fracture

5. Pathological fracture

6. ยังมี radial palsy after closed reduction ( ขอนี้

แหละที่ทําใหงงวาจะตอบ D ดีไม )

7. ไมสามารถจัดแนวกระดูกได

8. Non union

3.) ตอบ D. ทํา hyperextension of Rt. Wrist

คิดถึงภาวะtennis elbow หรือ lateral epicondylitis testโดย Cozen’s test is performed

by having the patient flex the elbow and extend the wrist against resistance. This will

contract the forearm muscles and usually irritate the insertions on the lateral epicondyle

reproducing the pain felt in the elbow.

Yergason test การตรวจเอ็นของกลามเนื้อ Biceps ตรวจโดยการใหผูปวยงอขอศอก 90

องศา และคว่ํามือ ใหผูตรวจออกแรงตานการหงายมือของผูปวย และดึงตนแขนลงดานลาง

อาการเจ็บที่เอ็น Biceps บริเวณขอไหลแสดงการอักเสบของเอ็นนี้

Valgus test or abduction stress test evaluates the medial collateral ligament of knee

4.) ตอบ A. Triceps - neck of humerus มี radial n. ทอดผาน ซึ่งเลี้ยง Triceps

5.) ตอบ B. Spinal stenosis

เนื่องจากอาการปวดสะโพกราวลงขาเวลาเดิน นั่งพักแลวดีขึ้นเปนอาการของ spinal

stenosis ซึ่งมักพบในคนที่อายุมาก

Spondylosis = ปวดมากขึ้นเวลาเปลี่ยนทา Protrude mucleus pulposus = มักมี

ประวัติยกของหนัก

6.) ตอบ D. arthrocentesis

จากประวัติคิดถึง 1) septic arthritis เนื่องจากผูปวยมีไข และมีขอเขาอักเสบ

2) fracture เนื่องจากทีประวัติอุบัติเหตุที่หัวเขา และ ปวดเขา แต ไมนาจะมีไข อีกทั้ง

นาจะมีอาการปวดเมื่อ1 สัปดาหกอนหนานี้แลวมาโรงพยาบาล

เพราะฉะนั้นจึงคิดถึง septic arthritis มากที่สุด การสงตรวจเพิ่มเติมคงเปน arthrocentesis

เพื่อ diagnosis ซึ่งเหมาะสมที่สุดสวนการทํา X ray อาจตองทําเชนกันเพื่อ ดูวามี fracture

หรือไม

7.) ตอบ C. resist pronation test

คิดถึงโรค Pronator teres syndrome มากที่สุดจึงคิดวานาจะทํา resist pronation test ตอ

Page 55: Nle step 2_2009 si115-116 and nle_step_2_2009 nctms editors cut key

SIRIRAJ

SIRIRA

J SIRIR

AJ

Pronator teres synd. เกิดจาก median nerve entrapment บริเวณ elbow จะมีอาการ

tenderness over the pronator teres muscle ที่aggravated by resisted pronation of the

forearm มีอาการปวดแขนและขอมือ ตรวจรางกายพบวามี abductor pollicis brevis weak

และมีชาสามนิ้วครึ่งดาน radial side ซึ่งทั้งmuscleนั้นและอาการชา เขาไดกับdistribution

ของ median nerve แต Tinel sign negative at wrist แสดงวาไมนาจะเปนโรค Carpal

tunnel syndrome นอกจากนี้ CTSจะไมมีอาการปวดแขน สวนใหญมักปวดขอมือมากกวา

8.) ตอบ B. steroid injection

คิดถึง carpal tunnel syndrome ดังนั้นการรักษาที่เหมาะสมคือ steroid injection

9.) ตอบ B. lumbar stenosis

ผูปวยรายนี้มาดวยอาการ back pain with sciatica และมี neurogenic claudication เขา

ไดกับ spinal stenosis ที่ระดับ lumbar มากที่สุด สวนตัวเลือกอื่นๆ D) arterial

insufficiency ไมนาจะมี sciatica และ claudication ในผูปวยรายนี้เขาไดกับ neurogenic

cause มากกวา vascular cause, A)spinal instability อาการปวดจะมากขึ้นเมื่อขยับตัว

เปลี่ยนทาทาง ดีขึ้นเมื่ออยูเฉยๆ และไมอธิบาย claudication, สวน C) spondylolisthesis

เปนสาเหตุหนึ่งของ spinal stenosis จึงสามารถทําใหเกิดอาการทั้งหมดที่กลาวมาไดขึ้นกับ

ระดับความรุนแรงของการเลื่อน แตเนื่องจากในขณะนี้ไมมีขอมูลอื่นๆที่จะสามารถใหการ

วินิจฉัยภาวะนี้ได เชน ประวัติอุบัติเหตุกอนหนานี้, หรือภาพรังสีเพื่อชวยวินิจฉัย จึงคิดวา

ในขณะนี้นาจะใหการวินิจฉัยวา lumbar stenosis ไวกอน

Eye & ENT

1.) ตอบ A. Laser

ผูปวยเปน DM มี NV แลว เปน PDR (Proliferative Diabetic Retinopathy) ซึ่ง

Complication ที่สําคัญที่ตองระวังเมื่อมี NV แลวคือ เกิด tractional retinal detachment

ตามมาได การรักษาใหควบคุม DM ใหดี และ PRP (PanRetinal Photocoagulation)

(Argon laser) บริเวณที่มี ischemia เพื่อปองกันการสราง VEGF ซึ่งจะสราง NV ขึน้มาใหม

อีก แตถาเปน NPDR (ยังไมมี NV เกิดขึ้น) ก็ยังไมตอง PRP

3.) ตอบ E. SNHL

หูอื้อขวา ตรวจweber lateralization to left แสดงวาขางขวาเสียแบบ SNHL เพราะ

weber จะ lateralize ขาง conductive หรือ ขางตรงขามกับ SNHL

ตรวจ rinne AC>BC ในขางปกติ หรือ SNHL , choice ทุกขอ ยกเวนขอ E เปน conductive

hearing lossหมด

4.) ตอบ D. myringotomy

I/C for myringotomy

1. Severe acute otitis media (middle ear infection)

2. Hearing loss greater than 30 dB in patients with otitis media with effusion

present for greater than three months.

3. Poor response to antibiotics for otitis media.

4. Impending mastoiditis or intracranial complications due to otitis media.

5. Recurrent episodes of acute otitis media - (more than 3 episodes in 6 months or

more than 4 episodes in 12 months).

6. Chronic retraction of tympanic membrane or pars flaccida. (may eventually form

a cholesteatoma)

7. Barotrauma

8. Autophony (hear body sounds such as breathing) due to patulous (wide open)

eustachian tube.

9. Craniofacial anomalies that predispose to middle ear dysfunction (e.g. cleft

Palate ).

10. Middle ear dysfunction due to head and neck radiation and skull base surgery.

Page 56: Nle step 2_2009 si115-116 and nle_step_2_2009 nctms editors cut key

SIRIRAJ

SIRIRA

J SIRIR

AJ

5.) ตอบ B. Malignant otitis externa

Malignant otitis externa เปน otitis externa + skull base osteomyelitis จาก

Pseudomonas พบใน IC host esp. DM

S&S: severe otalgia, hearing loss, fever ตรวจหูพบgranulation tissueใน ext ear

canal, TM ปกติ

ผูปวยอาจมีอาการหูอื้อ หูตึงถามีเนื้อเยื่ออักเสบบวมอุดกั้นในชองหู และเนื่องจาก

osteomyelitisลามไปตามskull baseจึงinvolve CN ที่โดนบอยสุดคือ CN7 ถาลามถึงjugular

foramen, hypoglossal canal จะโดน CN9, 10, 11, 12ดวย

Cholesteatoma: การสะสมของkeratinizing stratified sq. epi. และมีgramulation

tissue, inflammatory cell การขยายตัวของcholesteatomaจะทําลายกระดูกขางเคียง

เนื่องจากมีcollagenase, enzอื่น ๆ และinflammatory mediator

S&S: การไดยินลดลง น้ําหนวกเปน ๆ หาย ๆ ถามีอาการแทรกซอนจะ ปวดหู, ไข, ปวด

ศีรษะ, เวียนหัวรุนแรง, อัมพาตของใบหนา ถายังไมมีภาวะแทรกซอนก็จะมีแตอาการทาง

หูเทานั้น นอกจากนี้ cholesteatoma มักจะไมทําใหมี ear canal edema

Psychi

1.) ตอบ C. ขาขวาถูกตัดขาด 2 ขาง หลังอุบัติเหตุมา 6 เดือน

ประเภทของความพิการ

- 1.พิการทางการมองเห็น : คนที่มีสายตาขางที่ดีกวาเมื่อใชแวนสายตาธรรมดา แลว

มองเห็นนอยกวา 6/18 หรือ 20/70 จนมองไมเห็นแมแตแสงสวาง หรือมีลานสายตาแคบ

กวา 30 องศา

- 2. พิการทางการไดยินและสื่อความหมาย :

- เด็กอายุนอยกวา 7 ป คือ ความดังเฉลี่ยเกิน 40 เดซิเบล

- เด็กอายุเกิน 7 ป ถึงผูใหญ คือ ความดัง 55 เดซิเบลขึ้นไป

- 3. พิการทางกาย หรือการเคลื่อนไหว : คนที่มีความผิดปกติหรือความบกพรองของ

รางกายเห็นชัดเจน และคนที่สูญเสียความสามารถในการเคลื่อนไหว ทําใหไมสามารถ

ประกอบกิจวัตรในชีวิตประจําวันได

- 4. พิการทางจิตใจหรือพฤติกรรม : คนที่มีความผิดปกติ ความบกพรองทางจิตใจ

หรือสมองในสวนรับรู อารมณ ความคิด จนไมสามารถควบคุมพฤติกรรมที่จําเปนในการ

ดูแลตนเอง หรืออยูรวมกับผูอื่น

- 5. พิการทางสติปญญาหรือการเรียนรู :

-เด็กเรียนชา IQ 70-90

-เด็กปญญาออน IQ ต่ํากวา 70

- 6. พิการซ้ําซอน คือ มีความพิการมากกวา 1 ลักษณะ

2 ตอบ ขอ D)

เหตุผล เพราะผูปวยอยูในเกณฑ mild MR (IQ 55-69) ซึ่งสามารถเรียนจนจบประถม 6

ได โดยเรียนรวมกับเด็กปกติได สามารถฝกใหปรับตัวเขากับสังคมได ฝกอาชีพได เลี้ยง

ตนเองได แตตองการคําแนะนําและการชวยเหลือบาง เพราะฉะนั้นก็นาจะใหเขาเรียนได

แตอาจตองมีครูชวยดูแลพิเศษ

4.) ตอบ B. ไมตองทําอะไร

ในการตรวจวินิจฉัยคัดกรองมะเร็งลําไสใหญ ไดมีการแบงปจจัยในการเกิดโรคมะเร็ง

ลําไสใหญ เปน 3 กลุม ดังนี้

กลุมที่ 1 คนที่มีความเสี่ยงเทากับประชากรทั่วไป (Average risk) คือ กลุมคนที่

อายุมากกวา 50 ป โดยไมมีประวัติมะเร็งลําไสใหญในครอบครัว กลุมนี้ควรตรวจคัดกรอง

ดวยการตรวจหาเลือดในอุจจาระหรือไดรับการสองกลอง ลําไสใหญหลังอายุ 50 ป

กลุม ที่ 2 กลุมที่มีความเสี่ยงสูงกวาประชากรทั่วไป กลุมคนที่ เคยวินิจฉัยวาเปน

ติ่งเนื้อลําไสใหญ ควรไดรับการตรวจดวยการสองกลองทุก 3 - 5 ป กลุมคนที่เคยไดรับการ

ตรวจรักษาดวยการผาตัดมะเร็งลําไสใหญ ควรไดรับการตรวจดวยการสองกลองทุก2 - 3 ป

Page 57: Nle step 2_2009 si115-116 and nle_step_2_2009 nctms editors cut key

SIRIRAJ

SIRIRA

J SIRIR

AJ

สําหรับผูปวยโรคลําไสอักเสบเรื้อรัง (Ulchative colitis, Crohn’s disease) จะ มีความ

เสี่ยงตอมะเร็งลําไสใหญเพิ่มมากขึ้น หลังจากปวยเปนโรคลําไสอักเสบเรื้อรังประมาณ 8 -

10 ป จึงควรไดรับการสองกลองทุก 1 - 2 ป สวนคนที่มีญาติเปนมะเร็งลําไสใหญควร

ไดรับการตรวจคัดกรองดวยการ สองกลองเริ่มที่อายุ 40 ป หรือ 10 ป กอนอายุของญาติที่

ปวยเปนมะเร็งลําไสใหญ

กลุมที่ 3 กลุมที่มีความผิดปกติทางพันธุกรรมที่เสี่ยงตอการเปนโรคมะเร็งลําไส

ใหญ คือ คนที่มีญาติหลายคนเปนมะเร็งลําไสใหญ มะเร็งรังไข มะเร็งมดลูก หรือมีญาติ

เปนมะเร็งลําไสใหญ เมื่ออายุนอยกวา 50 ป หรือมีติ่งเนื้อที่ไมใชมะเร็งมากกวา 10 ปใน

ลําไสใหญ กลุมนี้ควรไดรับการตรวจอยางละเอียด เพื่อหาความผิดปกติทางพันธุกรรมจาก

แพทยผูเชี่ยวชาญดานมะเร็งวิทยา

5.) ตอบ A. ใหยาคลายกังวล

นาจะเปน Acute stress disorder อาการนาน 2 วัน-4 สัปดาห(ถาเปน posttraumatic stress

disorder) เปนโรควิตกกังวัลที่เกิดภายหลังที่ผูปวยเผชิญเหตุการณรุนแรง

การวินิจฉัย

1. ผูปวยเผชิญเหตุการณรุนแรง หรือทําใหรูสึกไมปลอดภัย

2. เหตุการณนั้นทําใหเกิดความกลัวที่รุนแรง หรือรูสึกสิ้นหวัง

3. คิดถึงเหตุการณนั้นทั้งขณะตื่นและหลับ(ฝน)

4. มีพฤติกรรมหลีกเลี่ยงเหตุการณนั้น หรือสิ่งที่เกี่ยวพันกับเหตุการณนั้น

5. มีอาการตื่นเตนตกใจงาย ทั้งๆที่ไมเคยเปนมากอน

6. อาการรุนแรงจนเกิดปญหากับหนาที่การงานความสัมพันธระหวางบุคคล และ

ชีวิตประจําวัน

การรักษา

- Ventilation and support,relaxation technique

- ใหยา amitriptyline/imipramine ลดซึมเศรา,alprazolam คลายกังวล,propanolol ลด

อาการตื่นเตน

6.) ตอบ C. เพิ่มยา diazepam

ในผูปวยรายนี้มีอาการของ mood disorder รวมกับมี psychosis จึงไดรับยา

Haloperidol ซึ่งเปนยาในกลุม typical antipsychotics (high potency) ที่มักเกิด side effect

คือ EPS ไดมาก ( EPS is a cluster of symptoms consisting of akathisia, parkinsonism,

dystonias.) นอกจากนี้ยังพบ common side effect อื่นไดแก anticholinergic effect เชน dry

mouth ,weight gain, tremor,muscle stiffness,muscle cramping และมี serious side effects

คือ tardive dyskinesia Neuroleptic malignant syndrome ในผูปวยรายนี้หลังไดรับยา

haloperidolมากินแลวมีอาการกระสับกระสาย นั่งนิ่งไมได เดินทั้งวัน เขาไดกับ akathisia

(ความรูสึกกระวนกระวายในใจ จนผูปวยอยูนิ่งไมได นั่งไมติด ผุดลุก ผุดนั่ง เดินไปเดินมา

ตองขยับแขนขา หรือ เคลื่อนไหวตลอดเวลา) การรักษามี 2 วิธี คือ วิธีที่หนึ่ง การรักษาโดย

การหยุดยาหรือลดยาที่เปนสาเหตุ (ในโจทยขอนี้ คือ Haloperidol แตไมมี choice อิอิ) วิธี

ที่สอง คือ การรักษาโดยการใชยาโดยจะใชยาในกลุม B-Blocker เชน Propranolol,Atenolol

หรือ ยาในกลุม benzodiazepines เชน lorazepam ,diazepam หรือยาในกลุมAnticholinergic

เชน Trihexyphenidyl hydrochloride(Benzhexol)

Fluoxetine เปนยา anti-depressants กลุม SSRIs

7.) ตอบ A.social phobia

เพราะเปนความกลัวตอการตองไปอยูในสถานการณที่ผูปวยอาจถูกจองมองจากผูอื่น

และผูปวยพยายามหลีกเลี่ยงหรือหลบหนีจากสิ่งที่ทําใหกลัว มีอาการอยางนอย 6 เดือน

: ไมเขากับ panic disorder ที่จะเกิดขึ้นเองโดยไมมีสิ่งใดมากระตุนและ มี recurrent

panic attack

: สําหรับ adjustment disorder ตองวินิจฉัยโดย rule outภาวะอื่นๆออกกอน

8.) ตอบ C. แนะนําปรึกษาจิตแพทย

เพราะเด็กคนนี้ เขาไดกับเกณฑวินิจฉัย ดังนี้ ... จึงควรไดรับการดูแลจากจิตแพทยเด็ก

และวัยรุน

ADHD เกณฑการวินิจฉัย (ตาม DSM-IV-TR criteria)

Page 58: Nle step 2_2009 si115-116 and nle_step_2_2009 nctms editors cut key

SIRIRAJ

SIRIRA

J SIRIR

AJ

ก. มีอาการตามขอ ๑ หรือ ๒ นานอยางนอย ๖ เดือน โดยอาการรุนแรงจนเปนปญหา

(maladaptive) และไมตรงกับระดับพัฒนาการ (developmental level)

๑. มีอาการสมาธิสั้น (Inattention) อยางนอย ๖ อาการ

๑.๑ ไมมีสมาธิสนใจรายละเอียด หรือทําผิดพลาดดวยความเลินเลอ (careless

mistakes) ในการเรียนหรือกิจกรรมตางๆ

๑.๒ มักไมมีสมาธิตอเนื่อง (sustaining attention) กับงานหรือการเลน

๑.๓ ดูเหมือนไมฟงเมื่อคนพูดดวย

๑.๔ มักทําตามคําสั่งไมจบหรือทํางานไมเสร็จ ซึ่งไมไดเกิดจากการดื้อ หรือไม

เขาใจคําสั่ง

๑.๕ มักมีปญหาในการจัดระบบงานหรือกิจกรรม

๑.๖ มักหลีกเลี่ยงหรือไมชอบที่จะทําสิ่งที่ตองใชความพยายาม เชนทําการบาน

๑.๗ ทําของหายบอยๆ

๑.๘ วอกแวก (distract) งาย

๑.๙ ลืมกิจวัตรบอยๆ

๒. มีอาการซน - หุนหัน (hyperactivity - impulsivity) อยางนอย ๖ อาการนานอยาง

นอย ๖ เดือน โดยอาการรุนแรงจนเปนปญหา (maladaptive) และไมตรงกับระดับ

พัฒนาการ (developmental level)

โรคสมาธิสั้ซุกซนอยูไมนิ่ง (Hyperactivity)

๒.๑ มักยุกยิก (figets) มือเทาไมนิ่ง หรือ squirms in seat

๒.๒ มักลุกจากที่นั่งขณะอยูในหองเรียนหรือสถานที่ที่ควรนั่งกับที่

๒.๓ มักวิ่งหรือปนปายในสถานการณที่ไมเหมาะสม

๒.๔ มักเลนหรือทํากิจกรรมเงียบๆ ไมได

๒.๕ มักเคลื่อนไหวตลอดเวลา (on to go) ราวกับติดเครื่องยนต

๒.๖ มักพูดมาก

หุนหันพลันแลน (Impulsivity)

๒.๗ มักพูดโพลง (blurts out) คําตอบกอนคําถามจะจบ

๒.๘ มักทนรอคิวไมได

๒.๙ ชอบขัดจังหวะ (interrupt) หรือ แทรก (intrude) ผูอื่นในวงสนทนาหรือการเลน

ข. เกิดกอนอายุ ๗ ป

ค. อาการเหลานี้ปรากฏใหเห็นในชีวิตประจําวันอยางนอย ๒ สถานที่หรือมากกวา เชน ที่

บาน

ที่โรงเรียน ที่ทํางาน ฯลฯ

ง. เห็นผลกระทบของอาการดังกลาวชัดเจนตอการเขาสังคม การเรียน หรือการงาน

จ. อาการดังกลาวไมไดเปนอาการของโรคจิต หรือโรคที่มีพัฒนาการผิดปกติ

9.) ตอบ D. normal grief reaction (ไมแนใจ)

จากอาการของผูปวย ที่รูสึกเศราโศกเสียใจหลังจากที่มารดาเสียชีวิต นอนไมหลับ ไป

เรียนหนังสือไมได กินไมได มา 2 สัปดาห คิดวาเปนอาการของ normal grief reaction

เพราะอาการตางๆยังเขาไมไดกับ criteria ของ MDD

MDD : ตองมีอาการมาเทากับหรือนานกวา 2 week โดยที่มีอาการเกือบทุกวัน

มี อยางนอย 5 ขอตอไปนี้ โดยตองมีขอ 1 หรือ 2 อยางใดอยางนึง

1. ซึมเศรา

2. Anhedonia ความสนใจในสิ่งตางๆลดลงอยางมาก

3. เบื่ออาหารหรือน้ําหนักลดมากกวารอยละ 5 ใน 1 เดือน

4. นอนไมหลับ หรือนอนมากกวาปกติ

5. psychomotor agitation or retardarion

6. ออนเพลีย ไมมีแรง

7. รูสึกตนเองไรคา หรือรูสึกผิด

8. สมาธิลดลง ลังเลใจ

9. คิดเรื่องการตาย หรือ ฆาตัวตาย

อาการตางๆเหลานี้ ตองทําให เสีย function !!!!!!!

Page 59: Nle step 2_2009 si115-116 and nle_step_2_2009 nctms editors cut key

SIRIRAJ

SIRIRA

J SIRIR

AJ

Normal grief reaction ตอง DDx กับ MDD แตจะไม Dx MDD หากอาการยังไมถึง 1 เดือน

Forensic

2.) ตอบ b. เบรกไมทัน

กําหนดใหระดับอางอิงคือพื้นถนน

- ถาคนขับเบรกทันแลวชนพอดี ดานหนารถที่เบรกกะทันหันจะมีลักษณะที่ต่ําลง

ดังนั้นตําแหนงกระดูกหักของผูตายและตําแหนงของกันชนรถที่บุบควรมีระดับความสูงที่

ไมตรงกัน โดยความสูงของกันชนที่บุบจะมีระดับสูงกวาความสูงของตําแหนงกระดูกที่

หัก ( ถาเทียบกับตัวเลขที่โจทยใหมาก็จะประมาณวา กันชนที่บุบสูงจากพื้น 15 cm. เทา

เดิม แตตําแหนงกระดูกหักควรจะต่ํากวานั้นเชน 10 cm. เปนตน)

- ถาคนขับเบรกไมทัน รถจะชนกับผูตายตรงๆ ดังนั้นตําแหนงที่กันชนบุบและ

ตําแหนงที่กระดูกหักควรตรงกัน จากโจทย ระดับของกันชนที่บุบและกระดูกที่หักสูงจาก

พื้น 15 cm. เทากัน ( ดังเชนรูป A ระดับสูงจากพื้นตรงกับตําแหนงที่ 1เทากัน ถา

กําหนดใหตําแหนงที่ 1 สูงจากพื้น 15 cm.) ดังนั้นขอนี้จึงสรุปไดวา คนขับเบรกไมทัน

หรืออีกกรณีนึงคือรถขับมาดวยความเรงและไมไดเบรก ดานหนารถจะลอยขึ้นขณะที่ชน

ดังนั้นตําแหนงกระดูกที่หักจะอยูสูงกวาตําแหนงที่กันชนบุบ

สวนบาดแผลถลอกอื่นๆ อาจเกิดจาก secondary injury ดังรูป B

3.) ตอบ A. carbon monoxide poisoning

deaths from carbon monoxide poisoning often result in a cherry red / pink coloration

to the skin, lips and internal body organs

CO inhalation เปนสาเหตุที่พบมากที่สุดใน smoke inhalation การตรวจดวยตาเปลาจะ

พบวา hypostasis, กลามเนื้อ, อวัยวะภายในและเลือด มีสีแดงสด (cherry red) แตเนื่องจาก

สีแดงดังกลาวสามารถเกิดจากสารอื่นก็ได เชน cyanide หรือในศพที่แชเย็นเปนเวลานาน

ดังนั้นจึงตองตรวจทางหองปฏิบัติการเพื่อยืนยันระดับของ COHb อีกครั้ง ในกรณีตรงกัน

ขาม เราอาจตรวจพบระดับของ COHb ในเลือดสูงถึงจุดที่ทํ าใหถึงแกความตายได แมวา

จะตรวจไมพบวาศพมีสีแดงก็ตาม

ในหลายรายที่ไดรับการสรุปวาตายจาก CO intoxication นั้น ในความเปนจริง

แลวการตายดังกลาวนาจะเกิดจากการผสมผสานของกระบวนการที่ซับซอนหลายๆ

กระบวนการ มากกวาจาก CO เพียงอยางเดียว เนื่องจากระดับของ COHb

ที่พบในเลือดของศพที่ตายในกองเพลิงแมจะอยูในระดับที่เปนพิษแตมักจะไมสูงถึงระดับ

ที่ทําใหตายไดโดยตัวของมันเอง โดยมักจะตํ่ากวาระดับที่พบในการตายที่เกิดจากการสูด

CO โดยตรง (เชนการตายโดยสูดควันจากทอไอเสียรถยนต)ประมาณ 20% โดยพบวา ใน

ศพที่ตายจากการไดรับ CO โดยตรงนั้น สวนใหญระดับ COHb ในเลือดจะมากกวา 70%

สวนกรณีที่ตายในกองเพลิงนั้นคาเฉลี่ยของ COHb ในเลือดจะประมาณ 57% และชวงที่

พบบอยที่สุดคือ 30-40 % ระดับของ COHb ในเลือดที่ทําใหตาย จะแปรผันตามสุขภาพ

และอายุของผูตายเปนสําคัญดังนั้น ในกรณีที่ผูตายมีโรคประจําตัวอยูกอน เชนโรคหลอด

เลือดหัวใจ ก็อาจตายไดแมจะตรวจพบ COHb ในเลือดตํ่ากวาระดับที่ทําใหตายโดยทั่วไป

ซึ่งในคนหนุมสาวที่มีสุขภาพดีนั้น ระดับ COHb ที่ทําใหตายคือ 50-60% สวนในเด็กนั้น

ระดับที่จะทําใหถึงแกความตายจะมีคาสูงขึ้นไปอีก ทั้งนี้เนื่องจากมีอัตราการเผาผลาญที่สูง

กวาผูใหญ

4.) ตอบ a. Domestic violence

จากการตรวจรางกายที่พบรอบฟกช้ําทั้งเกาและใหมมีหลายstageแสดงถึงการโดนทํา

รายหลายครั้ง และตําแหนงที่พบรอยช้ําเปนตําแหนงที่ไมนาจะเกิดจากการหกลมได จึงคิด

วาควรตอบขอนี้ 1